• Shuffle
    Toggle On
    Toggle Off
  • Alphabetize
    Toggle On
    Toggle Off
  • Front First
    Toggle On
    Toggle Off
  • Both Sides
    Toggle On
    Toggle Off
  • Read
    Toggle On
    Toggle Off
Reading...
Front

Card Range To Study

through

image

Play button

image

Play button

image

Progress

1/120

Click to flip

Use LEFT and RIGHT arrow keys to navigate between flashcards;

Use UP and DOWN arrow keys to flip the card;

H to show hint;

A reads text to speech;

120 Cards in this Set

  • Front
  • Back
Columbus and other early explorers shared which of the following objectives?

a. to prove that the world was round rather than flat

b. to find places to establish European colonies

c. to test new sailing technologies

d. to cut out intermediaries in the trade with Asia

e. to find a faster route to move soldiers and supplies to the Middle East to fight the Crusades
The correct answer is D. Answer D was the basic reason why Columbus and other explorers set out to find a direct sea route to Asia. They wanted to cut out the Arab merchants who bought the goods from Asian caravans and resold them to European merchants. Answer A is incorrect. Educated Europeans were aware that the world was not flat. Columbus was actually looking for a faster route to Asia. He misjudged the circumference of the globe and thought that by sailing west, he would reach Asia more quickly than by going around Africa. Answer B was an unintended by-product of explorations. Answer C is incorrect. Answer E sounds good, but the last Crusade was fought in the late 1200s.
Under the Treaty of Tordesillas, Spain and Portugal divided the world so that

a. Spain received North America, and Portugal received South America

b. Spain received all of North and South America except Brazil, which Portugal received

c. Spain received all non-Christian lands, and Portugal received all Christian lands

d. Spain received half of North America and South America and all of Asia, whereas Portugal received Brazil and Africa

e. Spain received all of North and South America, and Portugal received Asia and Africa
The correct answer is B. After the treaty was signed in 1494, it is true that areas changed hands frequently over the next several centuries, but you could take an educated to answer if you weren't sure of the answers. Portuguese is the language in Brazil, so the chances are that at some time Portugal ruled Brazil. This narrows your choices to answers A, B, and D You could be confident in eliminating answer A, because except for Brazil, Spanish is the national language of the rest of the countries of South America. That still leaves answers B and D There are fewer facts in answer B to be wrong, so that's a better bet than answer D It also happens to be the correct answer. Answer D is incorrect, because Spain received North and South America and India—not all of Asia—and Portugal received Brazil and Africa.
The Dutch based their claim to New Netherlands on a voyage by

a. Jacques Cartier

b. Henry Hudson

c. John Cabot

d. Peter Stuyvesant

e. Francis Drake
The Dutch based their claim to New Netherlands on a voyage by

a. Jacques Cartier

b. Henry Hudson

c. John Cabot

d. Peter Stuyvesant

e. Francis Drake



The correct answer is B. Henry Hudson, answer B, sailed for both England and the Netherlands. It was during his voyage for the Dutch in 1609 that he claimed the area for the Netherlands. Answer A, Jacques Cartier, sailed for his native France and claimed the area of Labrador and the St. Lawrence River Valley for France. Answer C, John Cabot, an Italian sailing for England, reached the coasts of Newfoundland and Nova Scotia. Answer D, Peter Stuyvesant, was the governor of the Dutch colony of New Netherlands when it fell to the English in 1664. Answer E, Francis Drake, was a privateer in the service of England during the reign of Queen Elizabeth.
Which of the following would have been least likely to be interested in English efforts to explore and settle North America?

a. Puritan family

b. London merchant

c. unemployed farm laborer

d. maid servant

e. landowning English aristocrat
The correct answer is E. You know answer A isn't the least likely, because Puritans established Massachusetts Bay Colony. Answer B is always someone who was likely to be interested, because London merchants founded the joint-stock companies that underwrote Massachusetts Bay and other colonies like Jamestown. Answers C and D make sense, because an unemployed farm laborer and a maid might well think there would be work in a new, unsettled land. Both were sources of the class of indentured servants who immigrated to the colonies. Answer E would be the one person who would be the least interested in the colonies. The average aristocrat himself was content with his title; his money was tied up in land, so he wouldn't have any to invest; and he would see no reason to emigrate. The exception is Lord Baltimore, the Catholic aristocrat who received a land grant to establish a colony as a haven for Roman Catholics; however, the question says "least likely," so you're looking for a generalization. (If you are thinking of William Penn, he wasn't a landed aristocrat.)
"The English who came first to this country were but a handful of people, forlorn, poor, and distressed. My father was then sachem, he relieved their distresses in the most kind and hospitable manner. He gave them land to plant and to build upon."
The Native American father and son referenced by this quotation are



a. Powhatan and Pocahantas

b. Massasoit and Metacom

c. Tecumseh and the Prophet

d. Powhatan and Massasoit

e. Tecumseh and Metacom
The correct answer is B. Massasoit helped the Pilgrims, who returned the honor by waging war against his son Metacom, who succeeded him, and the Wampanoags who had first aided them. By 1670, a full-scale war had erupted between the colonists and an alliance of Native American groups, which lasted until 1676. The colonists called Metacom, King Philip, and the war is often referred to as King Philip's War. Answer A is incorrect, because Pocahantas was Powhatan's daughter. Answer C is incorrect, because they were brothers and urged their fellow Native Americans not to sell land to European Americans. Answers D and E list unrelated Native Americans.
One system that developed to attract European immigrant was the

a. quitrent

b. peonage

c. headright

d. sharecropping

e. tenancy
The correct answer is C. The system of giving land to each immigrant (head) entering a colony was called the headright. The headright entitled the person to lay claim to a certain number of acres of land on which no one was already living. The immigrant would have to plant a crop, build a shelter, and live on the land. Initially, the land was free, but after a time, the proprietors began to charge a tax, called a quitrent, answer A. Answer B is a form of economic bondage. Answers D and E as applied to the situation of freed slaves in the South after the Civil War and their descendants well into the twentieth century was peonage. Answer D is a system that developed after the Civil War in the South in which former slaves farmed land for a share of the profits at harvest time in exchange for food, shelter, clothing, seeds, and farm implements. The freed slaves were charged so much for the materials that they never earned enough to leave the farm. Another economic system that developed after the Civil War in the South, answer E is the leasing or renting of land. With this system, too, former slaves did not make enough to be free of debt.
All of the following are true about Africans in the North American colonies in the 1600s EXCEPT

a. the first Africans were brought as indentured servants

b. laws enslaving Africans were passed in Virginia and Maryland in the 1660s

c. there were enslaved Africans in the Middle Colonies

d. the number of enslaved Africans in the Southern Colonies was small because plantation agriculture had not yet developed

e. New Englanders built a prosperous business based on slave trading
The correct answer is D. By the 1680s, tobacco and rice cultivation dominated the economies of the Southern Colonies. Both required large parcels of land to be profitable and a cheap and steady source of labor. Indentured servants were becoming harder to find and enslaved Africans appeared to provide a limitless supply of workers. As plantation agriculture became an important part of the economy, Virginia and Maryland passed enslavement laws, answer B, in the 1660s.
All of the following contributed to the establishment of slavery in the colonies EXCEPT

a. the decline in the number of Europeans who were willing to become indentured servants

b. Bacon's Rebellion

c. the importance of tobacco to the economy of Southern colonies

d. the establishment of the Royal Africa Company

e. the importance of the triangular trade to the economy of New England
The correct answer is E. Don't be confused by causes and effects. Answer E is an effect, or result, of the establishment of slavery, not a cause. Answer B may have confused you. Bacon's Rebellion was a result of former indentured servants on the Virginia frontier clashing with the powerful coastal planters. Within 20 years after the uprising's collapse, slavery had become well-established in Virginia. Historians reason that coastal planters saw the danger to the status quo inherent in a rising class of new, small landowners and moved to ensure that they did not become so plentiful that they would seek to seize political power from the established planters. Slaves became the planters' solution.
The pamphlet The Discovery of Witches would most likely have found an audience in which of the following colonies?

a. Rhode Island

b. Massachusetts Bay

c. Pennsylvania

d. Maryland

e. Virginia
The correct answer is B. The Discovery of Witches was written by Matthew Hopkins, a witchfinder in Massachusetts Bay Colony. Witches were of grave concern to Puritans. Think Salem witch trials in the 1692.

Return to Section Report Question: 10 of 120
Jump to question: 1 (answered) 2 (answered) 3 (answered) 4 (answered) 5 (answered) 6 (answered) 7 (answered) 8 (answered) 9 (answered) 10 (answered) 11 (answered) 12 (answered) 13 (answered) 14 (answered) 15 (answered) 16 (answered) 17 (answered) 18 (answered) 19 (answered) 20 (answered) 21 (answered) 22 (answered) 23 (answered) 24 (answered) 25 (answered) 26 (answered) 27 (answered) 28 (answered) 29 (answered) 30 (answered) 31 (answered) 32 (answered) 33 (answered) 34 (answered) 35 (answered) 36 (answered) 37 (answered) 38 (answered) 39 (answered) 40 (answered) 41 (answered) 42 (answered) 43 (answered) 44 (answered) 45 (answered) 46 (answered) 47 (answered) 48 (answered) 49 (answered) 50 (answered) 51 (answered) 52 (answered) 53 (answered) 54 (answered) 55 (answered) 56 (answered) 57 (answered) 58 (answered) 59 (answered) 60 (answered) 61 (answered) 62 (answered) 63 (answered) 64 (answered) 65 (answered) 66 (answered) 67 (answered) 68 (answered) 69 (answered) 70 (answered) 71 (answered) 72 (answered) 73 (answered) 74 (answered) 75 (answered) 76 (answered) 77 (answered) 78 (answered) 79 (answered) 80 (answered) 81 (answered) 82 (answered) 83 (answered) 84 (answered) 85 (answered) 86 (answered) 87 (answered) 88 (answered) 89 (answered) 90 (answered) 91 (answered) 92 (answered) 93 (answered) 94 (answered) 95 (answered) 96 (answered) 97 (answered) 98 (answered) 99 (answered)100 (answered)101 (answered)102 (answered)103 (answered)104 (answered)105 (answered)106 (answered)107 (answered)108 (answered)109 (answered)110 (answered)111 (answered)112 (answered)113 (answered)114 (answered)115 (answered)116 (answered)117 (answered)118 (answered)119 (answered)120 (answered)


Adoption of which of the following was an admission by Congregational leaders that the church was no longer a central factor in people's lives?

a. Second Mayflower Compact

b. Holy Experiment

c. Halfway Covenant

d. Act of Religious Toleration

e. Fundamental Orders of Connecticut



The correct answer is C. Answer A sounds good, but never existed. Answer B is the term William Penn gave to Pennsylvania when he founded it. Answer D is a law passed by the Maryland legislature, the first religious toleration statute in the colonies. Answer E was the first constitution drawn up in the colonies.

Return to Section Report Question: 11 of 120
Jump to question: 1 (answered) 2 (answered) 3 (answered) 4 (answered) 5 (answered) 6 (answered) 7 (answered) 8 (answered) 9 (answered) 10 (answered) 11 (answered) 12 (answered) 13 (answered) 14 (answered) 15 (answered) 16 (answered) 17 (answered) 18 (answered) 19 (answered) 20 (answered) 21 (answered) 22 (answered) 23 (answered) 24 (answered) 25 (answered) 26 (answered) 27 (answered) 28 (answered) 29 (answered) 30 (answered) 31 (answered) 32 (answered) 33 (answered) 34 (answered) 35 (answered) 36 (answered) 37 (answered) 38 (answered) 39 (answered) 40 (answered) 41 (answered) 42 (answered) 43 (answered) 44 (answered) 45 (answered) 46 (answered) 47 (answered) 48 (answered) 49 (answered) 50 (answered) 51 (answered) 52 (answered) 53 (answered) 54 (answered) 55 (answered) 56 (answered) 57 (answered) 58 (answered) 59 (answered) 60 (answered) 61 (answered) 62 (answered) 63 (answered) 64 (answered) 65 (answered) 66 (answered) 67 (answered) 68 (answered) 69 (answered) 70 (answered) 71 (answered) 72 (answered) 73 (answered) 74 (answered) 75 (answered) 76 (answered) 77 (answered) 78 (answered) 79 (answered) 80 (answered) 81 (answered) 82 (answered) 83 (answered) 84 (answered) 85 (answered) 86 (answered) 87 (answered) 88 (answered) 89 (answered) 90 (answered) 91 (answered) 92 (answered) 93 (answered) 94 (answered) 95 (answered) 96 (answered) 97 (answered) 98 (answered) 99 (answered)100 (answered)101 (answered)102 (answered)103 (answered)104 (answered)105 (answered)106 (answered)107 (answered)108 (answered)109 (answered)110 (answered)111 (answered)112 (answered)113 (answered)114 (answered)115 (answered)116 (answered)117 (answered)118 (answered)119 (answered)120 (answered)


As far as the British government was concerned, the reason for the founding of Georgia was

a. to establish a colony for the poor from English debtors' prisons

b. as a haven for religious dissenters

c. to return a profit for the Crown

d. to establish a colony based on slavery from its beginning

e. to establish a buffer between South Carolina and Spanish Florida



The correct answer is E. The key phrase in the question is "as far as the British government." Answer A was a reason of James Oglethorpe, one of the eight original proprietors, but it was not the reason the government was interested. Don't confuse Georgia with Plymouth, Massachusetts Bay, Maryland, or Pennsylvania, answer B. Answer C is incorrect, because Georgia was a proprietary colony. Answer D is the opposite of what was true; the proprietors banned slavery at the inception of the colony, but later gave in to angry colonists who said they could not turn a profit without the use of slaves.

Return to Section Report Question: 12 of 120
Jump to question: 1 (answered) 2 (answered) 3 (answered) 4 (answered) 5 (answered) 6 (answered) 7 (answered) 8 (answered) 9 (answered) 10 (answered) 11 (answered) 12 (answered) 13 (answered) 14 (answered) 15 (answered) 16 (answered) 17 (answered) 18 (answered) 19 (answered) 20 (answered) 21 (answered) 22 (answered) 23 (answered) 24 (answered) 25 (answered) 26 (answered) 27 (answered) 28 (answered) 29 (answered) 30 (answered) 31 (answered) 32 (answered) 33 (answered) 34 (answered) 35 (answered) 36 (answered) 37 (answered) 38 (answered) 39 (answered) 40 (answered) 41 (answered) 42 (answered) 43 (answered) 44 (answered) 45 (answered) 46 (answered) 47 (answered) 48 (answered) 49 (answered) 50 (answered) 51 (answered) 52 (answered) 53 (answered) 54 (answered) 55 (answered) 56 (answered) 57 (answered) 58 (answered) 59 (answered) 60 (answered) 61 (answered) 62 (answered) 63 (answered) 64 (answered) 65 (answered) 66 (answered) 67 (answered) 68 (answered) 69 (answered) 70 (answered) 71 (answered) 72 (answered) 73 (answered) 74 (answered) 75 (answered) 76 (answered) 77 (answered) 78 (answered) 79 (answered) 80 (answered) 81 (answered) 82 (answered) 83 (answered) 84 (answered) 85 (answered) 86 (answered) 87 (answered) 88 (answered) 89 (answered) 90 (answered) 91 (answered) 92 (answered) 93 (answered) 94 (answered) 95 (answered) 96 (answered) 97 (answered) 98 (answered) 99 (answered)100 (answered)101 (answered)102 (answered)103 (answered)104 (answered)105 (answered)106 (answered)107 (answered)108 (answered)109 (answered)110 (answered)111 (answered)112 (answered)113 (answered)114 (answered)115 (answered)116 (answered)117 (answered)118 (answered)119 (answered)120 (answered)


The large urban centers of the North American colonies were located mostly

a. in the South

b. in the Middle Colonies

c. in the interior

d. in New England

e. in valleys



The correct answer is B. The Middle Colonies were more welcoming to immigrants than New England, answer D, was—both the environment and in policies toward religion. The weather was milder and the land, more fertile. A larger population meant not only a larger area under settlement, but also an increase in the number of townspeople per village, town, and city. Answer A is incorrect. Most of the southern population was rural, living either on small farms or on plantations. Answer C is incorrect. Relatively speaking, little of the interior had been settled by 1776. The majority of the colonial population still lived along the coast. Answer E is incorrect. The majority of the population lived along the coastal plain.

Return to Section Report Question: 13 of 120
Jump to question: 1 (answered) 2 (answered) 3 (answered) 4 (answered) 5 (answered) 6 (answered) 7 (answered) 8 (answered) 9 (answered) 10 (answered) 11 (answered) 12 (answered) 13 (answered) 14 (answered) 15 (answered) 16 (answered) 17 (answered) 18 (answered) 19 (answered) 20 (answered) 21 (answered) 22 (answered) 23 (answered) 24 (answered) 25 (answered) 26 (answered) 27 (answered) 28 (answered) 29 (answered) 30 (answered) 31 (answered) 32 (answered) 33 (answered) 34 (answered) 35 (answered) 36 (answered) 37 (answered) 38 (answered) 39 (answered) 40 (answered) 41 (answered) 42 (answered) 43 (answered) 44 (answered) 45 (answered) 46 (answered) 47 (answered) 48 (answered) 49 (answered) 50 (answered) 51 (answered) 52 (answered) 53 (answered) 54 (answered) 55 (answered) 56 (answered) 57 (answered) 58 (answered) 59 (answered) 60 (answered) 61 (answered) 62 (answered) 63 (answered) 64 (answered) 65 (answered) 66 (answered) 67 (answered) 68 (answered) 69 (answered) 70 (answered) 71 (answered) 72 (answered) 73 (answered) 74 (answered) 75 (answered) 76 (answered) 77 (answered) 78 (answered) 79 (answered) 80 (answered) 81 (answered) 82 (answered) 83 (answered) 84 (answered) 85 (answered) 86 (answered) 87 (answered) 88 (answered) 89 (answered) 90 (answered) 91 (answered) 92 (answered) 93 (answered) 94 (answered) 95 (answered) 96 (answered) 97 (answered) 98 (answered) 99 (answered)100 (answered)101 (answered)102 (answered)103 (answered)104 (answered)105 (answered)106 (answered)107 (answered)108 (answered)109 (answered)110 (answered)111 (answered)112 (answered)113 (answered)114 (answered)115 (answered)116 (answered)117 (answered)118 (answered)119 (answered)120 (answered)


An unintended result of the First Great Awakening was

a. a new level of intolerance toward all religions

b. a more democratic outlook among colonists and less reliance on traditional authority and social class

c. a general lessening of interest in religion

d. that evangelism delayed the establishment of schools and colleges

e. a dramatic increase in membership in traditional churches



The correct answer is B. There is no evidence to support answer A. Common sense eliminates answer C, since the question is about a revival of interest in religion. Answer D is the opposite of what occurred. A number of colleges were founded in this period as a result of the idea that people could think for themselves. Answer E is incorrect; many people left the older churches to join newer ones like Methodism and Presbyterianism.

Return to Section Report Question: 14 of 120
Jump to question: 1 (answered) 2 (answered) 3 (answered) 4 (answered) 5 (answered) 6 (answered) 7 (answered) 8 (answered) 9 (answered) 10 (answered) 11 (answered) 12 (answered) 13 (answered) 14 (answered) 15 (answered) 16 (answered) 17 (answered) 18 (answered) 19 (answered) 20 (answered) 21 (answered) 22 (answered) 23 (answered) 24 (answered) 25 (answered) 26 (answered) 27 (answered) 28 (answered) 29 (answered) 30 (answered) 31 (answered) 32 (answered) 33 (answered) 34 (answered) 35 (answered) 36 (answered) 37 (answered) 38 (answered) 39 (answered) 40 (answered) 41 (answered) 42 (answered) 43 (answered) 44 (answered) 45 (answered) 46 (answered) 47 (answered) 48 (answered) 49 (answered) 50 (answered) 51 (answered) 52 (answered) 53 (answered) 54 (answered) 55 (answered) 56 (answered) 57 (answered) 58 (answered) 59 (answered) 60 (answered) 61 (answered) 62 (answered) 63 (answered) 64 (answered) 65 (answered) 66 (answered) 67 (answered) 68 (answered) 69 (answered) 70 (answered) 71 (answered) 72 (answered) 73 (answered) 74 (answered) 75 (answered) 76 (answered) 77 (answered) 78 (answered) 79 (answered) 80 (answered) 81 (answered) 82 (answered) 83 (answered) 84 (answered) 85 (answered) 86 (answered) 87 (answered) 88 (answered) 89 (answered) 90 (answered) 91 (answered) 92 (answered) 93 (answered) 94 (answered) 95 (answered) 96 (answered) 97 (answered) 98 (answered) 99 (answered)100 (answered)101 (answered)102 (answered)103 (answered)104 (answered)105 (answered)106 (answered)107 (answered)108 (answered)109 (answered)110 (answered)111 (answered)112 (answered)113 (answered)114 (answered)115 (answered)116 (answered)117 (answered)118 (answered)119 (answered)120 (answered)


Which of the following would be least likely to be found in a colonial gentleman's library in the mid-1700s?

a. writings of John Locke

b. book of sermons

c. Latin version of the Aenid

d. novel written by a colonial author

e. collection of poetry by an English poet



The correct answer is D. While colonists may have been developing their own uniquely American view of politics, their literary tastes were still very much influenced by what the British wrote and read, which included religious writings. The American novel did not come into its own until the early national period with the works of James Fenimore Cooper.

Return to Section Report Question: 15 of 120
Jump to question: 1 (answered) 2 (answered) 3 (answered) 4 (answered) 5 (answered) 6 (answered) 7 (answered) 8 (answered) 9 (answered) 10 (answered) 11 (answered) 12 (answered) 13 (answered) 14 (answered) 15 (answered) 16 (answered) 17 (answered) 18 (answered) 19 (answered) 20 (answered) 21 (answered) 22 (answered) 23 (answered) 24 (answered) 25 (answered) 26 (answered) 27 (answered) 28 (answered) 29 (answered) 30 (answered) 31 (answered) 32 (answered) 33 (answered) 34 (answered) 35 (answered) 36 (answered) 37 (answered) 38 (answered) 39 (answered) 40 (answered) 41 (answered) 42 (answered) 43 (answered) 44 (answered) 45 (answered) 46 (answered) 47 (answered) 48 (answered) 49 (answered) 50 (answered) 51 (answered) 52 (answered) 53 (answered) 54 (answered) 55 (answered) 56 (answered) 57 (answered) 58 (answered) 59 (answered) 60 (answered) 61 (answered) 62 (answered) 63 (answered) 64 (answered) 65 (answered) 66 (answered) 67 (answered) 68 (answered) 69 (answered) 70 (answered) 71 (answered) 72 (answered) 73 (answered) 74 (answered) 75 (answered) 76 (answered) 77 (answered) 78 (answered) 79 (answered) 80 (answered) 81 (answered) 82 (answered) 83 (answered) 84 (answered) 85 (answered) 86 (answered) 87 (answered) 88 (answered) 89 (answered) 90 (answered) 91 (answered) 92 (answered) 93 (answered) 94 (answered) 95 (answered) 96 (answered) 97 (answered) 98 (answered) 99 (answered)100 (answered)101 (answered)102 (answered)103 (answered)104 (answered)105 (answered)106 (answered)107 (answered)108 (answered)109 (answered)110 (answered)111 (answered)112 (answered)113 (answered)114 (answered)115 (answered)116 (answered)117 (answered)118 (answered)119 (answered)120 (answered)


The most important port and the largest city in the North American colonies by 1776 was

a. Charleston

b. Baltimore

c. Philadelphia

d. New York

e. Boston



The correct answer is C. Answer A, Charleston, could be ruled out, because it was in a southern colony and there was less population in that region than in the Middle or New England colonies. For the same reason, rule out answer B, Baltimore. That would leave you with three good choices, but the long, cold winters could help you rule out Boston, answer E. Remember that the delegates to the Continental Congress chose Philadelphia as the site of their meetings, so it must have been a big and important city. Pennsylvania by the 1700s had become known as the breadbasket of the colonies, which means that it grew a lot of food, more than its people used, which means it had to ship it from someplace—Philadelphia.

Return to Section Report Question: 16 of 120
Jump to question: 1 (answered) 2 (answered) 3 (answered) 4 (answered) 5 (answered) 6 (answered) 7 (answered) 8 (answered) 9 (answered) 10 (answered) 11 (answered) 12 (answered) 13 (answered) 14 (answered) 15 (answered) 16 (answered) 17 (answered) 18 (answered) 19 (answered) 20 (answered) 21 (answered) 22 (answered) 23 (answered) 24 (answered) 25 (answered) 26 (answered) 27 (answered) 28 (answered) 29 (answered) 30 (answered) 31 (answered) 32 (answered) 33 (answered) 34 (answered) 35 (answered) 36 (answered) 37 (answered) 38 (answered) 39 (answered) 40 (answered) 41 (answered) 42 (answered) 43 (answered) 44 (answered) 45 (answered) 46 (answered) 47 (answered) 48 (answered) 49 (answered) 50 (answered) 51 (answered) 52 (answered) 53 (answered) 54 (answered) 55 (answered) 56 (answered) 57 (answered) 58 (answered) 59 (answered) 60 (answered) 61 (answered) 62 (answered) 63 (answered) 64 (answered) 65 (answered) 66 (answered) 67 (answered) 68 (answered) 69 (answered) 70 (answered) 71 (answered) 72 (answered) 73 (answered) 74 (answered) 75 (answered) 76 (answered) 77 (answered) 78 (answered) 79 (answered) 80 (answered) 81 (answered) 82 (answered) 83 (answered) 84 (answered) 85 (answered) 86 (answered) 87 (answered) 88 (answered) 89 (answered) 90 (answered) 91 (answered) 92 (answered) 93 (answered) 94 (answered) 95 (answered) 96 (answered) 97 (answered) 98 (answered) 99 (answered)100 (answered)101 (answered)102 (answered)103 (answered)104 (answered)105 (answered)106 (answered)107 (answered)108 (answered)109 (answered)110 (answered)111 (answered)112 (answered)113 (answered)114 (answered)115 (answered)116 (answered)117 (answered)118 (answered)119 (answered)120 (answered)


The only colony in which enslaved African Americans outnumbered European Americans in the mid-1700s was

a. Maryland

b. Virginia

c. North Carolina

d. South Carolina

e. Georgia



The correct answer is D. If you didn't know this answer, you could at least eliminate Georgia, because it was not founded until 1732 and it originally banned slavery. South Carolina's major export was rice, not tobacco, and the unhealthy work of cultivating rice in wet, bug-infested fields attracted few indentured servants. As a result, enslaved Africans soon became the main source of workers and a large slave population developed. By the 1730s, the ratio of African Americans to European Americans was 2 to 1.

Return to Section Report Question: 17 of 120
Jump to question: 1 (answered) 2 (answered) 3 (answered) 4 (answered) 5 (answered) 6 (answered) 7 (answered) 8 (answered) 9 (answered) 10 (answered) 11 (answered) 12 (answered) 13 (answered) 14 (answered) 15 (answered) 16 (answered) 17 (answered) 18 (answered) 19 (answered) 20 (answered) 21 (answered) 22 (answered) 23 (answered) 24 (answered) 25 (answered) 26 (answered) 27 (answered) 28 (answered) 29 (answered) 30 (answered) 31 (answered) 32 (answered) 33 (answered) 34 (answered) 35 (answered) 36 (answered) 37 (answered) 38 (answered) 39 (answered) 40 (answered) 41 (answered) 42 (answered) 43 (answered) 44 (answered) 45 (answered) 46 (answered) 47 (answered) 48 (answered) 49 (answered) 50 (answered) 51 (answered) 52 (answered) 53 (answered) 54 (answered) 55 (answered) 56 (answered) 57 (answered) 58 (answered) 59 (answered) 60 (answered) 61 (answered) 62 (answered) 63 (answered) 64 (answered) 65 (answered) 66 (answered) 67 (answered) 68 (answered) 69 (answered) 70 (answered) 71 (answered) 72 (answered) 73 (answered) 74 (answered) 75 (answered) 76 (answered) 77 (answered) 78 (answered) 79 (answered) 80 (answered) 81 (answered) 82 (answered) 83 (answered) 84 (answered) 85 (answered) 86 (answered) 87 (answered) 88 (answered) 89 (answered) 90 (answered) 91 (answered) 92 (answered) 93 (answered) 94 (answered) 95 (answered) 96 (answered) 97 (answered) 98 (answered) 99 (answered)100 (answered)101 (answered)102 (answered)103 (answered)104 (answered)105 (answered)106 (answered)107 (answered)108 (answered)109 (answered)110 (answered)111 (answered)112 (answered)113 (answered)114 (answered)115 (answered)116 (answered)117 (answered)118 (answered)119 (answered)120 (answered)


According to which of the following theories were colonies primarily sources of raw materials for their home countries and markets for the latter's manufactured goods?

a. protectionism

b. capitalism

c. imperialism

d. mercantilism

e. socialism



The correct answer is D. Answer A refers to an economic theory that promotes high tariffs on imports to protect internal manufacturers and farmers from competition in their own national market. Answer B is an economic system in which the means of production are owned by individuals and manipulated for private profit. Answer C is a policy under which a powerful nation attempts to create an empire by dominating the economies, political systems, and cultural institutions of less powerful nations. Answer E is an economic system in which the means of production are owned by the public for the benefit of the public.

Return to Section Report Question: 18 of 120
Jump to question: 1 (answered) 2 (answered) 3 (answered) 4 (answered) 5 (answered) 6 (answered) 7 (answered) 8 (answered) 9 (answered) 10 (answered) 11 (answered) 12 (answered) 13 (answered) 14 (answered) 15 (answered) 16 (answered) 17 (answered) 18 (answered) 19 (answered) 20 (answered) 21 (answered) 22 (answered) 23 (answered) 24 (answered) 25 (answered) 26 (answered) 27 (answered) 28 (answered) 29 (answered) 30 (answered) 31 (answered) 32 (answered) 33 (answered) 34 (answered) 35 (answered) 36 (answered) 37 (answered) 38 (answered) 39 (answered) 40 (answered) 41 (answered) 42 (answered) 43 (answered) 44 (answered) 45 (answered) 46 (answered) 47 (answered) 48 (answered) 49 (answered) 50 (answered) 51 (answered) 52 (answered) 53 (answered) 54 (answered) 55 (answered) 56 (answered) 57 (answered) 58 (answered) 59 (answered) 60 (answered) 61 (answered) 62 (answered) 63 (answered) 64 (answered) 65 (answered) 66 (answered) 67 (answered) 68 (answered) 69 (answered) 70 (answered) 71 (answered) 72 (answered) 73 (answered) 74 (answered) 75 (answered) 76 (answered) 77 (answered) 78 (answered) 79 (answered) 80 (answered) 81 (answered) 82 (answered) 83 (answered) 84 (answered) 85 (answered) 86 (answered) 87 (answered) 88 (answered) 89 (answered) 90 (answered) 91 (answered) 92 (answered) 93 (answered) 94 (answered) 95 (answered) 96 (answered) 97 (answered) 98 (answered) 99 (answered)100 (answered)101 (answered)102 (answered)103 (answered)104 (answered)105 (answered)106 (answered)107 (answered)108 (answered)109 (answered)110 (answered)111 (answered)112 (answered)113 (answered)114 (answered)115 (answered)116 (answered)117 (answered)118 (answered)119 (answered)120 (answered)


"Great talkers, little doers."
Who probably wrote this aphorism?



a. Phillis Wheatley

b. Benjamin Franklin

c. Abigail Adams

d. Thomas Jefferson

e. John Dickinson



The correct answer is B. This aphorism, or saying, is from one of the issues of Franklin's Poor Richard's Almanac. Almanacs were popular forms of entertainment in the 1700s. Franklin in the guise of Poor Richard dispensed useful weather and crop information for sailors and farmers, political satire, and homey wisdom. Answer A, Phillis Wheatley, a former slave, was the first African American poet published in the colonies. Answer C, Abigail Adams, wife of John Adams, the second president of the United States, has become famous for her correspondence with her husband while he was a delegate to the Continental Congress. Answer D, Thomas Jefferson, is a good distracter, since one of the things he is known for is writing—the Declaration of Independence. Answer E, John Dickinson, is also a good distracter, because he was the author of Letters from a Farmer in Pennsylvania.

Return to Section Report Question: 19 of 120
Jump to question: 1 (answered) 2 (answered) 3 (answered) 4 (answered) 5 (answered) 6 (answered) 7 (answered) 8 (answered) 9 (answered) 10 (answered) 11 (answered) 12 (answered) 13 (answered) 14 (answered) 15 (answered) 16 (answered) 17 (answered) 18 (answered) 19 (answered) 20 (answered) 21 (answered) 22 (answered) 23 (answered) 24 (answered) 25 (answered) 26 (answered) 27 (answered) 28 (answered) 29 (answered) 30 (answered) 31 (answered) 32 (answered) 33 (answered) 34 (answered) 35 (answered) 36 (answered) 37 (answered) 38 (answered) 39 (answered) 40 (answered) 41 (answered) 42 (answered) 43 (answered) 44 (answered) 45 (answered) 46 (answered) 47 (answered) 48 (answered) 49 (answered) 50 (answered) 51 (answered) 52 (answered) 53 (answered) 54 (answered) 55 (answered) 56 (answered) 57 (answered) 58 (answered) 59 (answered) 60 (answered) 61 (answered) 62 (answered) 63 (answered) 64 (answered) 65 (answered) 66 (answered) 67 (answered) 68 (answered) 69 (answered) 70 (answered) 71 (answered) 72 (answered) 73 (answered) 74 (answered) 75 (answered) 76 (answered) 77 (answered) 78 (answered) 79 (answered) 80 (answered) 81 (answered) 82 (answered) 83 (answered) 84 (answered) 85 (answered) 86 (answered) 87 (answered) 88 (answered) 89 (answered) 90 (answered) 91 (answered) 92 (answered) 93 (answered) 94 (answered) 95 (answered) 96 (answered) 97 (answered) 98 (answered) 99 (answered)100 (answered)101 (answered)102 (answered)103 (answered)104 (answered)105 (answered)106 (answered)107 (answered)108 (answered)109 (answered)110 (answered)111 (answered)112 (answered)113 (answered)114 (answered)115 (answered)116 (answered)117 (answered)118 (answered)119 (answered)120 (answered)


Native Americans allied with the French in the French and Indian War because

a. the French paid better prices for Native American furs

b. the French were more interested in the fur trade than in colonization

c. the British refused to accept Native American aid

d. the French promised in any peace treaty to limit British settlements to east of the Appalachians

e. the British could not control the colonists who raided Native American villages



The correct answer is B. One of the main areas of conflict was the movement of more and more British colonists west of the Appalachians. The Native Americans sided with the French who held the territory as part of New France and were attempting to rout the settlers.

Return to Section Report Question: 20 of 120
Jump to question: 1 (answered) 2 (answered) 3 (answered) 4 (answered) 5 (answered) 6 (answered) 7 (answered) 8 (answered) 9 (answered) 10 (answered) 11 (answered) 12 (answered) 13 (answered) 14 (answered) 15 (answered) 16 (answered) 17 (answered) 18 (answered) 19 (answered) 20 (answered) 21 (answered) 22 (answered) 23 (answered) 24 (answered) 25 (answered) 26 (answered) 27 (answered) 28 (answered) 29 (answered) 30 (answered) 31 (answered) 32 (answered) 33 (answered) 34 (answered) 35 (answered) 36 (answered) 37 (answered) 38 (answered) 39 (answered) 40 (answered) 41 (answered) 42 (answered) 43 (answered) 44 (answered) 45 (answered) 46 (answered) 47 (answered) 48 (answered) 49 (answered) 50 (answered) 51 (answered) 52 (answered) 53 (answered) 54 (answered) 55 (answered) 56 (answered) 57 (answered) 58 (answered) 59 (answered) 60 (answered) 61 (answered) 62 (answered) 63 (answered) 64 (answered) 65 (answered) 66 (answered) 67 (answered) 68 (answered) 69 (answered) 70 (answered) 71 (answered) 72 (answered) 73 (answered) 74 (answered) 75 (answered) 76 (answered) 77 (answered) 78 (answered) 79 (answered) 80 (answered) 81 (answered) 82 (answered) 83 (answered) 84 (answered) 85 (answered) 86 (answered) 87 (answered) 88 (answered) 89 (answered) 90 (answered) 91 (answered) 92 (answered) 93 (answered) 94 (answered) 95 (answered) 96 (answered) 97 (answered) 98 (answered) 99 (answered)100 (answered)101 (answered)102 (answered)103 (answered)104 (answered)105 (answered)106 (answered)107 (answered)108 (answered)109 (answered)110 (answered)111 (answered)112 (answered)113 (answered)114 (answered)115 (answered)116 (answered)117 (answered)118 (answered)119 (answered)120 (answered)


As a result of Pontiac's Rebellion,

a. the French and Indian began

b. the Native American alliance was crushed at the Battle of Fallen Timbers

c. the British issued the Proclamation of 1763

d. the French closed the border with the British colonies

e. the British government negotiated the Treaty of Greenville



The correct answer is C. Answer A is incorrect; the results of the French and Indian led to Pontiac's Rebellion. Answers B and E are related, but belong to the early national period of U.S. history. President George Washington sent troops against a Native American uprising in the Old Northwest. The Native Americans were beaten at Fallen Timbers and forced to agree to the Treaty of Greenville. Answer D is illogical; the French were gone by 1763.

Return to Section Report Question: 21 of 120
Jump to question: 1 (answered) 2 (answered) 3 (answered) 4 (answered) 5 (answered) 6 (answered) 7 (answered) 8 (answered) 9 (answered) 10 (answered) 11 (answered) 12 (answered) 13 (answered) 14 (answered) 15 (answered) 16 (answered) 17 (answered) 18 (answered) 19 (answered) 20 (answered) 21 (answered) 22 (answered) 23 (answered) 24 (answered) 25 (answered) 26 (answered) 27 (answered) 28 (answered) 29 (answered) 30 (answered) 31 (answered) 32 (answered) 33 (answered) 34 (answered) 35 (answered) 36 (answered) 37 (answered) 38 (answered) 39 (answered) 40 (answered) 41 (answered) 42 (answered) 43 (answered) 44 (answered) 45 (answered) 46 (answered) 47 (answered) 48 (answered) 49 (answered) 50 (answered) 51 (answered) 52 (answered) 53 (answered) 54 (answered) 55 (answered) 56 (answered) 57 (answered) 58 (answered) 59 (answered) 60 (answered) 61 (answered) 62 (answered) 63 (answered) 64 (answered) 65 (answered) 66 (answered) 67 (answered) 68 (answered) 69 (answered) 70 (answered) 71 (answered) 72 (answered) 73 (answered) 74 (answered) 75 (answered) 76 (answered) 77 (answered) 78 (answered) 79 (answered) 80 (answered) 81 (answered) 82 (answered) 83 (answered) 84 (answered) 85 (answered) 86 (answered) 87 (answered) 88 (answered) 89 (answered) 90 (answered) 91 (answered) 92 (answered) 93 (answered) 94 (answered) 95 (answered) 96 (answered) 97 (answered) 98 (answered) 99 (answered)100 (answered)101 (answered)102 (answered)103 (answered)104 (answered)105 (answered)106 (answered)107 (answered)108 (answered)109 (answered)110 (answered)111 (answered)112 (answered)113 (answered)114 (answered)115 (answered)116 (answered)117 (answered)118 (answered)119 (answered)120 (answered)


The most popular subject for a painting among upper- and middle-class people in the colonial period was

a. a landscape

b. an historical theme

c. a portrait

d. everyday life

e. a religious subject



The correct answer is C. Like British aristocrats, wealthy landowners, merchants, and professionals as well as wealthy artisans wanted themselves and their family members captured for posterity. Paintings of landscapes and everyday life, answers A and D, became very popular during the romantic period in the 1800s. Answer B, historical themes, were popular so long as the subjects glorified the new nation. The popularity of religious themes, answer E, was somewhat limited.

Return to Section Report Question: 22 of 120
Jump to question: 1 (answered) 2 (answered) 3 (answered) 4 (answered) 5 (answered) 6 (answered) 7 (answered) 8 (answered) 9 (answered) 10 (answered) 11 (answered) 12 (answered) 13 (answered) 14 (answered) 15 (answered) 16 (answered) 17 (answered) 18 (answered) 19 (answered) 20 (answered) 21 (answered) 22 (answered) 23 (answered) 24 (answered) 25 (answered) 26 (answered) 27 (answered) 28 (answered) 29 (answered) 30 (answered) 31 (answered) 32 (answered) 33 (answered) 34 (answered) 35 (answered) 36 (answered) 37 (answered) 38 (answered) 39 (answered) 40 (answered) 41 (answered) 42 (answered) 43 (answered) 44 (answered) 45 (answered) 46 (answered) 47 (answered) 48 (answered) 49 (answered) 50 (answered) 51 (answered) 52 (answered) 53 (answered) 54 (answered) 55 (answered) 56 (answered) 57 (answered) 58 (answered) 59 (answered) 60 (answered) 61 (answered) 62 (answered) 63 (answered) 64 (answered) 65 (answered) 66 (answered) 67 (answered) 68 (answered) 69 (answered) 70 (answered) 71 (answered) 72 (answered) 73 (answered) 74 (answered) 75 (answered) 76 (answered) 77 (answered) 78 (answered) 79 (answered) 80 (answered) 81 (answered) 82 (answered) 83 (answered) 84 (answered) 85 (answered) 86 (answered) 87 (answered) 88 (answered) 89 (answered) 90 (answered) 91 (answered) 92 (answered) 93 (answered) 94 (answered) 95 (answered) 96 (answered) 97 (answered) 98 (answered) 99 (answered)100 (answered)101 (answered)102 (answered)103 (answered)104 (answered)105 (answered)106 (answered)107 (answered)108 (answered)109 (answered)110 (answered)111 (answered)112 (answered)113 (answered)114 (answered)115 (answered)116 (answered)117 (answered)118 (answered)119 (answered)120 (answered)


Which of the following groups led attempts to abolish slavery before the American Revolution?

a. Roman Catholics

b. Puritans

c. white Southern artisans and craftworkers

d. Quakers

e. the New York legislature



The correct answer is D. Maryland, the colony most closely associated with Roman Catholics, answer A, was the second colony to make slavery a lifelong condition. Answer B is incorrect. Answer C is incorrect; there were few non-African American artisans and craftworkers in the South. Most of this kind of work was done by slaves. Answer E is incorrect; New York's legislature legalized slavery in the late 1600s.

Return to Section Report Question: 23 of 120
Jump to question: 1 (answered) 2 (answered) 3 (answered) 4 (answered) 5 (answered) 6 (answered) 7 (answered) 8 (answered) 9 (answered) 10 (answered) 11 (answered) 12 (answered) 13 (answered) 14 (answered) 15 (answered) 16 (answered) 17 (answered) 18 (answered) 19 (answered) 20 (answered) 21 (answered) 22 (answered) 23 (answered) 24 (answered) 25 (answered) 26 (answered) 27 (answered) 28 (answered) 29 (answered) 30 (answered) 31 (answered) 32 (answered) 33 (answered) 34 (answered) 35 (answered) 36 (answered) 37 (answered) 38 (answered) 39 (answered) 40 (answered) 41 (answered) 42 (answered) 43 (answered) 44 (answered) 45 (answered) 46 (answered) 47 (answered) 48 (answered) 49 (answered) 50 (answered) 51 (answered) 52 (answered) 53 (answered) 54 (answered) 55 (answered) 56 (answered) 57 (answered) 58 (answered) 59 (answered) 60 (answered) 61 (answered) 62 (answered) 63 (answered) 64 (answered) 65 (answered) 66 (answered) 67 (answered) 68 (answered) 69 (answered) 70 (answered) 71 (answered) 72 (answered) 73 (answered) 74 (answered) 75 (answered) 76 (answered) 77 (answered) 78 (answered) 79 (answered) 80 (answered) 81 (answered) 82 (answered) 83 (answered) 84 (answered) 85 (answered) 86 (answered) 87 (answered) 88 (answered) 89 (answered) 90 (answered) 91 (answered) 92 (answered) 93 (answered) 94 (answered) 95 (answered) 96 (answered) 97 (answered) 98 (answered) 99 (answered)100 (answered)101 (answered)102 (answered)103 (answered)104 (answered)105 (answered)106 (answered)107 (answered)108 (answered)109 (answered)110 (answered)111 (answered)112 (answered)113 (answered)114 (answered)115 (answered)116 (answered)117 (answered)118 (answered)119 (answered)120 (answered)


Of the following colonies, which had the most diverse ethnic and religious population?

a. Massachusetts Bay

b. Pennsylvania

c. Virginia

d. Rhode Island

e. North Carolina



The correct answer is B. The Middle Colonies had a smaller English population and more nationalities with their diverse religions than either of the other two regions. Since Pennsylvania is the only Middle Colony listed, it's the answer. Even if you didn't know the answer, you could eliminate answers to get to it. Answers A and B were New England colonies whose strict religious practices and limiting of political rights to church members along with the climate and poor soil made them unattractive to other ethnic and religious groups. Based on numbers alone, you can rule out answers C and E They were Southern Colonies, whose large African populations meant that close to half the populations of both were homogeneous.

Return to Section Report Question: 24 of 120
Jump to question: 1 (answered) 2 (answered) 3 (answered) 4 (answered) 5 (answered) 6 (answered) 7 (answered) 8 (answered) 9 (answered) 10 (answered) 11 (answered) 12 (answered) 13 (answered) 14 (answered) 15 (answered) 16 (answered) 17 (answered) 18 (answered) 19 (answered) 20 (answered) 21 (answered) 22 (answered) 23 (answered) 24 (answered) 25 (answered) 26 (answered) 27 (answered) 28 (answered) 29 (answered) 30 (answered) 31 (answered) 32 (answered) 33 (answered) 34 (answered) 35 (answered) 36 (answered) 37 (answered) 38 (answered) 39 (answered) 40 (answered) 41 (answered) 42 (answered) 43 (answered) 44 (answered) 45 (answered) 46 (answered) 47 (answered) 48 (answered) 49 (answered) 50 (answered) 51 (answered) 52 (answered) 53 (answered) 54 (answered) 55 (answered) 56 (answered) 57 (answered) 58 (answered) 59 (answered) 60 (answered) 61 (answered) 62 (answered) 63 (answered) 64 (answered) 65 (answered) 66 (answered) 67 (answered) 68 (answered) 69 (answered) 70 (answered) 71 (answered) 72 (answered) 73 (answered) 74 (answered) 75 (answered) 76 (answered) 77 (answered) 78 (answered) 79 (answered) 80 (answered) 81 (answered) 82 (answered) 83 (answered) 84 (answered) 85 (answered) 86 (answered) 87 (answered) 88 (answered) 89 (answered) 90 (answered) 91 (answered) 92 (answered) 93 (answered) 94 (answered) 95 (answered) 96 (answered) 97 (answered) 98 (answered) 99 (answered)100 (answered)101 (answered)102 (answered)103 (answered)104 (answered)105 (answered)106 (answered)107 (answered)108 (answered)109 (answered)110 (answered)111 (answered)112 (answered)113 (answered)114 (answered)115 (answered)116 (answered)117 (answered)118 (answered)119 (answered)120 (answered)


All of the following are characteristics of the British colonial system as applied to the North American colonies EXCEPT

a. English common law as the legal system

b. representative colonial assemblies

c. participation in the British trading network

d. centralized government for the colonies

e. oversight by the Board of Trade



The correct answer is D. The lack of a centralized government was a major reason why Parliament and the monarchy could not control the activities of the colonies. This lack of centralization may also be the reason for the framers choice of a federal system for the new United States.

Return to Section Report Question: 25 of 120
Jump to question: 1 (answered) 2 (answered) 3 (answered) 4 (answered) 5 (answered) 6 (answered) 7 (answered) 8 (answered) 9 (answered) 10 (answered) 11 (answered) 12 (answered) 13 (answered) 14 (answered) 15 (answered) 16 (answered) 17 (answered) 18 (answered) 19 (answered) 20 (answered) 21 (answered) 22 (answered) 23 (answered) 24 (answered) 25 (answered) 26 (answered) 27 (answered) 28 (answered) 29 (answered) 30 (answered) 31 (answered) 32 (answered) 33 (answered) 34 (answered) 35 (answered) 36 (answered) 37 (answered) 38 (answered) 39 (answered) 40 (answered) 41 (answered) 42 (answered) 43 (answered) 44 (answered) 45 (answered) 46 (answered) 47 (answered) 48 (answered) 49 (answered) 50 (answered) 51 (answered) 52 (answered) 53 (answered) 54 (answered) 55 (answered) 56 (answered) 57 (answered) 58 (answered) 59 (answered) 60 (answered) 61 (answered) 62 (answered) 63 (answered) 64 (answered) 65 (answered) 66 (answered) 67 (answered) 68 (answered) 69 (answered) 70 (answered) 71 (answered) 72 (answered) 73 (answered) 74 (answered) 75 (answered) 76 (answered) 77 (answered) 78 (answered) 79 (answered) 80 (answered) 81 (answered) 82 (answered) 83 (answered) 84 (answered) 85 (answered) 86 (answered) 87 (answered) 88 (answered) 89 (answered) 90 (answered) 91 (answered) 92 (answered) 93 (answered) 94 (answered) 95 (answered) 96 (answered) 97 (answered) 98 (answered) 99 (answered)100 (answered)101 (answered)102 (answered)103 (answered)104 (answered)105 (answered)106 (answered)107 (answered)108 (answered)109 (answered)110 (answered)111 (answered)112 (answered)113 (answered)114 (answered)115 (answered)116 (answered)117 (answered)118 (answered)119 (answered)120 (answered)


The first attempt by colonists to unite for a political purpose was

a. Annapolis Convention

b. American System

c. Hartford Convention

d. Albany Plan of Union

e. Dominion of New England



The correct answer is D. Answer D was proposed by Benjamin Franklin and modeled after the Iroquois Confederacy. Although the delegates in Albany agreed, neither Parliament nor the colonial legislatures approved the plan. Answer A was the convention called to revise the Articles of Confederation, which instead called for a convention to write a new constitution. Answer B was Henry Clay's proposal for an economic system that would provide financial aid to each section of the new nation. Answer C was the assembly called by New England Federalists who opposed the War of 1812; its timing and its support of states' rights brought discredit to the Federalist Party. Answer E was the political unit created by Parliament in 1684 to govern all land north of Pennsylvania. With the Glorious Restoration in 1688, this was later reversed.

Return to Section Report Question: 26 of 120
Jump to question: 1 (answered) 2 (answered) 3 (answered) 4 (answered) 5 (answered) 6 (answered) 7 (answered) 8 (answered) 9 (answered) 10 (answered) 11 (answered) 12 (answered) 13 (answered) 14 (answered) 15 (answered) 16 (answered) 17 (answered) 18 (answered) 19 (answered) 20 (answered) 21 (answered) 22 (answered) 23 (answered) 24 (answered) 25 (answered) 26 (answered) 27 (answered) 28 (answered) 29 (answered) 30 (answered) 31 (answered) 32 (answered) 33 (answered) 34 (answered) 35 (answered) 36 (answered) 37 (answered) 38 (answered) 39 (answered) 40 (answered) 41 (answered) 42 (answered) 43 (answered) 44 (answered) 45 (answered) 46 (answered) 47 (answered) 48 (answered) 49 (answered) 50 (answered) 51 (answered) 52 (answered) 53 (answered) 54 (answered) 55 (answered) 56 (answered) 57 (answered) 58 (answered) 59 (answered) 60 (answered) 61 (answered) 62 (answered) 63 (answered) 64 (answered) 65 (answered) 66 (answered) 67 (answered) 68 (answered) 69 (answered) 70 (answered) 71 (answered) 72 (answered) 73 (answered) 74 (answered) 75 (answered) 76 (answered) 77 (answered) 78 (answered) 79 (answered) 80 (answered) 81 (answered) 82 (answered) 83 (answered) 84 (answered) 85 (answered) 86 (answered) 87 (answered) 88 (answered) 89 (answered) 90 (answered) 91 (answered) 92 (answered) 93 (answered) 94 (answered) 95 (answered) 96 (answered) 97 (answered) 98 (answered) 99 (answered)100 (answered)101 (answered)102 (answered)103 (answered)104 (answered)105 (answered)106 (answered)107 (answered)108 (answered)109 (answered)110 (answered)111 (answered)112 (answered)113 (answered)114 (answered)115 (answered)116 (answered)117 (answered)118 (answered)119 (answered)120 (answered)


Which of the following is not a true statement about the Navigation Acts?



a. One purpose of the Navigation Acts was to ensure that raw materials were shipped only to England.

b. A second purpose of the Navigation Acts was to punish English sailors who jumped ship.

c. Under the Navigation Acts, all colonial exports and imports had to be shipped through England.

d. Initial colonial opposition to the Navigation Acts was light, because the acts were not rigidly enforced.

e. One purpose of the Navigation Acts was to cut out other nations from profitable trade with the colonies.



The correct answer is B. Don't confuse the Navigation Acts with the impressment of sailors that precipitated conflict with Great Britain in the early 1800s. In answer E, the major trade competitor at the time was the Netherlands.

Return to Section Report Question: 27 of 120
Jump to question: 1 (answered) 2 (answered) 3 (answered) 4 (answered) 5 (answered) 6 (answered) 7 (answered) 8 (answered) 9 (answered) 10 (answered) 11 (answered) 12 (answered) 13 (answered) 14 (answered) 15 (answered) 16 (answered) 17 (answered) 18 (answered) 19 (answered) 20 (answered) 21 (answered) 22 (answered) 23 (answered) 24 (answered) 25 (answered) 26 (answered) 27 (answered) 28 (answered) 29 (answered) 30 (answered) 31 (answered) 32 (answered) 33 (answered) 34 (answered) 35 (answered) 36 (answered) 37 (answered) 38 (answered) 39 (answered) 40 (answered) 41 (answered) 42 (answered) 43 (answered) 44 (answered) 45 (answered) 46 (answered) 47 (answered) 48 (answered) 49 (answered) 50 (answered) 51 (answered) 52 (answered) 53 (answered) 54 (answered) 55 (answered) 56 (answered) 57 (answered) 58 (answered) 59 (answered) 60 (answered) 61 (answered) 62 (answered) 63 (answered) 64 (answered) 65 (answered) 66 (answered) 67 (answered) 68 (answered) 69 (answered) 70 (answered) 71 (answered) 72 (answered) 73 (answered) 74 (answered) 75 (answered) 76 (answered) 77 (answered) 78 (answered) 79 (answered) 80 (answered) 81 (answered) 82 (answered) 83 (answered) 84 (answered) 85 (answered) 86 (answered) 87 (answered) 88 (answered) 89 (answered) 90 (answered) 91 (answered) 92 (answered) 93 (answered) 94 (answered) 95 (answered) 96 (answered) 97 (answered) 98 (answered) 99 (answered)100 (answered)101 (answered)102 (answered)103 (answered)104 (answered)105 (answered)106 (answered)107 (answered)108 (answered)109 (answered)110 (answered)111 (answered)112 (answered)113 (answered)114 (answered)115 (answered)116 (answered)117 (answered)118 (answered)119 (answered)120 (answered)


Which of the following asserted Parliament's right to tax the colonies to raise revenue?

a. Stamp Act

b. Molasses Act

c. Sugar Act

d. Currency Act

e. Hat Act



The correct answer is C. The previous acts, answers B and E, had been meant to regulate trade. Answer A, Stamp Act, was the first internal tax, that is, on business within the colonies and not between the colonies and Great Britain or another nation. Answer D forbade the colonies to issue their own money.

Return to Section Report Question: 28 of 120
Jump to question: 1 (answered) 2 (answered) 3 (answered) 4 (answered) 5 (answered) 6 (answered) 7 (answered) 8 (answered) 9 (answered) 10 (answered) 11 (answered) 12 (answered) 13 (answered) 14 (answered) 15 (answered) 16 (answered) 17 (answered) 18 (answered) 19 (answered) 20 (answered) 21 (answered) 22 (answered) 23 (answered) 24 (answered) 25 (answered) 26 (answered) 27 (answered) 28 (answered) 29 (answered) 30 (answered) 31 (answered) 32 (answered) 33 (answered) 34 (answered) 35 (answered) 36 (answered) 37 (answered) 38 (answered) 39 (answered) 40 (answered) 41 (answered) 42 (answered) 43 (answered) 44 (answered) 45 (answered) 46 (answered) 47 (answered) 48 (answered) 49 (answered) 50 (answered) 51 (answered) 52 (answered) 53 (answered) 54 (answered) 55 (answered) 56 (answered) 57 (answered) 58 (answered) 59 (answered) 60 (answered) 61 (answered) 62 (answered) 63 (answered) 64 (answered) 65 (answered) 66 (answered) 67 (answered) 68 (answered) 69 (answered) 70 (answered) 71 (answered) 72 (answered) 73 (answered) 74 (answered) 75 (answered) 76 (answered) 77 (answered) 78 (answered) 79 (answered) 80 (answered) 81 (answered) 82 (answered) 83 (answered) 84 (answered) 85 (answered) 86 (answered) 87 (answered) 88 (answered) 89 (answered) 90 (answered) 91 (answered) 92 (answered) 93 (answered) 94 (answered) 95 (answered) 96 (answered) 97 (answered) 98 (answered) 99 (answered)100 (answered)101 (answered)102 (answered)103 (answered)104 (answered)105 (answered)106 (answered)107 (answered)108 (answered)109 (answered)110 (answered)111 (answered)112 (answered)113 (answered)114 (answered)115 (answered)116 (answered)117 (answered)118 (answered)119 (answered)120 (answered)


Which of the following urged the establishment of Committees of Correspondence?

a. Sam Adams

b. Paul Revere

c. John Hancock

d. Patrick Henry

e. James Otis



The correct answer is A. Answer B, Paul Revere, was not only the spreader of the alarm "the British are coming," but also a participant in the Boston Tea Party and silversmith and engraver by trade. He designed the new nation's first currency. Answer C, John Hancock, was the first signer of the Declaration of Independence and president of the Second Continental Congress at the time. Answer D, Patrick Henry, is famous for the phrase "give me liberty or give me death," his support for independence, and his opposition to ratification of the U.S. Constitution. Answer E, James Otis, with Sam Adams wrote the Circular Letter that set out the colonists' opposition to the Townshend Acts.

Return to Section Report Question: 29 of 120
Jump to question: 1 (answered) 2 (answered) 3 (answered) 4 (answered) 5 (answered) 6 (answered) 7 (answered) 8 (answered) 9 (answered) 10 (answered) 11 (answered) 12 (answered) 13 (answered) 14 (answered) 15 (answered) 16 (answered) 17 (answered) 18 (answered) 19 (answered) 20 (answered) 21 (answered) 22 (answered) 23 (answered) 24 (answered) 25 (answered) 26 (answered) 27 (answered) 28 (answered) 29 (answered) 30 (answered) 31 (answered) 32 (answered) 33 (answered) 34 (answered) 35 (answered) 36 (answered) 37 (answered) 38 (answered) 39 (answered) 40 (answered) 41 (answered) 42 (answered) 43 (answered) 44 (answered) 45 (answered) 46 (answered) 47 (answered) 48 (answered) 49 (answered) 50 (answered) 51 (answered) 52 (answered) 53 (answered) 54 (answered) 55 (answered) 56 (answered) 57 (answered) 58 (answered) 59 (answered) 60 (answered) 61 (answered) 62 (answered) 63 (answered) 64 (answered) 65 (answered) 66 (answered) 67 (answered) 68 (answered) 69 (answered) 70 (answered) 71 (answered) 72 (answered) 73 (answered) 74 (answered) 75 (answered) 76 (answered) 77 (answered) 78 (answered) 79 (answered) 80 (answered) 81 (answered) 82 (answered) 83 (answered) 84 (answered) 85 (answered) 86 (answered) 87 (answered) 88 (answered) 89 (answered) 90 (answered) 91 (answered) 92 (answered) 93 (answered) 94 (answered) 95 (answered) 96 (answered) 97 (answered) 98 (answered) 99 (answered)100 (answered)101 (answered)102 (answered)103 (answered)104 (answered)105 (answered)106 (answered)107 (answered)108 (answered)109 (answered)110 (answered)111 (answered)112 (answered)113 (answered)114 (answered)115 (answered)116 (answered)117 (answered)118 (answered)119 (answered)120 (answered)


What is the significance of the Intolerable Acts?



a. The Intolerable Acts closed the port of Boston and dismissed the Massachusetts legislature.

b. taken with the Quebec Act, the Intolerable Acts showed a lack of understanding of colonial interests.

c. The Intolerable Acts took away all civil rights from the citizens of Canada.

d. In passing the Intolerable Acts, Parliament shifted its focus from persuasion to punishment.

e. Parliament focused on specific issues rather than on the broader picture of colonial discontent.



The correct answer is D. Answer A is incorrect, because the questions asks for the significance of the Intolerable Acts. Answer A merely tells you some of the provisions of the acts. Answer B is incorrect, because the acts hit directly at colonial interests, for example, trade, self-government, and territorial expansion. Answer C is incorrect; the Quebec Act, which was passed at the same time as the Coercive Acts (the proper title for the Intolerable Acts), allowed the French citizens of Quebec to keep the French civil code of laws, which was more limited in the areas of rights than the English code. Answer E is incorrect, because underlying the provisions of the acts was the broader issue of self-government.

Return to Section Report Question: 30 of 120
Jump to question: 1 (answered) 2 (answered) 3 (answered) 4 (answered) 5 (answered) 6 (answered) 7 (answered) 8 (answered) 9 (answered) 10 (answered) 11 (answered) 12 (answered) 13 (answered) 14 (answered) 15 (answered) 16 (answered) 17 (answered) 18 (answered) 19 (answered) 20 (answered) 21 (answered) 22 (answered) 23 (answered) 24 (answered) 25 (answered) 26 (answered) 27 (answered) 28 (answered) 29 (answered) 30 (answered) 31 (answered) 32 (answered) 33 (answered) 34 (answered) 35 (answered) 36 (answered) 37 (answered) 38 (answered) 39 (answered) 40 (answered) 41 (answered) 42 (answered) 43 (answered) 44 (answered) 45 (answered) 46 (answered) 47 (answered) 48 (answered) 49 (answered) 50 (answered) 51 (answered) 52 (answered) 53 (answered) 54 (answered) 55 (answered) 56 (answered) 57 (answered) 58 (answered) 59 (answered) 60 (answered) 61 (answered) 62 (answered) 63 (answered) 64 (answered) 65 (answered) 66 (answered) 67 (answered) 68 (answered) 69 (answered) 70 (answered) 71 (answered) 72 (answered) 73 (answered) 74 (answered) 75 (answered) 76 (answered) 77 (answered) 78 (answered) 79 (answered) 80 (answered) 81 (answered) 82 (answered) 83 (answered) 84 (answered) 85 (answered) 86 (answered) 87 (answered) 88 (answered) 89 (answered) 90 (answered) 91 (answered) 92 (answered) 93 (answered) 94 (answered) 95 (answered) 96 (answered) 97 (answered) 98 (answered) 99 (answered)100 (answered)101 (answered)102 (answered)103 (answered)104 (answered)105 (answered)106 (answered)107 (answered)108 (answered)109 (answered)110 (answered)111 (answered)112 (answered)113 (answered)114 (answered)115 (answered)116 (answered)117 (answered)118 (answered)119 (answered)120 (answered)


Which of the following is NOT a true statement about the Olive Branch Petition?

a. It was rejected by George III.

b. It was a concession to moderates in the Second Continental Congress.

c. It was followed by the colonists' Declaration of the Causes and Necessity of Taking Up Arms.

d. It was directed to the monarch and Parliament.

e. It was countered by George III's issuance of the Proclamation of Rebellion.



The correct answer is D. The Olive Branch Petition was addressed solely to George III and restated the colonists' loyalty to the king.

Return to Section Report Question: 31 of 120
Jump to question: 1 (answered) 2 (answered) 3 (answered) 4 (answered) 5 (answered) 6 (answered) 7 (answered) 8 (answered) 9 (answered) 10 (answered) 11 (answered) 12 (answered) 13 (answered) 14 (answered) 15 (answered) 16 (answered) 17 (answered) 18 (answered) 19 (answered) 20 (answered) 21 (answered) 22 (answered) 23 (answered) 24 (answered) 25 (answered) 26 (answered) 27 (answered) 28 (answered) 29 (answered) 30 (answered) 31 (answered) 32 (answered) 33 (answered) 34 (answered) 35 (answered) 36 (answered) 37 (answered) 38 (answered) 39 (answered) 40 (answered) 41 (answered) 42 (answered) 43 (answered) 44 (answered) 45 (answered) 46 (answered) 47 (answered) 48 (answered) 49 (answered) 50 (answered) 51 (answered) 52 (answered) 53 (answered) 54 (answered) 55 (answered) 56 (answered) 57 (answered) 58 (answered) 59 (answered) 60 (answered) 61 (answered) 62 (answered) 63 (answered) 64 (answered) 65 (answered) 66 (answered) 67 (answered) 68 (answered) 69 (answered) 70 (answered) 71 (answered) 72 (answered) 73 (answered) 74 (answered) 75 (answered) 76 (answered) 77 (answered) 78 (answered) 79 (answered) 80 (answered) 81 (answered) 82 (answered) 83 (answered) 84 (answered) 85 (answered) 86 (answered) 87 (answered) 88 (answered) 89 (answered) 90 (answered) 91 (answered) 92 (answered) 93 (answered) 94 (answered) 95 (answered) 96 (answered) 97 (answered) 98 (answered) 99 (answered)100 (answered)101 (answered)102 (answered)103 (answered)104 (answered)105 (answered)106 (answered)107 (answered)108 (answered)109 (answered)110 (answered)111 (answered)112 (answered)113 (answered)114 (answered)115 (answered)116 (answered)117 (answered)118 (answered)119 (answered)120 (answered)


Which of the following is not a reason why colonists hesitated to declare independence in 1775?



a. How to attract foreign support was an unresolved issue.

b. Many could not see themselves as traitors, which is what a declaration of independence would mean.

c. How would Americans organize themselves since nations at the time had some form of monarchy.

d. Breaking political ties with England meant turning their back on their heritage.

e. Once unleashed, the fury of ordinary colonists could turn against the well to do and the rebellion could slide into anarchy.



The correct answer is A. Answer A is incorrect, because in 1775 the Second Continental Congress had sent representatives to France, Spain, and the Netherlands, Great Britain's traditional enemies, looking for military and economic aid. The wealthy, traditional, and more conservative colonial leaders had already seen the results of mob violence in the stoning of tax collectors, the destruction of property, and mass rallies and parades, so answer E was a real concern to them.

Return to Section Report Question: 32 of 120
Jump to question: 1 (answered) 2 (answered) 3 (answered) 4 (answered) 5 (answered) 6 (answered) 7 (answered) 8 (answered) 9 (answered) 10 (answered) 11 (answered) 12 (answered) 13 (answered) 14 (answered) 15 (answered) 16 (answered) 17 (answered) 18 (answered) 19 (answered) 20 (answered) 21 (answered) 22 (answered) 23 (answered) 24 (answered) 25 (answered) 26 (answered) 27 (answered) 28 (answered) 29 (answered) 30 (answered) 31 (answered) 32 (answered) 33 (answered) 34 (answered) 35 (answered) 36 (answered) 37 (answered) 38 (answered) 39 (answered) 40 (answered) 41 (answered) 42 (answered) 43 (answered) 44 (answered) 45 (answered) 46 (answered) 47 (answered) 48 (answered) 49 (answered) 50 (answered) 51 (answered) 52 (answered) 53 (answered) 54 (answered) 55 (answered) 56 (answered) 57 (answered) 58 (answered) 59 (answered) 60 (answer
Adoption of which of the following was an admission by Congregational leaders that the church was no longer a central factor in people's lives?

a. Second Mayflower Compact

b. Holy Experiment

c. Halfway Covenant

d. Act of Religious Toleration

e. Fundamental Orders of Connecticut
The correct answer is C. Answer A sounds good, but never existed. Answer B is the term William Penn gave to Pennsylvania when he founded it. Answer D is a law passed by the Maryland legislature, the first religious toleration statute in the colonies. Answer E was the first constitution drawn up in the colonies.

Return to Section Report Question: 11 of 120
Jump to question: 1 (answered) 2 (answered) 3 (answered) 4 (answered) 5 (answered) 6 (answered) 7 (answered) 8 (answered) 9 (answered) 10 (answered) 11 (answered) 12 (answered) 13 (answered) 14 (answered) 15 (answered) 16 (answered) 17 (answered) 18 (answered) 19 (answered) 20 (answered) 21 (answered) 22 (answered) 23 (answered) 24 (answered) 25 (answered) 26 (answered) 27 (answered) 28 (answered) 29 (answered) 30 (answered) 31 (answered) 32 (answered) 33 (answered) 34 (answered) 35 (answered) 36 (answered) 37 (answered) 38 (answered) 39 (answered) 40 (answered) 41 (answered) 42 (answered) 43 (answered) 44 (answered) 45 (answered) 46 (answered) 47 (answered) 48 (answered) 49 (answered) 50 (answered) 51 (answered) 52 (answered) 53 (answered) 54 (answered) 55 (answered) 56 (answered) 57 (answered) 58 (answered) 59 (answered) 60 (answered) 61 (answered) 62 (answered) 63 (answered) 64 (answered) 65 (answered) 66 (answered) 67 (answered) 68 (answered) 69 (answered) 70 (answered) 71 (answered) 72 (answered) 73 (answered) 74 (answered) 75 (answered) 76 (answered) 77 (answered) 78 (answered) 79 (answered) 80 (answered) 81 (answered) 82 (answered) 83 (answered) 84 (answered) 85 (answered) 86 (answered) 87 (answered) 88 (answered) 89 (answered) 90 (answered) 91 (answered) 92 (answered) 93 (answered) 94 (answered) 95 (answered) 96 (answered) 97 (answered) 98 (answered) 99 (answered)100 (answered)101 (answered)102 (answered)103 (answered)104 (answered)105 (answered)106 (answered)107 (answered)108 (answered)109 (answered)110 (answered)111 (answered)112 (answered)113 (answered)114 (answered)115 (answered)116 (answered)117 (answered)118 (answered)119 (answered)120 (answered)


As far as the British government was concerned, the reason for the founding of Georgia was

a. to establish a colony for the poor from English debtors' prisons

b. as a haven for religious dissenters

c. to return a profit for the Crown

d. to establish a colony based on slavery from its beginning

e. to establish a buffer between South Carolina and Spanish Florida



The correct answer is E. The key phrase in the question is "as far as the British government." Answer A was a reason of James Oglethorpe, one of the eight original proprietors, but it was not the reason the government was interested. Don't confuse Georgia with Plymouth, Massachusetts Bay, Maryland, or Pennsylvania, answer B. Answer C is incorrect, because Georgia was a proprietary colony. Answer D is the opposite of what was true; the proprietors banned slavery at the inception of the colony, but later gave in to angry colonists who said they could not turn a profit without the use of slaves.

Return to Section Report Question: 12 of 120
Jump to question: 1 (answered) 2 (answered) 3 (answered) 4 (answered) 5 (answered) 6 (answered) 7 (answered) 8 (answered) 9 (answered) 10 (answered) 11 (answered) 12 (answered) 13 (answered) 14 (answered) 15 (answered) 16 (answered) 17 (answered) 18 (answered) 19 (answered) 20 (answered) 21 (answered) 22 (answered) 23 (answered) 24 (answered) 25 (answered) 26 (answered) 27 (answered) 28 (answered) 29 (answered) 30 (answered) 31 (answered) 32 (answered) 33 (answered) 34 (answered) 35 (answered) 36 (answered) 37 (answered) 38 (answered) 39 (answered) 40 (answered) 41 (answered) 42 (answered) 43 (answered) 44 (answered) 45 (answered) 46 (answered) 47 (answered) 48 (answered) 49 (answered) 50 (answered) 51 (answered) 52 (answered) 53 (answered) 54 (answered) 55 (answered) 56 (answered) 57 (answered) 58 (answered) 59 (answered) 60 (answered) 61 (answered) 62 (answered) 63 (answered) 64 (answered) 65 (answered) 66 (answered) 67 (answered) 68 (answered) 69 (answered) 70 (answered) 71 (answered) 72 (answered) 73 (answered) 74 (answered) 75 (answered) 76 (answered) 77 (answered) 78 (answered) 79 (answered) 80 (answered) 81 (answered) 82 (answered) 83 (answered) 84 (answered) 85 (answered) 86 (answered) 87 (answered) 88 (answered) 89 (answered) 90 (answered) 91 (answered) 92 (answered) 93 (answered) 94 (answered) 95 (answered) 96 (answered) 97 (answered) 98 (answered) 99 (answered)100 (answered)101 (answered)102 (answered)103 (answered)104 (answered)105 (answered)106 (answered)107 (answered)108 (answered)109 (answered)110 (answered)111 (answered)112 (answered)113 (answered)114 (answered)115 (answered)116 (answered)117 (answered)118 (answered)119 (answered)120 (answered)


The large urban centers of the North American colonies were located mostly

a. in the South

b. in the Middle Colonies

c. in the interior

d. in New England

e. in valleys



The correct answer is B. The Middle Colonies were more welcoming to immigrants than New England, answer D, was—both the environment and in policies toward religion. The weather was milder and the land, more fertile. A larger population meant not only a larger area under settlement, but also an increase in the number of townspeople per village, town, and city. Answer A is incorrect. Most of the southern population was rural, living either on small farms or on plantations. Answer C is incorrect. Relatively speaking, little of the interior had been settled by 1776. The majority of the colonial population still lived along the coast. Answer E is incorrect. The majority of the population lived along the coastal plain.

Return to Section Report Question: 13 of 120
Jump to question: 1 (answered) 2 (answered) 3 (answered) 4 (answered) 5 (answered) 6 (answered) 7 (answered) 8 (answered) 9 (answered) 10 (answered) 11 (answered) 12 (answered) 13 (answered) 14 (answered) 15 (answered) 16 (answered) 17 (answered) 18 (answered) 19 (answered) 20 (answered) 21 (answered) 22 (answered) 23 (answered) 24 (answered) 25 (answered) 26 (answered) 27 (answered) 28 (answered) 29 (answered) 30 (answered) 31 (answered) 32 (answered) 33 (answered) 34 (answered) 35 (answered) 36 (answered) 37 (answered) 38 (answered) 39 (answered) 40 (answered) 41 (answered) 42 (answered) 43 (answered) 44 (answered) 45 (answered) 46 (answered) 47 (answered) 48 (answered) 49 (answered) 50 (answered) 51 (answered) 52 (answered) 53 (answered) 54 (answered) 55 (answered) 56 (answered) 57 (answered) 58 (answered) 59 (answered) 60 (answered) 61 (answered) 62 (answered) 63 (answered) 64 (answered) 65 (answered) 66 (answered) 67 (answered) 68 (answered) 69 (answered) 70 (answered) 71 (answered) 72 (answered) 73 (answered) 74 (answered) 75 (answered) 76 (answered) 77 (answered) 78 (answered) 79 (answered) 80 (answered) 81 (answered) 82 (answered) 83 (answered) 84 (answered) 85 (answered) 86 (answered) 87 (answered) 88 (answered) 89 (answered) 90 (answered) 91 (answered) 92 (answered) 93 (answered) 94 (answered) 95 (answered) 96 (answered) 97 (answered) 98 (answered) 99 (answered)100 (answered)101 (answered)102 (answered)103 (answered)104 (answered)105 (answered)106 (answered)107 (answered)108 (answered)109 (answered)110 (answered)111 (answered)112 (answered)113 (answered)114 (answered)115 (answered)116 (answered)117 (answered)118 (answered)119 (answered)120 (answered)


An unintended result of the First Great Awakening was

a. a new level of intolerance toward all religions

b. a more democratic outlook among colonists and less reliance on traditional authority and social class

c. a general lessening of interest in religion

d. that evangelism delayed the establishment of schools and colleges

e. a dramatic increase in membership in traditional churches



The correct answer is B. There is no evidence to support answer A. Common sense eliminates answer C, since the question is about a revival of interest in religion. Answer D is the opposite of what occurred. A number of colleges were founded in this period as a result of the idea that people could think for themselves. Answer E is incorrect; many people left the older churches to join newer ones like Methodism and Presbyterianism.

Return to Section Report Question: 14 of 120
Jump to question: 1 (answered) 2 (answered) 3 (answered) 4 (answered) 5 (answered) 6 (answered) 7 (answered) 8 (answered) 9 (answered) 10 (answered) 11 (answered) 12 (answered) 13 (answered) 14 (answered) 15 (answered) 16 (answered) 17 (answered) 18 (answered) 19 (answered) 20 (answered) 21 (answered) 22 (answered) 23 (answered) 24 (answered) 25 (answered) 26 (answered) 27 (answered) 28 (answered) 29 (answered) 30 (answered) 31 (answered) 32 (answered) 33 (answered) 34 (answered) 35 (answered) 36 (answered) 37 (answered) 38 (answered) 39 (answered) 40 (answered) 41 (answered) 42 (answered) 43 (answered) 44 (answered) 45 (answered) 46 (answered) 47 (answered) 48 (answered) 49 (answered) 50 (answered) 51 (answered) 52 (answered) 53 (answered) 54 (answered) 55 (answered) 56 (answered) 57 (answered) 58 (answered) 59 (answered) 60 (answered) 61 (answered) 62 (answered) 63 (answered) 64 (answered) 65 (answered) 66 (answered) 67 (answered) 68 (answered) 69 (answered) 70 (answered) 71 (answered) 72 (answered) 73 (answered) 74 (answered) 75 (answered) 76 (answered) 77 (answered) 78 (answered) 79 (answered) 80 (answered) 81 (answered) 82 (answered) 83 (answered) 84 (answered) 85 (answered) 86 (answered) 87 (answered) 88 (answered) 89 (answered) 90 (answered) 91 (answered) 92 (answered) 93 (answered) 94 (answered) 95 (answered) 96 (answered) 97 (answered) 98 (answered) 99 (answered)100 (answered)101 (answered)102 (answered)103 (answered)104 (answered)105 (answered)106 (answered)107 (answered)108 (answered)109 (answered)110 (answered)111 (answered)112 (answered)113 (answered)114 (answered)115 (answered)116 (answered)117 (answered)118 (answered)119 (answered)120 (answered)


Which of the following would be least likely to be found in a colonial gentleman's library in the mid-1700s?

a. writings of John Locke

b. book of sermons

c. Latin version of the Aenid

d. novel written by a colonial author

e. collection of poetry by an English poet



The correct answer is D. While colonists may have been developing their own uniquely American view of politics, their literary tastes were still very much influenced by what the British wrote and read, which included religious writings. The American novel did not come into its own until the early national period with the works of James Fenimore Cooper.

Return to Section Report Question: 15 of 120
Jump to question: 1 (answered) 2 (answered) 3 (answered) 4 (answered) 5 (answered) 6 (answered) 7 (answered) 8 (answered) 9 (answered) 10 (answered) 11 (answered) 12 (answered) 13 (answered) 14 (answered) 15 (answered) 16 (answered) 17 (answered) 18 (answered) 19 (answered) 20 (answered) 21 (answered) 22 (answered) 23 (answered) 24 (answered) 25 (answered) 26 (answered) 27 (answered) 28 (answered) 29 (answered) 30 (answered) 31 (answered) 32 (answered) 33 (answered) 34 (answered) 35 (answered) 36 (answered) 37 (answered) 38 (answered) 39 (answered) 40 (answered) 41 (answered) 42 (answered) 43 (answered) 44 (answered) 45 (answered) 46 (answered) 47 (answered) 48 (answered) 49 (answered) 50 (answered) 51 (answered) 52 (answered) 53 (answered) 54 (answered) 55 (answered) 56 (answered) 57 (answered) 58 (answered) 59 (answered) 60 (answered) 61 (answered) 62 (answered) 63 (answered) 64 (answered) 65 (answered) 66 (answered) 67 (answered) 68 (answered) 69 (answered) 70 (answered) 71 (answered) 72 (answered) 73 (answered) 74 (answered) 75 (answered) 76 (answered) 77 (answered) 78 (answered) 79 (answered) 80 (answered) 81 (answered) 82 (answered) 83 (answered) 84 (answered) 85 (answered) 86 (answered) 87 (answered) 88 (answered) 89 (answered) 90 (answered) 91 (answered) 92 (answered) 93 (answered) 94 (answered) 95 (answered) 96 (answered) 97 (answered) 98 (answered) 99 (answered)100 (answered)101 (answered)102 (answered)103 (answered)104 (answered)105 (answered)106 (answered)107 (answered)108 (answered)109 (answered)110 (answered)111 (answered)112 (answered)113 (answered)114 (answered)115 (answered)116 (answered)117 (answered)118 (answered)119 (answered)120 (answered)


The most important port and the largest city in the North American colonies by 1776 was

a. Charleston

b. Baltimore

c. Philadelphia

d. New York

e. Boston



The correct answer is C. Answer A, Charleston, could be ruled out, because it was in a southern colony and there was less population in that region than in the Middle or New England colonies. For the same reason, rule out answer B, Baltimore. That would leave you with three good choices, but the long, cold winters could help you rule out Boston, answer E. Remember that the delegates to the Continental Congress chose Philadelphia as the site of their meetings, so it must have been a big and important city. Pennsylvania by the 1700s had become known as the breadbasket of the colonies, which means that it grew a lot of food, more than its people used, which means it had to ship it from someplace—Philadelphia.

Return to Section Report Question: 16 of 120
Jump to question: 1 (answered) 2 (answered) 3 (answered) 4 (answered) 5 (answered) 6 (answered) 7 (answered) 8 (answered) 9 (answered) 10 (answered) 11 (answered) 12 (answered) 13 (answered) 14 (answered) 15 (answered) 16 (answered) 17 (answered) 18 (answered) 19 (answered) 20 (answered) 21 (answered) 22 (answered) 23 (answered) 24 (answered) 25 (answered) 26 (answered) 27 (answered) 28 (answered) 29 (answered) 30 (answered) 31 (answered) 32 (answered) 33 (answered) 34 (answered) 35 (answered) 36 (answered) 37 (answered) 38 (answered) 39 (answered) 40 (answered) 41 (answered) 42 (answered) 43 (answered) 44 (answered) 45 (answered) 46 (answered) 47 (answered) 48 (answered) 49 (answered) 50 (answered) 51 (answered) 52 (answered) 53 (answered) 54 (answered) 55 (answered) 56 (answered) 57 (answered) 58 (answered) 59 (answered) 60 (answered) 61 (answered) 62 (answered) 63 (answered) 64 (answered) 65 (answered) 66 (answered) 67 (answered) 68 (answered) 69 (answered) 70 (answered) 71 (answered) 72 (answered) 73 (answered) 74 (answered) 75 (answered) 76 (answered) 77 (answered) 78 (answered) 79 (answered) 80 (answered) 81 (answered) 82 (answered) 83 (answered) 84 (answered) 85 (answered) 86 (answered) 87 (answered) 88 (answered) 89 (answered) 90 (answered) 91 (answered) 92 (answered) 93 (answered) 94 (answered) 95 (answered) 96 (answered) 97 (answered) 98 (answered) 99 (answered)100 (answered)101 (answered)102 (answered)103 (answered)104 (answered)105 (answered)106 (answered)107 (answered)108 (answered)109 (answered)110 (answered)111 (answered)112 (answered)113 (answered)114 (answered)115 (answered)116 (answered)117 (answered)118 (answered)119 (answered)120 (answered)


The only colony in which enslaved African Americans outnumbered European Americans in the mid-1700s was

a. Maryland

b. Virginia

c. North Carolina

d. South Carolina

e. Georgia



The correct answer is D. If you didn't know this answer, you could at least eliminate Georgia, because it was not founded until 1732 and it originally banned slavery. South Carolina's major export was rice, not tobacco, and the unhealthy work of cultivating rice in wet, bug-infested fields attracted few indentured servants. As a result, enslaved Africans soon became the main source of workers and a large slave population developed. By the 1730s, the ratio of African Americans to European Americans was 2 to 1.

Return to Section Report Question: 17 of 120
Jump to question: 1 (answered) 2 (answered) 3 (answered) 4 (answered) 5 (answered) 6 (answered) 7 (answered) 8 (answered) 9 (answered) 10 (answered) 11 (answered) 12 (answered) 13 (answered) 14 (answered) 15 (answered) 16 (answered) 17 (answered) 18 (answered) 19 (answered) 20 (answered) 21 (answered) 22 (answered) 23 (answered) 24 (answered) 25 (answered) 26 (answered) 27 (answered) 28 (answered) 29 (answered) 30 (answered) 31 (answered) 32 (answered) 33 (answered) 34 (answered) 35 (answered) 36 (answered) 37 (answered) 38 (answered) 39 (answered) 40 (answered) 41 (answered) 42 (answered) 43 (answered) 44 (answered) 45 (answered) 46 (answered) 47 (answered) 48 (answered) 49 (answered) 50 (answered) 51 (answered) 52 (answered) 53 (answered) 54 (answered) 55 (answered) 56 (answered) 57 (answered) 58 (answered) 59 (answered) 60 (answered) 61 (answered) 62 (answered) 63 (answered) 64 (answered) 65 (answered) 66 (answered) 67 (answered) 68 (answered) 69 (answered) 70 (answered) 71 (answered) 72 (answered) 73 (answered) 74 (answered) 75 (answered) 76 (answered) 77 (answered) 78 (answered) 79 (answered) 80 (answered) 81 (answered) 82 (answered) 83 (answered) 84 (answered) 85 (answered) 86 (answered) 87 (answered) 88 (answered) 89 (answered) 90 (answered) 91 (answered) 92 (answered) 93 (answered) 94 (answered) 95 (answered) 96 (answered) 97 (answered) 98 (answered) 99 (answered)100 (answered)101 (answered)102 (answered)103 (answered)104 (answered)105 (answered)106 (answered)107 (answered)108 (answered)109 (answered)110 (answered)111 (answered)112 (answered)113 (answered)114 (answered)115 (answered)116 (answered)117 (answered)118 (answered)119 (answered)120 (answered)


According to which of the following theories were colonies primarily sources of raw materials for their home countries and markets for the latter's manufactured goods?

a. protectionism

b. capitalism

c. imperialism

d. mercantilism

e. socialism



The correct answer is D. Answer A refers to an economic theory that promotes high tariffs on imports to protect internal manufacturers and farmers from competition in their own national market. Answer B is an economic system in which the means of production are owned by individuals and manipulated for private profit. Answer C is a policy under which a powerful nation attempts to create an empire by dominating the economies, political systems, and cultural institutions of less powerful nations. Answer E is an economic system in which the means of production are owned by the public for the benefit of the public.

Return to Section Report Question: 18 of 120
Jump to question: 1 (answered) 2 (answered) 3 (answered) 4 (answered) 5 (answered) 6 (answered) 7 (answered) 8 (answered) 9 (answered) 10 (answered) 11 (answered) 12 (answered) 13 (answered) 14 (answered) 15 (answered) 16 (answered) 17 (answered) 18 (answered) 19 (answered) 20 (answered) 21 (answered) 22 (answered) 23 (answered) 24 (answered) 25 (answered) 26 (answered) 27 (answered) 28 (answered) 29 (answered) 30 (answered) 31 (answered) 32 (answered) 33 (answered) 34 (answered) 35 (answered) 36 (answered) 37 (answered) 38 (answered) 39 (answered) 40 (answered) 41 (answered) 42 (answered) 43 (answered) 44 (answered) 45 (answered) 46 (answered) 47 (answered) 48 (answered) 49 (answered) 50 (answered) 51 (answered) 52 (answered) 53 (answered) 54 (answered) 55 (answered) 56 (answered) 57 (answered) 58 (answered) 59 (answered) 60 (answered) 61 (answered) 62 (answered) 63 (answered) 64 (answered) 65 (answered) 66 (answered) 67 (answered) 68 (answered) 69 (answered) 70 (answered) 71 (answered) 72 (answered) 73 (answered) 74 (answered) 75 (answered) 76 (answered) 77 (answered) 78 (answered) 79 (answered) 80 (answered) 81 (answered) 82 (answered) 83 (answered) 84 (answered) 85 (answered) 86 (answered) 87 (answered) 88 (answered) 89 (answered) 90 (answered) 91 (answered) 92 (answered) 93 (answered) 94 (answered) 95 (answered) 96 (answered) 97 (answered) 98 (answered) 99 (answered)100 (answered)101 (answered)102 (answered)103 (answered)104 (answered)105 (answered)106 (answered)107 (answered)108 (answered)109 (answered)110 (answered)111 (answered)112 (answered)113 (answered)114 (answered)115 (answered)116 (answered)117 (answered)118 (answered)119 (answered)120 (answered)


"Great talkers, little doers."
Who probably wrote this aphorism?



a. Phillis Wheatley

b. Benjamin Franklin

c. Abigail Adams

d. Thomas Jefferson

e. John Dickinson



The correct answer is B. This aphorism, or saying, is from one of the issues of Franklin's Poor Richard's Almanac. Almanacs were popular forms of entertainment in the 1700s. Franklin in the guise of Poor Richard dispensed useful weather and crop information for sailors and farmers, political satire, and homey wisdom. Answer A, Phillis Wheatley, a former slave, was the first African American poet published in the colonies. Answer C, Abigail Adams, wife of John Adams, the second president of the United States, has become famous for her correspondence with her husband while he was a delegate to the Continental Congress. Answer D, Thomas Jefferson, is a good distracter, since one of the things he is known for is writing—the Declaration of Independence. Answer E, John Dickinson, is also a good distracter, because he was the author of Letters from a Farmer in Pennsylvania.

Return to Section Report Question: 19 of 120
Jump to question: 1 (answered) 2 (answered) 3 (answered) 4 (answered) 5 (answered) 6 (answered) 7 (answered) 8 (answered) 9 (answered) 10 (answered) 11 (answered) 12 (answered) 13 (answered) 14 (answered) 15 (answered) 16 (answered) 17 (answered) 18 (answered) 19 (answered) 20 (answered) 21 (answered) 22 (answered) 23 (answered) 24 (answered) 25 (answered) 26 (answered) 27 (answered) 28 (answered) 29 (answered) 30 (answered) 31 (answered) 32 (answered) 33 (answered) 34 (answered) 35 (answered) 36 (answered) 37 (answered) 38 (answered) 39 (answered) 40 (answered) 41 (answered) 42 (answered) 43 (answered) 44 (answered) 45 (answered) 46 (answered) 47 (answered) 48 (answered) 49 (answered) 50 (answered) 51 (answered) 52 (answered) 53 (answered) 54 (answered) 55 (answered) 56 (answered) 57 (answered) 58 (answered) 59 (answered) 60 (answered) 61 (answered) 62 (answered) 63 (answered) 64 (answered) 65 (answered) 66 (answered) 67 (answered) 68 (answered) 69 (answered) 70 (answered) 71 (answered) 72 (answered) 73 (answered) 74 (answered) 75 (answered) 76 (answered) 77 (answered) 78 (answered) 79 (answered) 80 (answered) 81 (answered) 82 (answered) 83 (answered) 84 (answered) 85 (answered) 86 (answered) 87 (answered) 88 (answered) 89 (answered) 90 (answered) 91 (answered) 92 (answered) 93 (answered) 94 (answered) 95 (answered) 96 (answered) 97 (answered) 98 (answered) 99 (answered)100 (answered)101 (answered)102 (answered)103 (answered)104 (answered)105 (answered)106 (answered)107 (answered)108 (answered)109 (answered)110 (answered)111 (answered)112 (answered)113 (answered)114 (answered)115 (answered)116 (answered)117 (answered)118 (answered)119 (answered)120 (answered)


Native Americans allied with the French in the French and Indian War because

a. the French paid better prices for Native American furs

b. the French were more interested in the fur trade than in colonization

c. the British refused to accept Native American aid

d. the French promised in any peace treaty to limit British settlements to east of the Appalachians

e. the British could not control the colonists who raided Native American villages



The correct answer is B. One of the main areas of conflict was the movement of more and more British colonists west of the Appalachians. The Native Americans sided with the French who held the territory as part of New France and were attempting to rout the settlers.

Return to Section Report Question: 20 of 120
Jump to question: 1 (answered) 2 (answered) 3 (answered) 4 (answered) 5 (answered) 6 (answered) 7 (answered) 8 (answered) 9 (answered) 10 (answered) 11 (answered) 12 (answered) 13 (answered) 14 (answered) 15 (answered) 16 (answered) 17 (answered) 18 (answered) 19 (answered) 20 (answered) 21 (answered) 22 (answered) 23 (answered) 24 (answered) 25 (answered) 26 (answered) 27 (answered) 28 (answered) 29 (answered) 30 (answered) 31 (answered) 32 (answered) 33 (answered) 34 (answered) 35 (answered) 36 (answered) 37 (answered) 38 (answered) 39 (answered) 40 (answered) 41 (answered) 42 (answered) 43 (answered) 44 (answered) 45 (answered) 46 (answered) 47 (answered) 48 (answered) 49 (answered) 50 (answered) 51 (answered) 52 (answered) 53 (answered) 54 (answered) 55 (answered) 56 (answered) 57 (answered) 58 (answered) 59 (answered) 60 (answered) 61 (answered) 62 (answered) 63 (answered) 64 (answered) 65 (answered) 66 (answered) 67 (answered) 68 (answered) 69 (answered) 70 (answered) 71 (answered) 72 (answered) 73 (answered) 74 (answered) 75 (answered) 76 (answered) 77 (answered) 78 (answered) 79 (answered) 80 (answered) 81 (answered) 82 (answered) 83 (answered) 84 (answered) 85 (answered) 86 (answered) 87 (answered) 88 (answered) 89 (answered) 90 (answered) 91 (answered) 92 (answered) 93 (answered) 94 (answered) 95 (answered) 96 (answered) 97 (answered) 98 (answered) 99 (answered)100 (answered)101 (answered)102 (answered)103 (answered)104 (answered)105 (answered)106 (answered)107 (answered)108 (answered)109 (answered)110 (answered)111 (answered)112 (answered)113 (answered)114 (answered)115 (answered)116 (answered)117 (answered)118 (answered)119 (answered)120 (answered)


As a result of Pontiac's Rebellion,

a. the French and Indian began

b. the Native American alliance was crushed at the Battle of Fallen Timbers

c. the British issued the Proclamation of 1763

d. the French closed the border with the British colonies

e. the British government negotiated the Treaty of Greenville



The correct answer is C. Answer A is incorrect; the results of the French and Indian led to Pontiac's Rebellion. Answers B and E are related, but belong to the early national period of U.S. history. President George Washington sent troops against a Native American uprising in the Old Northwest. The Native Americans were beaten at Fallen Timbers and forced to agree to the Treaty of Greenville. Answer D is illogical; the French were gone by 1763.

Return to Section Report Question: 21 of 120
Jump to question: 1 (answered) 2 (answered) 3 (answered) 4 (answered) 5 (answered) 6 (answered) 7 (answered) 8 (answered) 9 (answered) 10 (answered) 11 (answered) 12 (answered) 13 (answered) 14 (answered) 15 (answered) 16 (answered) 17 (answered) 18 (answered) 19 (answered) 20 (answered) 21 (answered) 22 (answered) 23 (answered) 24 (answered) 25 (answered) 26 (answered) 27 (answered) 28 (answered) 29 (answered) 30 (answered) 31 (answered) 32 (answered) 33 (answered) 34 (answered) 35 (answered) 36 (answered) 37 (answered) 38 (answered) 39 (answered) 40 (answered) 41 (answered) 42 (answered) 43 (answered) 44 (answered) 45 (answered) 46 (answered) 47 (answered) 48 (answered) 49 (answered) 50 (answered) 51 (answered) 52 (answered) 53 (answered) 54 (answered) 55 (answered) 56 (answered) 57 (answered) 58 (answered) 59 (answered) 60 (answered) 61 (answered) 62 (answered) 63 (answered) 64 (answered) 65 (answered) 66 (answered) 67 (answered) 68 (answered) 69 (answered) 70 (answered) 71 (answered) 72 (answered) 73 (answered) 74 (answered) 75 (answered) 76 (answered) 77 (answered) 78 (answered) 79 (answered) 80 (answered) 81 (answered) 82 (answered) 83 (answered) 84 (answered) 85 (answered) 86 (answered) 87 (answered) 88 (answered) 89 (answered) 90 (answered) 91 (answered) 92 (answered) 93 (answered) 94 (answered) 95 (answered) 96 (answered) 97 (answered) 98 (answered) 99 (answered)100 (answered)101 (answered)102 (answered)103 (answered)104 (answered)105 (answered)106 (answered)107 (answered)108 (answered)109 (answered)110 (answered)111 (answered)112 (answered)113 (answered)114 (answered)115 (answered)116 (answered)117 (answered)118 (answered)119 (answered)120 (answered)


The most popular subject for a painting among upper- and middle-class people in the colonial period was

a. a landscape

b. an historical theme

c. a portrait

d. everyday life

e. a religious subject



The correct answer is C. Like British aristocrats, wealthy landowners, merchants, and professionals as well as wealthy artisans wanted themselves and their family members captured for posterity. Paintings of landscapes and everyday life, answers A and D, became very popular during the romantic period in the 1800s. Answer B, historical themes, were popular so long as the subjects glorified the new nation. The popularity of religious themes, answer E, was somewhat limited.

Return to Section Report Question: 22 of 120
Jump to question: 1 (answered) 2 (answered) 3 (answered) 4 (answered) 5 (answered) 6 (answered) 7 (answered) 8 (answered) 9 (answered) 10 (answered) 11 (answered) 12 (answered) 13 (answered) 14 (answered) 15 (answered) 16 (answered) 17 (answered) 18 (answered) 19 (answered) 20 (answered) 21 (answered) 22 (answered) 23 (answered) 24 (answered) 25 (answered) 26 (answered) 27 (answered) 28 (answered) 29 (answered) 30 (answered) 31 (answered) 32 (answered) 33 (answered) 34 (answered) 35 (answered) 36 (answered) 37 (answered) 38 (answered) 39 (answered) 40 (answered) 41 (answered) 42 (answered) 43 (answered) 44 (answered) 45 (answered) 46 (answered) 47 (answered) 48 (answered) 49 (answered) 50 (answered) 51 (answered) 52 (answered) 53 (answered) 54 (answered) 55 (answered) 56 (answered) 57 (answered) 58 (answered) 59 (answered) 60 (answered) 61 (answered) 62 (answered) 63 (answered) 64 (answered) 65 (answered) 66 (answered) 67 (answered) 68 (answered) 69 (answered) 70 (answered) 71 (answered) 72 (answered) 73 (answered) 74 (answered) 75 (answered) 76 (answered) 77 (answered) 78 (answered) 79 (answered) 80 (answered) 81 (answered) 82 (answered) 83 (answered) 84 (answered) 85 (answered) 86 (answered) 87 (answered) 88 (answered) 89 (answered) 90 (answered) 91 (answered) 92 (answered) 93 (answered) 94 (answered) 95 (answered) 96 (answered) 97 (answered) 98 (answered) 99 (answered)100 (answered)101 (answered)102 (answered)103 (answered)104 (answered)105 (answered)106 (answered)107 (answered)108 (answered)109 (answered)110 (answered)111 (answered)112 (answered)113 (answered)114 (answered)115 (answered)116 (answered)117 (answered)118 (answered)119 (answered)120 (answered)


Which of the following groups led attempts to abolish slavery before the American Revolution?

a. Roman Catholics

b. Puritans

c. white Southern artisans and craftworkers

d. Quakers

e. the New York legislature



The correct answer is D. Maryland, the colony most closely associated with Roman Catholics, answer A, was the second colony to make slavery a lifelong condition. Answer B is incorrect. Answer C is incorrect; there were few non-African American artisans and craftworkers in the South. Most of this kind of work was done by slaves. Answer E is incorrect; New York's legislature legalized slavery in the late 1600s.

Return to Section Report Question: 23 of 120
Jump to question: 1 (answered) 2 (answered) 3 (answered) 4 (answered) 5 (answered) 6 (answered) 7 (answered) 8 (answered) 9 (answered) 10 (answered) 11 (answered) 12 (answered) 13 (answered) 14 (answered) 15 (answered) 16 (answered) 17 (answered) 18 (answered) 19 (answered) 20 (answered) 21 (answered) 22 (answered) 23 (answered) 24 (answered) 25 (answered) 26 (answered) 27 (answered) 28 (answered) 29 (answered) 30 (answered) 31 (answered) 32 (answered) 33 (answered) 34 (answered) 35 (answered) 36 (answered) 37 (answered) 38 (answered) 39 (answered) 40 (answered) 41 (answered) 42 (answered) 43 (answered) 44 (answered) 45 (answered) 46 (answered) 47 (answered) 48 (answered) 49 (answered) 50 (answered) 51 (answered) 52 (answered) 53 (answered) 54 (answered) 55 (answered) 56 (answered) 57 (answered) 58 (answered) 59 (answered) 60 (answered) 61 (answered) 62 (answered) 63 (answered) 64 (answered) 65 (answered) 66 (answered) 67 (answered) 68 (answered) 69 (answered) 70 (answered) 71 (answered) 72 (answered) 73 (answered) 74 (answered) 75 (answered) 76 (answered) 77 (answered) 78 (answered) 79 (answered) 80 (answered) 81 (answered) 82 (answered) 83 (answered) 84 (answered) 85 (answered) 86 (answered) 87 (answered) 88 (answered) 89 (answered) 90 (answered) 91 (answered) 92 (answered) 93 (answered) 94 (answered) 95 (answered) 96 (answered) 97 (answered) 98 (answered) 99 (answered)100 (answered)101 (answered)102 (answered)103 (answered)104 (answered)105 (answered)106 (answered)107 (answered)108 (answered)109 (answered)110 (answered)111 (answered)112 (answered)113 (answered)114 (answered)115 (answered)116 (answered)117 (answered)118 (answered)119 (answered)120 (answered)


Of the following colonies, which had the most diverse ethnic and religious population?

a. Massachusetts Bay

b. Pennsylvania

c. Virginia

d. Rhode Island

e. North Carolina



The correct answer is B. The Middle Colonies had a smaller English population and more nationalities with their diverse religions than either of the other two regions. Since Pennsylvania is the only Middle Colony listed, it's the answer. Even if you didn't know the answer, you could eliminate answers to get to it. Answers A and B were New England colonies whose strict religious practices and limiting of political rights to church members along with the climate and poor soil made them unattractive to other ethnic and religious groups. Based on numbers alone, you can rule out answers C and E They were Southern Colonies, whose large African populations meant that close to half the populations of both were homogeneous.

Return to Section Report Question: 24 of 120
Jump to question: 1 (answered) 2 (answered) 3 (answered) 4 (answered) 5 (answered) 6 (answered) 7 (answered) 8 (answered) 9 (answered) 10 (answered) 11 (answered) 12 (answered) 13 (answered) 14 (answered) 15 (answered) 16 (answered) 17 (answered) 18 (answered) 19 (answered) 20 (answered) 21 (answered) 22 (answered) 23 (answered) 24 (answered) 25 (answered) 26 (answered) 27 (answered) 28 (answered) 29 (answered) 30 (answered) 31 (answered) 32 (answered) 33 (answered) 34 (answered) 35 (answered) 36 (answered) 37 (answered) 38 (answered) 39 (answered) 40 (answered) 41 (answered) 42 (answered) 43 (answered) 44 (answered) 45 (answered) 46 (answered) 47 (answered) 48 (answered) 49 (answered) 50 (answered) 51 (answered) 52 (answered) 53 (answered) 54 (answered) 55 (answered) 56 (answered) 57 (answered) 58 (answered) 59 (answered) 60 (answered) 61 (answered) 62 (answered) 63 (answered) 64 (answered) 65 (answered) 66 (answered) 67 (answered) 68 (answered) 69 (answered) 70 (answered) 71 (answered) 72 (answered) 73 (answered) 74 (answered) 75 (answered) 76 (answered) 77 (answered) 78 (answered) 79 (answered) 80 (answered) 81 (answered) 82 (answered) 83 (answered) 84 (answered) 85 (answered) 86 (answered) 87 (answered) 88 (answered) 89 (answered) 90 (answered) 91 (answered) 92 (answered) 93 (answered) 94 (answered) 95 (answered) 96 (answered) 97 (answered) 98 (answered) 99 (answered)100 (answered)101 (answered)102 (answered)103 (answered)104 (answered)105 (answered)106 (answered)107 (answered)108 (answered)109 (answered)110 (answered)111 (answered)112 (answered)113 (answered)114 (answered)115 (answered)116 (answered)117 (answered)118 (answered)119 (answered)120 (answered)


All of the following are characteristics of the British colonial system as applied to the North American colonies EXCEPT

a. English common law as the legal system

b. representative colonial assemblies

c. participation in the British trading network

d. centralized government for the colonies

e. oversight by the Board of Trade



The correct answer is D. The lack of a centralized government was a major reason why Parliament and the monarchy could not control the activities of the colonies. This lack of centralization may also be the reason for the framers choice of a federal system for the new United States.

Return to Section Report Question: 25 of 120
Jump to question: 1 (answered) 2 (answered) 3 (answered) 4 (answered) 5 (answered) 6 (answered) 7 (answered) 8 (answered) 9 (answered) 10 (answered) 11 (answered) 12 (answered) 13 (answered) 14 (answered) 15 (answered) 16 (answered) 17 (answered) 18 (answered) 19 (answered) 20 (answered) 21 (answered) 22 (answered) 23 (answered) 24 (answered) 25 (answered) 26 (answered) 27 (answered) 28 (answered) 29 (answered) 30 (answered) 31 (answered) 32 (answered) 33 (answered) 34 (answered) 35 (answered) 36 (answered) 37 (answered) 38 (answered) 39 (answered) 40 (answered) 41 (answered) 42 (answered) 43 (answered) 44 (answered) 45 (answered) 46 (answered) 47 (answered) 48 (answered) 49 (answered) 50 (answered) 51 (answered) 52 (answered) 53 (answered) 54 (answered) 55 (answered) 56 (answered) 57 (answered) 58 (answered) 59 (answered) 60 (answered) 61 (answered) 62 (answered) 63 (answered) 64 (answered) 65 (answered) 66 (answered) 67 (answered) 68 (answered) 69 (answered) 70 (answered) 71 (answered) 72 (answered) 73 (answered) 74 (answered) 75 (answered) 76 (answered) 77 (answered) 78 (answered) 79 (answered) 80 (answered) 81 (answered) 82 (answered) 83 (answered) 84 (answered) 85 (answered) 86 (answered) 87 (answered) 88 (answered) 89 (answered) 90 (answered) 91 (answered) 92 (answered) 93 (answered) 94 (answered) 95 (answered) 96 (answered) 97 (answered) 98 (answered) 99 (answered)100 (answered)101 (answered)102 (answered)103 (answered)104 (answered)105 (answered)106 (answered)107 (answered)108 (answered)109 (answered)110 (answered)111 (answered)112 (answered)113 (answered)114 (answered)115 (answered)116 (answered)117 (answered)118 (answered)119 (answered)120 (answered)


The first attempt by colonists to unite for a political purpose was

a. Annapolis Convention

b. American System

c. Hartford Convention

d. Albany Plan of Union

e. Dominion of New England



The correct answer is D. Answer D was proposed by Benjamin Franklin and modeled after the Iroquois Confederacy. Although the delegates in Albany agreed, neither Parliament nor the colonial legislatures approved the plan. Answer A was the convention called to revise the Articles of Confederation, which instead called for a convention to write a new constitution. Answer B was Henry Clay's proposal for an economic system that would provide financial aid to each section of the new nation. Answer C was the assembly called by New England Federalists who opposed the War of 1812; its timing and its support of states' rights brought discredit to the Federalist Party. Answer E was the political unit created by Parliament in 1684 to govern all land north of Pennsylvania. With the Glorious Restoration in 1688, this was later reversed.

Return to Section Report Question: 26 of 120
Jump to question: 1 (answered) 2 (answered) 3 (answered) 4 (answered) 5 (answered) 6 (answered) 7 (answered) 8 (answered) 9 (answered) 10 (answered) 11 (answered) 12 (answered) 13 (answered) 14 (answered) 15 (answered) 16 (answered) 17 (answered) 18 (answered) 19 (answered) 20 (answered) 21 (answered) 22 (answered) 23 (answered) 24 (answered) 25 (answered) 26 (answered) 27 (answered) 28 (answered) 29 (answered) 30 (answered) 31 (answered) 32 (answered) 33 (answered) 34 (answered) 35 (answered) 36 (answered) 37 (answered) 38 (answered) 39 (answered) 40 (answered) 41 (answered) 42 (answered) 43 (answered) 44 (answered) 45 (answered) 46 (answered) 47 (answered) 48 (answered) 49 (answered) 50 (answered) 51 (answered) 52 (answered) 53 (answered) 54 (answered) 55 (answered) 56 (answered) 57 (answered) 58 (answered) 59 (answered) 60 (answered) 61 (answered) 62 (answered) 63 (answered) 64 (answered) 65 (answered) 66 (answered) 67 (answered) 68 (answered) 69 (answered) 70 (answered) 71 (answered) 72 (answered) 73 (answered) 74 (answered) 75 (answered) 76 (answered) 77 (answered) 78 (answered) 79 (answered) 80 (answered) 81 (answered) 82 (answered) 83 (answered) 84 (answered) 85 (answered) 86 (answered) 87 (answered) 88 (answered) 89 (answered) 90 (answered) 91 (answered) 92 (answered) 93 (answered) 94 (answered) 95 (answered) 96 (answered) 97 (answered) 98 (answered) 99 (answered)100 (answered)101 (answered)102 (answered)103 (answered)104 (answered)105 (answered)106 (answered)107 (answered)108 (answered)109 (answered)110 (answered)111 (answered)112 (answered)113 (answered)114 (answered)115 (answered)116 (answered)117 (answered)118 (answered)119 (answered)120 (answered)


Which of the following is not a true statement about the Navigation Acts?



a. One purpose of the Navigation Acts was to ensure that raw materials were shipped only to England.

b. A second purpose of the Navigation Acts was to punish English sailors who jumped ship.

c. Under the Navigation Acts, all colonial exports and imports had to be shipped through England.

d. Initial colonial opposition to the Navigation Acts was light, because the acts were not rigidly enforced.

e. One purpose of the Navigation Acts was to cut out other nations from profitable trade with the colonies.



The correct answer is B. Don't confuse the Navigation Acts with the impressment of sailors that precipitated conflict with Great Britain in the early 1800s. In answer E, the major trade competitor at the time was the Netherlands.

Return to Section Report Question: 27 of 120
Jump to question: 1 (answered) 2 (answered) 3 (answered) 4 (answered) 5 (answered) 6 (answered) 7 (answered) 8 (answered) 9 (answered) 10 (answered) 11 (answered) 12 (answered) 13 (answered) 14 (answered) 15 (answered) 16 (answered) 17 (answered) 18 (answered) 19 (answered) 20 (answered) 21 (answered) 22 (answered) 23 (answered) 24 (answered) 25 (answered) 26 (answered) 27 (answered) 28 (answered) 29 (answered) 30 (answered) 31 (answered) 32 (answered) 33 (answered) 34 (answered) 35 (answered) 36 (answered) 37 (answered) 38 (answered) 39 (answered) 40 (answered) 41 (answered) 42 (answered) 43 (answered) 44 (answered) 45 (answered) 46 (answered) 47 (answered) 48 (answered) 49 (answered) 50 (answered) 51 (answered) 52 (answered) 53 (answered) 54 (answered) 55 (answered) 56 (answered) 57 (answered) 58 (answered) 59 (answered) 60 (answered) 61 (answered) 62 (answered) 63 (answered) 64 (answered) 65 (answered) 66 (answered) 67 (answered) 68 (answered) 69 (answered) 70 (answered) 71 (answered) 72 (answered) 73 (answered) 74 (answered) 75 (answered) 76 (answered) 77 (answered) 78 (answered) 79 (answered) 80 (answered) 81 (answered) 82 (answered) 83 (answered) 84 (answered) 85 (answered) 86 (answered) 87 (answered) 88 (answered) 89 (answered) 90 (answered) 91 (answered) 92 (answered) 93 (answered) 94 (answered) 95 (answered) 96 (answered) 97 (answered) 98 (answered) 99 (answered)100 (answered)101 (answered)102 (answered)103 (answered)104 (answered)105 (answered)106 (answered)107 (answered)108 (answered)109 (answered)110 (answered)111 (answered)112 (answered)113 (answered)114 (answered)115 (answered)116 (answered)117 (answered)118 (answered)119 (answered)120 (answered)


Which of the following asserted Parliament's right to tax the colonies to raise revenue?

a. Stamp Act

b. Molasses Act

c. Sugar Act

d. Currency Act

e. Hat Act



The correct answer is C. The previous acts, answers B and E, had been meant to regulate trade. Answer A, Stamp Act, was the first internal tax, that is, on business within the colonies and not between the colonies and Great Britain or another nation. Answer D forbade the colonies to issue their own money.

Return to Section Report Question: 28 of 120
Jump to question: 1 (answered) 2 (answered) 3 (answered) 4 (answered) 5 (answered) 6 (answered) 7 (answered) 8 (answered) 9 (answered) 10 (answered) 11 (answered) 12 (answered) 13 (answered) 14 (answered) 15 (answered) 16 (answered) 17 (answered) 18 (answered) 19 (answered) 20 (answered) 21 (answered) 22 (answered) 23 (answered) 24 (answered) 25 (answered) 26 (answered) 27 (answered) 28 (answered) 29 (answered) 30 (answered) 31 (answered) 32 (answered) 33 (answered) 34 (answered) 35 (answered) 36 (answered) 37 (answered) 38 (answered) 39 (answered) 40 (answered) 41 (answered) 42 (answered) 43 (answered) 44 (answered) 45 (answered) 46 (answered) 47 (answered) 48 (answered) 49 (answered) 50 (answered) 51 (answered) 52 (answered) 53 (answered) 54 (answered) 55 (answered) 56 (answered) 57 (answered) 58 (answered) 59 (answered) 60 (answered) 61 (answered) 62 (answered) 63 (answered) 64 (answered) 65 (answered) 66 (answered) 67 (answered) 68 (answered) 69 (answered) 70 (answered) 71 (answered) 72 (answered) 73 (answered) 74 (answered) 75 (answered) 76 (answered) 77 (answered) 78 (answered) 79 (answered) 80 (answered) 81 (answered) 82 (answered) 83 (answered) 84 (answered) 85 (answered) 86 (answered) 87 (answered) 88 (answered) 89 (answered) 90 (answered) 91 (answered) 92 (answered) 93 (answered) 94 (answered) 95 (answered) 96 (answered) 97 (answered) 98 (answered) 99 (answered)100 (answered)101 (answered)102 (answered)103 (answered)104 (answered)105 (answered)106 (answered)107 (answered)108 (answered)109 (answered)110 (answered)111 (answered)112 (answered)113 (answered)114 (answered)115 (answered)116 (answered)117 (answered)118 (answered)119 (answered)120 (answered)


Which of the following urged the establishment of Committees of Correspondence?

a. Sam Adams

b. Paul Revere

c. John Hancock

d. Patrick Henry

e. James Otis



The correct answer is A. Answer B, Paul Revere, was not only the spreader of the alarm "the British are coming," but also a participant in the Boston Tea Party and silversmith and engraver by trade. He designed the new nation's first currency. Answer C, John Hancock, was the first signer of the Declaration of Independence and president of the Second Continental Congress at the time. Answer D, Patrick Henry, is famous for the phrase "give me liberty or give me death," his support for independence, and his opposition to ratification of the U.S. Constitution. Answer E, James Otis, with Sam Adams wrote the Circular Letter that set out the colonists' opposition to the Townshend Acts.

Return to Section Report Question: 29 of 120
Jump to question: 1 (answered) 2 (answered) 3 (answered) 4 (answered) 5 (answered) 6 (answered) 7 (answered) 8 (answered) 9 (answered) 10 (answered) 11 (answered) 12 (answered) 13 (answered) 14 (answered) 15 (answered) 16 (answered) 17 (answered) 18 (answered) 19 (answered) 20 (answered) 21 (answered) 22 (answered) 23 (answered) 24 (answered) 25 (answered) 26 (answered) 27 (answered) 28 (answered) 29 (answered) 30 (answered) 31 (answered) 32 (answered) 33 (answered) 34 (answered) 35 (answered) 36 (answered) 37 (answered) 38 (answered) 39 (answered) 40 (answered) 41 (answered) 42 (answered) 43 (answered) 44 (answered) 45 (answered) 46 (answered) 47 (answered) 48 (answered) 49 (answered) 50 (answered) 51 (answered) 52 (answered) 53 (answered) 54 (answered) 55 (answered) 56 (answered) 57 (answered) 58 (answered) 59 (answered) 60 (answered) 61 (answered) 62 (answered) 63 (answered) 64 (answered) 65 (answered) 66 (answered) 67 (answered) 68 (answered) 69 (answered) 70 (answered) 71 (answered) 72 (answered) 73 (answered) 74 (answered) 75 (answered) 76 (answered) 77 (answered) 78 (answered) 79 (answered) 80 (answered) 81 (answered) 82 (answered) 83 (answered) 84 (answered) 85 (answered) 86 (answered) 87 (answered) 88 (answered) 89 (answered) 90 (answered) 91 (answered) 92 (answered) 93 (answered) 94 (answered) 95 (answered) 96 (answered) 97 (answered) 98 (answered) 99 (answered)100 (answered)101 (answered)102 (answered)103 (answered)104 (answered)105 (answered)106 (answered)107 (answered)108 (answered)109 (answered)110 (answered)111 (answered)112 (answered)113 (answered)114 (answered)115 (answered)116 (answered)117 (answered)118 (answered)119 (answered)120 (answered)


What is the significance of the Intolerable Acts?



a. The Intolerable Acts closed the port of Boston and dismissed the Massachusetts legislature.

b. taken with the Quebec Act, the Intolerable Acts showed a lack of understanding of colonial interests.

c. The Intolerable Acts took away all civil rights from the citizens of Canada.

d. In passing the Intolerable Acts, Parliament shifted its focus from persuasion to punishment.

e. Parliament focused on specific issues rather than on the broader picture of colonial discontent.



The correct answer is D. Answer A is incorrect, because the questions asks for the significance of the Intolerable Acts. Answer A merely tells you some of the provisions of the acts. Answer B is incorrect, because the acts hit directly at colonial interests, for example, trade, self-government, and territorial expansion. Answer C is incorrect; the Quebec Act, which was passed at the same time as the Coercive Acts (the proper title for the Intolerable Acts), allowed the French citizens of Quebec to keep the French civil code of laws, which was more limited in the areas of rights than the English code. Answer E is incorrect, because underlying the provisions of the acts was the broader issue of self-government.

Return to Section Report Question: 30 of 120
Jump to question: 1 (answered) 2 (answered) 3 (answered) 4 (answered) 5 (answered) 6 (answered) 7 (answered) 8 (answered) 9 (answered) 10 (answered) 11 (answered) 12 (answered) 13 (answered) 14 (answered) 15 (answered) 16 (answered) 17 (answered) 18 (answered) 19 (answered) 20 (answered) 21 (answered) 22 (answered) 23 (answered) 24 (answered) 25 (answered) 26 (answered) 27 (answered) 28 (answered) 29 (answered) 30 (answered) 31 (answered) 32 (answered) 33 (answered) 34 (answered) 35 (answered) 36 (answered) 37 (answered) 38 (answered) 39 (answered) 40 (answered) 41 (answered) 42 (answered) 43 (answered) 44 (answered) 45 (answered) 46 (answered) 47 (answered) 48 (answered) 49 (answered) 50 (answered) 51 (answered) 52 (answered) 53 (answered) 54 (answered) 55 (answered) 56 (answered) 57 (answered) 58 (answered) 59 (answered) 60 (answered) 61 (answered) 62 (answered) 63 (answered) 64 (answered) 65 (answered) 66 (answered) 67 (answered) 68 (answered) 69 (answered) 70 (answered) 71 (answered) 72 (answered) 73 (answered) 74 (answered) 75 (answered) 76 (answered) 77 (answered) 78 (answered) 79 (answered) 80 (answered) 81 (answered) 82 (answered) 83 (answered) 84 (answered) 85 (answered) 86 (answered) 87 (answered) 88 (answered) 89 (answered) 90 (answered) 91 (answered) 92 (answered) 93 (answered) 94 (answered) 95 (answered) 96 (answered) 97 (answered) 98 (answered) 99 (answered)100 (answered)101 (answered)102 (answered)103 (answered)104 (answered)105 (answered)106 (answered)107 (answered)108 (answered)109 (answered)110 (answered)111 (answered)112 (answered)113 (answered)114 (answered)115 (answered)116 (answered)117 (answered)118 (answered)119 (answered)120 (answered)


Which of the following is NOT a true statement about the Olive Branch Petition?

a. It was rejected by George III.

b. It was a concession to moderates in the Second Continental Congress.

c. It was followed by the colonists' Declaration of the Causes and Necessity of Taking Up Arms.

d. It was directed to the monarch and Parliament.

e. It was countered by George III's issuance of the Proclamation of Rebellion.



The correct answer is D. The Olive Branch Petition was addressed solely to George III and restated the colonists' loyalty to the king.

Return to Section Report Question: 31 of 120
Jump to question: 1 (answered) 2 (answered) 3 (answered) 4 (answered) 5 (answered) 6 (answered) 7 (answered) 8 (answered) 9 (answered) 10 (answered) 11 (answered) 12 (answered) 13 (answered) 14 (answered) 15 (answered) 16 (answered) 17 (answered) 18 (answered) 19 (answered) 20 (answered) 21 (answered) 22 (answered) 23 (answered) 24 (answered) 25 (answered) 26 (answered) 27 (answered) 28 (answered) 29 (answered) 30 (answered) 31 (answered) 32 (answered) 33 (answered) 34 (answered) 35 (answered) 36 (answered) 37 (answered) 38 (answered) 39 (answered) 40 (answered) 41 (answered) 42 (answered) 43 (answered) 44 (answered) 45 (answered) 46 (answered) 47 (answered) 48 (answered) 49 (answered) 50 (answered) 51 (answered) 52 (answered) 53 (answered) 54 (answered) 55 (answered) 56 (answered) 57 (answered) 58 (answered) 59 (answered) 60 (answered) 61 (answered) 62 (answered) 63 (answered) 64 (answered) 65 (answered) 66 (answered) 67 (answered) 68 (answered) 69 (answered) 70 (answered) 71 (answered) 72 (answered) 73 (answered) 74 (answered) 75 (answered) 76 (answered) 77 (answered) 78 (answered) 79 (answered) 80 (answered) 81 (answered) 82 (answered) 83 (answered) 84 (answered) 85 (answered) 86 (answered) 87 (answered) 88 (answered) 89 (answered) 90 (answered) 91 (answered) 92 (answered) 93 (answered) 94 (answered) 95 (answered) 96 (answered) 97 (answered) 98 (answered) 99 (answered)100 (answered)101 (answered)102 (answered)103 (answered)104 (answered)105 (answered)106 (answered)107 (answered)108 (answered)109 (answered)110 (answered)111 (answered)112 (answered)113 (answered)114 (answered)115 (answered)116 (answered)117 (answered)118 (answered)119 (answered)120 (answered)


Which of the following is not a reason why colonists hesitated to declare independence in 1775?



a. How to attract foreign support was an unresolved issue.

b. Many could not see themselves as traitors, which is what a declaration of independence would mean.

c. How would Americans organize themselves since nations at the time had some form of monarchy.

d. Breaking political ties with England meant turning their back on their heritage.

e. Once unleashed, the fury of ordinary colonists could turn against the well to do and the rebellion could slide into anarchy.



The correct answer is A. Answer A is incorrect, because in 1775 the Second Continental Congress had sent representatives to France, Spain, and the Netherlands, Great Britain's traditional enemies, looking for military and economic aid. The wealthy, traditional, and more conservative colonial leaders had already seen the results of mob violence in the stoning of tax collectors, the destruction of property, and mass rallies and parades, so answer E was a real concern to them.

Return to Section Report Question: 32 of 120
Jump to question: 1 (answered) 2 (answered) 3 (answered) 4 (answered) 5 (answered) 6 (answered) 7 (answered) 8 (answered) 9 (answered) 10 (answered) 11 (answered) 12 (answered) 13 (answered) 14 (answered) 15 (answered) 16 (answered) 17 (answered) 18 (answered) 19 (answered) 20 (answered) 21 (answered) 22 (answered) 23 (answered) 24 (answered) 25 (answered) 26 (answered) 27 (answered) 28 (answered) 29 (answered) 30 (answered) 31 (answered) 32 (answered) 33 (answered) 34 (answered) 35 (answered) 36 (answered) 37 (answered) 38 (answered) 39 (answered) 40 (answered) 41 (answered) 42 (answered) 43 (answered) 44 (answered) 45 (answered) 46 (answered) 47 (answered) 48 (answered) 49 (answered) 50 (answered) 51 (answered) 52 (answered) 53 (answered) 54 (answered) 55 (answered) 56 (answered) 57 (answered) 58 (answered) 59 (answered) 60 (answered) 61 (answered) 62 (answered) 63 (answered) 64 (answered) 65 (answered) 66 (answered) 67 (answered) 68 (answered) 69 (answered) 70 (answered) 71 (answered) 72 (answered) 73 (answered) 74 (answered) 75 (answered) 76 (answered) 77 (answered) 78 (answered) 79 (answered) 80 (answered) 81 (answered) 82 (answered) 83 (answered) 84 (answered) 85 (answered) 86 (answered) 87 (answered) 88 (answered) 89 (answered) 90 (answered) 91 (answered) 92 (answered) 93 (answered) 94 (answered) 95 (answered) 96 (answered) 97 (answered) 98 (answered) 99 (answered)100 (answered)101 (answered)102 (answered)103 (answered)104 (answered)105 (answered)106 (answered)107 (answered)108 (answered)109 (answered)110 (answered)111 (answered)112 (answered)113 (answered)114 (answered)115 (answered)116 (answered)117 (answered)118 (answered)119 (answered)120 (answered)


"These are the times that try men's souls. The summer soldier and the sunshine patriot will in this crisis, shrink from the service of his country; but he that stands it NOW, deserves the love and thanks of man and woman."
This quotation was most likely written by



a. George Washington

b. Thomas Jefferson

c. Thomas Paine

d. Hector St. John de Crevecoeur

e. James Fenimore Cooper



The correct answer is C. There is a clue in the second sentence, the word crisis. This quotation is from The Crisis by Thomas Paine. Answer A, George Washington, is not generally know for any writings calling his fellow colonists to rise up in rebellion. Answer B, Thomas Jefferson is just wrong; sometimes you can't always eliminate an answer based on elimination. Answer D, Hector St. John de Crevecoeur, wrote Letters from an American Farmer, published in 1782, a documentary of life in the colonies and new nation. Answer E, James Fenimore Cooper, was the first novelist in the early national period to use American themes in his works.

Return to Section Report Question: 33 of 120
Jump to question: 1 (answered) 2 (answered) 3 (answered) 4 (answered) 5 (answered) 6 (answered) 7 (answered) 8 (answered) 9 (answered) 10 (answered) 11 (answered) 12 (answered) 13 (answered) 14 (answered) 15 (answered) 16 (answered) 17 (answered) 18 (answered) 19 (answered) 20 (answered) 21 (answered) 22 (answered) 23 (answered) 24 (answered) 25 (answered) 26 (answered) 27 (answered) 28 (answered)
As far as the British government was concerned, the reason for the founding of Georgia was

a. to establish a colony for the poor from English debtors' prisons

b. as a haven for religious dissenters

c. to return a profit for the Crown

d. to establish a colony based on slavery from its beginning

e. to establish a buffer between South Carolina and Spanish Florida
The correct answer is E. The key phrase in the question is "as far as the British government." Answer A was a reason of James Oglethorpe, one of the eight original proprietors, but it was not the reason the government was interested. Don't confuse Georgia with Plymouth, Massachusetts Bay, Maryland, or Pennsylvania, answer B. Answer C is incorrect, because Georgia was a proprietary colony. Answer D is the opposite of what was true; the proprietors banned slavery at the inception of the colony, but later gave in to angry colonists who said they could not turn a profit without the use of slaves.
The large urban centers of the North American colonies were located mostly

a. in the South

b. in the Middle Colonies

c. in the interior

d. in New England

e. in valleys
The correct answer is B. The Middle Colonies were more welcoming to immigrants than New England, answer D, was—both the environment and in policies toward religion. The weather was milder and the land, more fertile. A larger population meant not only a larger area under settlement, but also an increase in the number of townspeople per village, town, and city. Answer A is incorrect. Most of the southern population was rural, living either on small farms or on plantations. Answer C is incorrect. Relatively speaking, little of the interior had been settled by 1776. The majority of the colonial population still lived along the coast. Answer E is incorrect. The majority of the population lived along the coastal plain.
An unintended result of the First Great Awakening was

a. a new level of intolerance toward all religions

b. a more democratic outlook among colonists and less reliance on traditional authority and social class

c. a general lessening of interest in religion

d. that evangelism delayed the establishment of schools and colleges

e. a dramatic increase in membership in traditional churches
The correct answer is B. There is no evidence to support answer A. Common sense eliminates answer C, since the question is about a revival of interest in religion. Answer D is the opposite of what occurred. A number of colleges were founded in this period as a result of the idea that people could think for themselves. Answer E is incorrect; many people left the older churches to join newer ones like Methodism and Presbyterianism.
Which of the following would be least likely to be found in a colonial gentleman's library in the mid-1700s?

a. writings of John Locke

b. book of sermons

c. Latin version of the Aenid

d. novel written by a colonial author

e. collection of poetry by an English poet
The correct answer is D. While colonists may have been developing their own uniquely American view of politics, their literary tastes were still very much influenced by what the British wrote and read, which included religious writings. The American novel did not come into its own until the early national period with the works of James Fenimore Cooper.
The most important port and the largest city in the North American colonies by 1776 was

a. Charleston

b. Baltimore

c. Philadelphia

d. New York

e. Boston
The correct answer is C. Answer A, Charleston, could be ruled out, because it was in a southern colony and there was less population in that region than in the Middle or New England colonies. For the same reason, rule out answer B, Baltimore. That would leave you with three good choices, but the long, cold winters could help you rule out Boston, answer E. Remember that the delegates to the Continental Congress chose Philadelphia as the site of their meetings, so it must have been a big and important city. Pennsylvania by the 1700s had become known as the breadbasket of the colonies, which means that it grew a lot of food, more than its people used, which means it had to ship it from someplace—Philadelphia.
The only colony in which enslaved African Americans outnumbered European Americans in the mid-1700s was

a. Maryland

b. Virginia

c. North Carolina

d. South Carolina

e. Georgia
The correct answer is D. If you didn't know this answer, you could at least eliminate Georgia, because it was not founded until 1732 and it originally banned slavery. South Carolina's major export was rice, not tobacco, and the unhealthy work of cultivating rice in wet, bug-infested fields attracted few indentured servants. As a result, enslaved Africans soon became the main source of workers and a large slave population developed. By the 1730s, the ratio of African Americans to European Americans was 2 to 1.
According to which of the following theories were colonies primarily sources of raw materials for their home countries and markets for the latter's manufactured goods?

a. protectionism

b. capitalism

c. imperialism

d. mercantilism

e. socialism
The correct answer is D. Answer A refers to an economic theory that promotes high tariffs on imports to protect internal manufacturers and farmers from competition in their own national market. Answer B is an economic system in which the means of production are owned by individuals and manipulated for private profit. Answer C is a policy under which a powerful nation attempts to create an empire by dominating the economies, political systems, and cultural institutions of less powerful nations. Answer E is an economic system in which the means of production are owned by the public for the benefit of the public.
Great talkers, little doers."
Who probably wrote this aphorism?



a. Phillis Wheatley

b. Benjamin Franklin

c. Abigail Adams

d. Thomas Jefferson

e. John Dickinson
The correct answer is B. This aphorism, or saying, is from one of the issues of Franklin's Poor Richard's Almanac. Almanacs were popular forms of entertainment in the 1700s. Franklin in the guise of Poor Richard dispensed useful weather and crop information for sailors and farmers, political satire, and homey wisdom. Answer A, Phillis Wheatley, a former slave, was the first African American poet published in the colonies. Answer C, Abigail Adams, wife of John Adams, the second president of the United States, has become famous for her correspondence with her husband while he was a delegate to the Continental Congress. Answer D, Thomas Jefferson, is a good distracter, since one of the things he is known for is writing—the Declaration of Independence. Answer E, John Dickinson, is also a good distracter, because he was the author of Letters from a Farmer in Pennsylvania.
Native Americans allied with the French in the French and Indian War because

a. the French paid better prices for Native American furs

b. the French were more interested in the fur trade than in colonization

c. the British refused to accept Native American aid

d. the French promised in any peace treaty to limit British settlements to east of the Appalachians

e. the British could not control the colonists who raided Native American villages
The correct answer is B. One of the main areas of conflict was the movement of more and more British colonists west of the Appalachians. The Native Americans sided with the French who held the territory as part of New France and were attempting to rout the settlers.
As a result of Pontiac's Rebellion,

a. the French and Indian began

b. the Native American alliance was crushed at the Battle of Fallen Timbers

c. the British issued the Proclamation of 1763

d. the French closed the border with the British colonies

e. the British government negotiated the Treaty of Greenville
The correct answer is C. Answer A is incorrect; the results of the French and Indian led to Pontiac's Rebellion. Answers B and E are related, but belong to the early national period of U.S. history. President George Washington sent troops against a Native American uprising in the Old Northwest. The Native Americans were beaten at Fallen Timbers and forced to agree to the Treaty of Greenville. Answer D is illogical; the French were gone by 1763.
The most popular subject for a painting among upper- and middle-class people in the colonial period was

a. a landscape

b. an historical theme

c. a portrait

d. everyday life

e. a religious subject
The correct answer is C. Like British aristocrats, wealthy landowners, merchants, and professionals as well as wealthy artisans wanted themselves and their family members captured for posterity. Paintings of landscapes and everyday life, answers A and D, became very popular during the romantic period in the 1800s. Answer B, historical themes, were popular so long as the subjects glorified the new nation. The popularity of religious themes, answer E, was somewhat limited.
Which of the following groups led attempts to abolish slavery before the American Revolution?

a. Roman Catholics

b. Puritans

c. white Southern artisans and craftworkers

d. Quakers

e. the New York legislature
The correct answer is D. Maryland, the colony most closely associated with Roman Catholics, answer A, was the second colony to make slavery a lifelong condition. Answer B is incorrect. Answer C is incorrect; there were few non-African American artisans and craftworkers in the South. Most of this kind of work was done by slaves. Answer E is incorrect; New York's legislature legalized slavery in the late 1600s.
Of the following colonies, which had the most diverse ethnic and religious population?

a. Massachusetts Bay

b. Pennsylvania

c. Virginia

d. Rhode Island

e. North Carolina
The correct answer is B. The Middle Colonies had a smaller English population and more nationalities with their diverse religions than either of the other two regions. Since Pennsylvania is the only Middle Colony listed, it's the answer. Even if you didn't know the answer, you could eliminate answers to get to it. Answers A and B were New England colonies whose strict religious practices and limiting of political rights to church members along with the climate and poor soil made them unattractive to other ethnic and religious groups. Based on numbers alone, you can rule out answers C and E They were Southern Colonies, whose large African populations meant that close to half the populations of both were homogeneous.
All of the following are characteristics of the British colonial system as applied to the North American colonies EXCEPT

a. English common law as the legal system

b. representative colonial assemblies

c. participation in the British trading network

d. centralized government for the colonies

e. oversight by the Board of Trade
The correct answer is D. The lack of a centralized government was a major reason why Parliament and the monarchy could not control the activities of the colonies. This lack of centralization may also be the reason for the framers choice of a federal system for the new United States.
The first attempt by colonists to unite for a political purpose was

a. Annapolis Convention

b. American System

c. Hartford Convention

d. Albany Plan of Union

e. Dominion of New England
The correct answer is D. Answer D was proposed by Benjamin Franklin and modeled after the Iroquois Confederacy. Although the delegates in Albany agreed, neither Parliament nor the colonial legislatures approved the plan. Answer A was the convention called to revise the Articles of Confederation, which instead called for a convention to write a new constitution. Answer B was Henry Clay's proposal for an economic system that would provide financial aid to each section of the new nation. Answer C was the assembly called by New England Federalists who opposed the War of 1812; its timing and its support of states' rights brought discredit to the Federalist Party. Answer E was the political unit created by Parliament in 1684 to govern all land north of Pennsylvania. With the Glorious Restoration in 1688, this was later reversed.
Which of the following is not a true statement about the Navigation Acts?



a. One purpose of the Navigation Acts was to ensure that raw materials were shipped only to England.

b. A second purpose of the Navigation Acts was to punish English sailors who jumped ship.

c. Under the Navigation Acts, all colonial exports and imports had to be shipped through England.

d. Initial colonial opposition to the Navigation Acts was light, because the acts were not rigidly enforced.

e. One purpose of the Navigation Acts was to cut out other nations from profitable trade with the colonies.
The correct answer is B. Don't confuse the Navigation Acts with the impressment of sailors that precipitated conflict with Great Britain in the early 1800s. In answer E, the major trade competitor at the time was the Netherlands.
Which of the following asserted Parliament's right to tax the colonies to raise revenue?

a. Stamp Act

b. Molasses Act

c. Sugar Act

d. Currency Act

e. Hat Act
The correct answer is C. The previous acts, answers B and E, had been meant to regulate trade. Answer A, Stamp Act, was the first internal tax, that is, on business within the colonies and not between the colonies and Great Britain or another nation. Answer D forbade the colonies to issue their own money.
Which of the following urged the establishment of Committees of Correspondence?

a. Sam Adams

b. Paul Revere

c. John Hancock

d. Patrick Henry

e. James Otis
The correct answer is A. Answer B, Paul Revere, was not only the spreader of the alarm "the British are coming," but also a participant in the Boston Tea Party and silversmith and engraver by trade. He designed the new nation's first currency. Answer C, John Hancock, was the first signer of the Declaration of Independence and president of the Second Continental Congress at the time. Answer D, Patrick Henry, is famous for the phrase "give me liberty or give me death," his support for independence, and his opposition to ratification of the U.S. Constitution. Answer E, James Otis, with Sam Adams wrote the Circular Letter that set out the colonists' opposition to the Townshend Acts.
What is the significance of the Intolerable Acts?



a. The Intolerable Acts closed the port of Boston and dismissed the Massachusetts legislature.

b. taken with the Quebec Act, the Intolerable Acts showed a lack of understanding of colonial interests.

c. The Intolerable Acts took away all civil rights from the citizens of Canada.

d. In passing the Intolerable Acts, Parliament shifted its focus from persuasion to punishment.

e. Parliament focused on specific issues rather than on the broader picture of colonial discontent.
The correct answer is D. Answer A is incorrect, because the questions asks for the significance of the Intolerable Acts. Answer A merely tells you some of the provisions of the acts. Answer B is incorrect, because the acts hit directly at colonial interests, for example, trade, self-government, and territorial expansion. Answer C is incorrect; the Quebec Act, which was passed at the same time as the Coercive Acts (the proper title for the Intolerable Acts), allowed the French citizens of Quebec to keep the French civil code of laws, which was more limited in the areas of rights than the English code. Answer E is incorrect, because underlying the provisions of the acts was the broader issue of self-government.
Which of the following is NOT a true statement about the Olive Branch Petition?

a. It was rejected by George III.

b. It was a concession to moderates in the Second Continental Congress.

c. It was followed by the colonists' Declaration of the Causes and Necessity of Taking Up Arms.

d. It was directed to the monarch and Parliament.

e. It was countered by George III's issuance of the Proclamation of Rebellion.
The correct answer is D. The Olive Branch Petition was addressed solely to George III and restated the colonists' loyalty to the king.
Which of the following is not a reason why colonists hesitated to declare independence in 1775?



a. How to attract foreign support was an unresolved issue.

b. Many could not see themselves as traitors, which is what a declaration of independence would mean.

c. How would Americans organize themselves since nations at the time had some form of monarchy.

d. Breaking political ties with England meant turning their back on their heritage.

e. Once unleashed, the fury of ordinary colonists could turn against the well to do and the rebellion could slide into anarchy.
The correct answer is A. Answer A is incorrect, because in 1775 the Second Continental Congress had sent representatives to France, Spain, and the Netherlands, Great Britain's traditional enemies, looking for military and economic aid. The wealthy, traditional, and more conservative colonial leaders had already seen the results of mob violence in the stoning of tax collectors, the destruction of property, and mass rallies and parades, so answer E was a real concern to them.
"These are the times that try men's souls. The summer soldier and the sunshine patriot will in this crisis, shrink from the service of his country; but he that stands it NOW, deserves the love and thanks of man and woman."
This quotation was most likely written by



a. George Washington

b. Thomas Jefferson

c. Thomas Paine

d. Hector St. John de Crevecoeur

e. James Fenimore Cooper
The correct answer is C. There is a clue in the second sentence, the word crisis. This quotation is from The Crisis by Thomas Paine. Answer A, George Washington, is not generally know for any writings calling his fellow colonists to rise up in rebellion. Answer B, Thomas Jefferson is just wrong; sometimes you can't always eliminate an answer based on elimination. Answer D, Hector St. John de Crevecoeur, wrote Letters from an American Farmer, published in 1782, a documentary of life in the colonies and new nation. Answer E, James Fenimore Cooper, was the first novelist in the early national period to use American themes in his works.
Loyalists were

a. concentrated in the South

b. mostly from the backcountry

c. the wealthier colonists

d. a mix of socioeconomic classes and geographic regions

e. mainly first-generation colonists
The correct answer is D. Answer B is illogical. Backcountry farmers would be the least interested in obeying the orders of a government several thousand miles away. Answers C and E are good distracters, because self-interest would make the wealthy inclined to want the status quo to continue, and first-generation English colonists would have closer ties to the home country. However, by 1776 first-generation colonists were more than likely to have some parental nationality other than English.
Which of the following was a major obstacle for the British army in fighting the colonists?

a. lack of supporters within the colonies

b. lack of Native American allies

c. support of a strong central government

d. length of its supply line

e. lack of well-trained soldiers
The correct answer is D. Although Great Britain was a far wealthier nation than the United States and had a population three times as large, its supply lines stretched 3,000 miles from North America to the British Isles. Resupplying the British army was a problem. Answer A is incorrect; it is estimated that at least a third of the Americans supported the British view. Answer B is also incorrect; the British had more Native American allies than did their former colonists. Answer C is also incorrect; the British had a strong central government in the monarchy and Parliament. Answer E is also incorrect; the British army made up of professional soldiers was one of the best fighting forces in the world in the late 1700s.
Under the Articles of Confederation the real power of the government lay

a. with the states

b. with the national courts

c. with the executive branch

d. with the people

e. with the army
The correct answer is A. Considering their experience as colonists under the strong central government of Great Britain, the delegates who drafted the Articles of Confederation kept power in the hands of the states. Answers B and C are incorrect, because there was no national judiciary system and the executive branch had very limited duties. Answers D and E are just wrong.
One of the reasons the Anti-Federalists lost in their attempt to block ratification of the Constitution was that they

a. opposed the addition of the Bill of Rights

b. continued to support the Articles of Confederation

c. lacked the resources to print their own pamphlets to counteract the Federalist Papers

d. tended to represent state and local interests and were not united on the national level

e. had no well-known leaders
The correct answer is D. Answer A is incorrect, because Anti-Federalist concerns about the power of the federal government over individuals spurred the addition of the Bill of Rights. Answer B is incorrect, because Anti-Federalists, too, believed the Articles were too weak to be effective. Answer C is incorrect. Richard Henry Lee, whose resolution calling for independence began the process that produced the Declaration of Independence, Sam Adams, and Patrick Henry were among the Anti-Federalists, so answer E is also wrong.
Hamilton exercised what type of power in establishing the First Bank?

a. reserved powers

b. implied powers

c. enumerated powers

d. expressed powers

e. inherent powers
The correct answer is B. Implied powers, answer B, are those derived from the "necessary and proper," or elastic, clause, by which the federal government has the powers required to carry out its functions. Answer A, reserved powers, refers to the powers that belong to the states as noted in the Tenth Amendment. The enumerated powers, answer C, are those listed in the Constitution as belonging to the federal government. These are stated in Article I, Section 8; Article II, Section 2; and the Sixteenth Amendment. Expressed powers, answer D, are the same as the enumerated powers. Answer E, inherent powers, refers to those powers that belong to the federal government because it is the federal government.
The most pressing economic need the new nation faced was

a. paying its war debts

b. developing a sound financial basis for the nation

c. settling the argument between New Englanders and Southerners over tariff rates

d. establishing a system to collect taxes

e. repealing the excise tax on whiskey
The correct answer is B. Answer A was part of putting the nation on a sound financial footing, so that it could pay its debts and grow. Answer C was necessary in order to pass a tariff law, but it was not the most pressing economic need. Enacting a tariff, like answer A, was one element in answer B. Answer D was a detail to achieve answer B. Answer E is illogical since it was passed as a way to raise revenue to run the government.
The important trade concession of the right of deposit at New Orleans was granted to the United States in

a. Jay's Treaty

b. the Open-Door Policy

c. the Rush-Bagot Convention

d. Pinckney's Treaty

e. Treaty of Paris
The correct answer is D. Answer D is also known as the Treaty of San Lorenzo. Answer A among other provisions settled the boundary with Canada west of the Great Lakes. Answer B was the term given to the policy of opening commercial trading in China to all foreign powers. In answer C, the United States and Great Britain agreed to a mutual disarmament in the Great Lakes. Answer E ended the Revolutionary War.
The root cause of the disagreement within Washington's Cabinet over the nation's response to the war between France and Great Britain was



a. American dislike of Great Britain and enthusiasm for the French Revolution

b. Hamilton's enthusiasm for the British and Jefferson's defense of the French Revolution

c. what, if any, role the new nation should play in world affairs

d. Citizen Genêt's disregard of the U.S. declaration of neutrality

e. disagreement over whether the alliance with France was still in effect
The correct answer is C. The question asks for the root cause, which means the basic cause. Answer A is incorrect, because the enthusiasm soon turned to horror with the excesses of the later revolution. Even Jefferson became disgusted, answer B. Answer D is too specific and too small an incident to be correct. Answer E is true in that there was disagreement between Federalists and Democratic-Republicans over whether the alliance was made with the French king or the French people, but this masked the real issue for the new nation, answer C.
The stated purpose of the Alien and Sedition Acts was to

a. deport French radicals

b. suppress radical foreign elements and their influence

c. prosecute Democratic-Republican journalists

d. scare radicals into leaving the country

e. destroy the Democratic-Republican opposition
The correct answer is B. The question asks you for the stated purpose. That's answer B. Answer C was the result of the enforcement of the acts, and answer E was what at least some of the Federalists hope would be the result of the acts. Answer D was an unforeseen result, and answer A was the purpose of one of the Alien Acts. There were four acts in all that are covered by the term "Alien and Sedition Acts." TIP: Always look for the key words in a question stem.
In his Farewell Address, George Washington spoke out against

a. war with Great Britain

b. honoring the alliance with France

c. the addition of new territory

d. joining alliances with foreign nations

e. making treaties with Native American nations
The correct answer is D. Washington's Farewell Address is noted frequently for his views on foreign policy (no entangling foreign alliances) and against parties based on sectional interests. You could have at least eliminated answer E since it was during Washington's administrations that the Treaty of Greenville was signed granting the United States rights to a large portion of the Old Northwest.
All of the following characterized American attitudes toward slavery between 1789 and 1800 EXCEPT



a. Slave owners' property rights were considered more important than freedom for African American slaves.

b. Slave owners were frightened by the success of the slave revolt in Santo Domingo.

c. Slave owners felt under attack by abolitionists.

d. African Americans had their hopes raised by the promise of the American Revolution, but were badly disappointed by the Constitution.

e. The question of what to do if slaves were freed seemed insurmountable.
The correct answer is C. Did you miss the dates in the question? Abolitionists didn't organize until the 1830s as part of the general movement toward social reform. TIP: Always check the key words in a question.
Marbury v. Madison established the principle of

a. right to privacy

b. one man, one vote

c. affirmative action

d. judicial review

e. strict construction
The correct answer is D. This is the "midnight judges" case in which John Adams signed judicial commissions for certain Federalists, which the new Jefferson administration refused to honor. Chief Justice John Marshall found for the plaintiff and in so doing established the right of the Supreme Court to determine the constitutionality of acts of Congress and the Executive. Answer A was established by Roe v. Wade. Answer B is the principle established by a series of voting cases in the latter half of the twentieth century. Answer C is a policy to provide equal access for women and minorities to jobs and schools from which they have been historically excluded. The relevant case is Regents of the University of California v. Bakke. Answer E is a philosophy of approaching court decisions. Strict constructionists, also known as judicial conservatives, believe that the Constitution should be interpreted as the framers wrote. On the other hand, loose constructionists, or judicial activists, believe the Constitution needs to be interpreted in light of modern life; the framers could not know or imagine how the nation has changed since 1789.
The War Hawks wanted war against

a. Great Britain

b. France

c. Mexico

d. Spain

e. Haiti
The correct answer is A. The War Hawks were members of Congress mainly from the western states that wanted the United States to declare war against Great Britain in the early 1800s. Because of the impressment of U.S. sailors by the British and British attacks on U.S. shipping, many Americans wanted to retaliate against Great Britain. However, the War Hawks also wanted to use the war as an excuse to invade Canada and seize it from the British. They saw Canada as a vast new area for U.S. settlement. The War Hawks prevailed and the United States declared war against the British in 1812, the War of 1812. Answer B is incorrect. Although the French also ignored U.S. rights on the high seas, the War Hawks didn't want to go to war against France. Mexico, answer C, was the object of U.S. war frenzy in 1846, the Mexican War. Answer D was involved in the Spanish American War in 1898. Answer E is a distractor.
The Embargo Act of 1807 resulted in all of the following EXCEPT

a. economic depression in the United States

b. closure of U.S. ports to all foreign ships

c. restriction of U.S. trade to the United States

d. economic depression in Great Britain and France

e. failure of the Federalists to win the election of 1808
The correct answer is D. The goal of Thomas Jefferson's embargo in 1807 was to hurt the economies of both Great Britain and France and thereby force them to recognize U.S. sovereignty on the high seas. However, it was the economy of the United States that was damaged by the embargo, answer A. Answers B and C were provisions of the law. Answer E was the unintended result of the Embargo Act.
President James Madison was duped by Napoleon into

a. dropping trade sanctions against France

b. imposing trade sanctions against Great Britain

c. declaring war on Great Britain

d. backing a takeover of West Florida from Spain, Napoleon's enemy

e. attacking Native American allies of the British on the western frontier
The correct answer is B. Macon's Bill No. 2 reopened U.S. trade with both Great Britain and France, but gave the president the authority if either nation agreed to stop seizing U.S. merchant ships to impose an embargo against the other nation. Napoleon tricked Madison into imposing the sanctions against Great Britain. He agreed to allow U.S. ships to sail freely, but never intended to honor the agreement. Answer A is incorrect, because Macon's Bill No. 2 had eliminated trade sanctions. Answer C is incorrect; the War of 1812 was two years away. Answer D is true in that Madison secretly supported the takeover of Spanish West Florida and Spain was an enemy of Napoleon, but Madison's support had to do with American territorial interests, not with Napoleon. Answer E is true, but had nothing to do with Napoleon; the attacks on the Native Americans again was caused by American interest in their lands.

Return to Section Report Question: 48 of 120
Jump to question: 1 (answered) 2 (answered) 3 (answered) 4 (answered) 5 (answered) 6 (answered) 7 (answered) 8 (answered) 9 (answered) 10 (answered) 11 (answered) 12 (answered) 13 (answered) 14 (answered) 15 (answered) 16 (answered) 17 (answered) 18 (answered) 19 (answered) 20 (answered) 21 (answered) 22 (answered) 23 (answered) 24 (answered) 25 (answered) 26 (answered) 27 (answered) 28 (answered) 29 (answered) 30 (answered) 31 (answered) 32 (answered) 33 (answered) 34 (answered) 35 (answered) 36 (answered) 37 (answered) 38 (answered) 39 (answered) 40 (answered) 41 (answered) 42 (answered) 43 (answered) 44 (answered) 45 (answered) 46 (answered) 47 (answered) 48 (answered) 49 (answered) 50 (answered) 51 (answered) 52 (answered) 53 (answered) 54 (answered) 55 (answered) 56 (answered) 57 (answered) 58 (answered) 59 (answered) 60 (answered) 61 (answered) 62 (answered) 63 (answered) 64 (answered) 65 (answered) 66 (answered) 67 (answered) 68 (answered) 69 (answered) 70 (answered) 71 (answered) 72 (answered) 73 (answered) 74 (answered) 75 (answered) 76 (answered) 77 (answered) 78 (answered) 79 (answered) 80 (answered) 81 (answered) 82 (answered) 83 (answered) 84 (answered) 85 (answered) 86 (answered) 87 (answered) 88 (answered) 89 (answered) 90 (answered) 91 (answered) 92 (answered) 93 (answered) 94 (answered) 95 (answered) 96 (answered) 97 (answered) 98 (answered) 99 (answered)100 (answered)101 (answered)102 (answered)103 (answered)104 (answered)105 (answered)106 (answered)107 (answered)108 (answered)109 (answered)110 (answered)111 (answered)112 (answered)113 (answered)114 (answered)115 (answered)116 (answered)117 (answered)118 (answered)119 (answered)120 (answered)


Which of the following dealt a crushing blow to Native American resistance in the Southeast?

a. Battle of Fallen Timbers

b. Battle of Tippecanoe

c. Battle of Horseshoe Bend

d. Sand Creek Massacre

e. Battle of Wounded Knee



The correct answer is C. Led by Andrew Jackson, a coalition of Tennessee militia and Choctaw defeated the Creek in 1814, ending major Native American resistance in the Southeast. Answer A refers to the defeat of a coalition of Native Americans in the Old Northwest in 1794. Answer B is the defeat of a coalition of Native Americans in the Indiana Territory in 1811. Answer D was an attack by Colorado militia led by Colonel John Chivington against a band of 500 Cheyenne who had surrendered. Answer E was the last battle in the Plains wars.

Return to Section Report Question: 49 of 120
Jump to question: 1 (answered) 2 (answered) 3 (answered) 4 (answered) 5 (answered) 6 (answered) 7 (answered) 8 (answered) 9 (answered) 10 (answered) 11 (answered) 12 (answered) 13 (answered) 14 (answered) 15 (answered) 16 (answered) 17 (answered) 18 (answered) 19 (answered) 20 (answered) 21 (answered) 22 (answered) 23 (answered) 24 (answered) 25 (answered) 26 (answered) 27 (answered) 28 (answered) 29 (answered) 30 (answered) 31 (answered) 32 (answered) 33 (answered) 34 (answered) 35 (answered) 36 (answered) 37 (answered) 38 (answered) 39 (answered) 40 (answered) 41 (answered) 42 (answered) 43 (answered) 44 (answered) 45 (answered) 46 (answered) 47 (answered) 48 (answered) 49 (answered) 50 (answered) 51 (answered) 52 (answered) 53 (answered) 54 (answered) 55 (answered) 56 (answered) 57 (answered) 58 (answered) 59 (answered) 60 (answered) 61 (answered) 62 (answered) 63 (answered) 64 (answered) 65 (answered) 66 (answered) 67 (answered) 68 (answered) 69 (answered) 70 (answered) 71 (answered) 72 (answered) 73 (answered) 74 (answered) 75 (answered) 76 (answered) 77 (answered) 78 (answered) 79 (answered) 80 (answered) 81 (answered) 82 (answered) 83 (answered) 84 (answered) 85 (answered) 86 (answered) 87 (answered) 88 (answered) 89 (answered) 90 (answered) 91 (answered) 92 (answered) 93 (answered) 94 (answered) 95 (answered) 96 (answered) 97 (answered) 98 (answered) 99 (answered)100 (answered)101 (answered)102 (answered)103 (answered)104 (answered)105 (answered)106 (answered)107 (answered)108 (answered)109 (answered)110 (answered)111 (answered)112 (answered)113 (answered)114 (answered)115 (answered)116 (answered)117 (answered)118 (answered)119 (answered)120 (answered)


Opposition to the American System came primarily from

a. the Southeast and New England

b. New England and the western states

c. the western states and the Southeast

d. New England and the Mid-Atlantic states

e. the Mid-Atlantic states and the western states



The correct answer is A. Many in the Southeast considered the internal improvements proposal an infringement on states' rights. There was an element of sectional rivalry in the opposition of New Englanders. They didn't want to lose their influence in national politics to the new western states. However, they objected on the practical grounds that they had banks and roads. Answers B and C are half wrong, because the western states is incorrect. Answer D is also half wrong, because Mid-Atlantic states is incorrect. TIP: In two-part answers like this, you can save yourself some time by reading the first part of each answer and eliminating those that are wrong. Then go back and read the second part of just the answers whose first half is correct. You could have eliminated answers C and E this way. Three seconds here, four second there, and you can gain a minute before you know it.

Return to Section Report Question: 50 of 120
Jump to question: 1 (answered) 2 (answered) 3 (answered) 4 (answered) 5 (answered) 6 (answered) 7 (answered) 8 (answered) 9 (answered) 10 (answered) 11 (answered) 12 (answered) 13 (answered) 14 (answered) 15 (answered) 16 (answered) 17 (answered) 18 (answered) 19 (answered) 20 (answered) 21 (answered) 22 (answered) 23 (answered) 24 (answered) 25 (answered) 26 (answered) 27 (answered) 28 (answered) 29 (answered) 30 (answered) 31 (answered) 32 (answered) 33 (answered) 34 (answered) 35 (answered) 36 (answered) 37 (answered) 38 (answered) 39 (answered) 40 (answered) 41 (answered) 42 (answered) 43 (answered) 44 (answered) 45 (answered) 46 (answered) 47 (answered) 48 (answered) 49 (answered) 50 (answered) 51 (answered) 52 (answered) 53 (answered) 54 (answered) 55 (answered) 56 (answered) 57 (answered) 58 (answered) 59 (answered) 60 (answered) 61 (answered) 62 (answered) 63 (answered) 64 (answered) 65 (answered) 66 (answered) 67 (answered) 68 (answered) 69 (answered) 70 (answered) 71 (answered) 72 (answered) 73 (answered) 74 (answered) 75 (answered) 76 (answered) 77 (answered) 78 (answered) 79 (answered) 80 (answered) 81 (answered) 82 (answered) 83 (answered) 84 (answered) 85 (answered) 86 (answered) 87 (answered) 88 (answered) 89 (answered) 90 (answered) 91 (answered) 92 (answered) 93 (answered) 94 (answered) 95 (answered) 96 (answered) 97 (answered) 98 (answered) 99 (answered)100 (answered)101 (answered)102 (answered)103 (answered)104 (answered)105 (answered)106 (answered)107 (answered)108 (answered)109 (answered)110 (answered)111 (answered)112 (answered)113 (answered)114 (answered)115 (answered)116 (answered)117 (answered)118 (answered)119 (answered)120 (answered)


The Tariff of 1816 was passed

a. in reaction to the inability of income taxes to raise enough money to run the government

b. to make up for revenue lost during the War of 1812

c. in reaction to the dumping of cheap British goods into the United States after the War of 1812

d. as an admission of the failure of the U.S. economy to grow

e. as a stopgap measure when the First Bank's charter ran out



The correct answer is C. The Tariff of 1816 is significant because it was the nation's first protective tariff. The modest tariff schedule passed in 1789 was to raise revenue. Answer A is incorrect. The right to tax income didn't become law until the Sixteenth Amendment was passed in 1913. Answer B is just incorrect. Answer D is incorrect; the tariff was meant to protect fledgling industries that sprang up during the war, but the economy was growing. Answer E is incorrect and illogical. The tariff had nothing to do with the banking system.

Return to Section Report Question: 51 of 120
Jump to question: 1 (answered) 2 (answered) 3 (answered) 4 (answered) 5 (answered) 6 (answered) 7 (answered) 8 (answered) 9 (answered) 10 (answered) 11 (answered) 12 (answered) 13 (answered) 14 (answered) 15 (answered) 16 (answered) 17 (answered) 18 (answered) 19 (answered) 20 (answered) 21 (answered) 22 (answered) 23 (answered) 24 (answered) 25 (answered) 26 (answered) 27 (answered) 28 (answered) 29 (answered) 30 (answered) 31 (answered) 32 (answered) 33 (answered) 34 (answered) 35 (answered) 36 (answered) 37 (answered) 38 (answered) 39 (answered) 40 (answered) 41 (answered) 42 (answered) 43 (answered) 44 (answered) 45 (answered) 46 (answered) 47 (answered) 48 (answered) 49 (answered) 50 (answered) 51 (answered) 52 (answered) 53 (answered) 54 (answered) 55 (answered) 56 (answered) 57 (answered) 58 (answered) 59 (answered) 60 (answered) 61 (answered) 62 (answered) 63 (answered) 64 (answered) 65 (answered) 66 (answered) 67 (answered) 68 (answered) 69 (answered) 70 (answered) 71 (answered) 72 (answered) 73 (answered) 74 (answered) 75 (answered) 76 (answered) 77 (answered) 78 (answered) 79 (answered) 80 (answered) 81 (answered) 82 (answered) 83 (answered) 84 (answered) 85 (answered) 86 (answered) 87 (answered) 88 (answered) 89 (answered) 90 (answered) 91 (answered) 92 (answered) 93 (answered) 94 (answered) 95 (answered) 96 (answered) 97 (answered) 98 (answered) 99 (answered)100 (answered)101 (answered)102 (answered)103 (answered)104 (answered)105 (answered)106 (answered)107 (answered)108 (answered)109 (answered)110 (answered)111 (answered)112 (answered)113 (answered)114 (answered)115 (answered)116 (answered)117 (answered)118 (answered)119 (answered)120 (answered)


Which of the following presidents enjoyed a period with little partisan politics?

a. George Washington

b. John Adams

c. Thomas Jefferson

d. James Monroe

e. Andrew Jackson



The correct answer is D. President James Monroe's administration is known as the "Era of Good Feelings" for its political harmony. Partisan politics began in George Washington's first term, answer A, as a result of the differing philosophies of Alexander Hamilton and Thomas Jefferson and continued through until the end of Madison's second term. Answer E, Andrew Jackson, had two highly partisan terms in office.

Return to Section Report Question: 52 of 120
Jump to question: 1 (answered) 2 (answered) 3 (answered) 4 (answered) 5 (answered) 6 (answered) 7 (answered) 8 (answered) 9 (answered) 10 (answered) 11 (answered) 12 (answered) 13 (answered) 14 (answered) 15 (answered) 16 (answered) 17 (answered) 18 (answered) 19 (answered) 20 (answered) 21 (answered) 22 (answered) 23 (answered) 24 (answered) 25 (answered) 26 (answered) 27 (answered) 28 (answered) 29 (answered) 30 (answered) 31 (answered) 32 (answered) 33 (answered) 34 (answered) 35 (answered) 36 (answered) 37 (answered) 38 (answered) 39 (answered) 40 (answered) 41 (answered) 42 (answered) 43 (answered) 44 (answered) 45 (answered) 46 (answered) 47 (answered) 48 (answered) 49 (answered) 50 (answered) 51 (answered) 52 (answered) 53 (answered) 54 (answered) 55 (answered) 56 (answered) 57 (answered) 58 (answered) 59 (answered) 60 (answered) 61 (answered) 62 (answered) 63 (answered) 64 (answered) 65 (answered) 66 (answered) 67 (answered) 68 (answered) 69 (answered) 70 (answered) 71 (answered) 72 (answered) 73 (answered) 74 (answered) 75 (answered) 76 (answered) 77 (answered) 78 (answered) 79 (answered) 80 (answered) 81 (answered) 82 (answered) 83 (answered) 84 (answered) 85 (answered) 86 (answered) 87 (answered) 88 (answered) 89 (answered) 90 (answered) 91 (answered) 92 (answered) 93 (answered) 94 (answered) 95 (answered) 96 (answered) 97 (answered) 98 (answered) 99 (answered)100 (answered)101 (answered)102 (answered)103 (answered)104 (answered)105 (answered)106 (answered)107 (answered)108 (answered)109 (answered)110 (answered)111 (answered)112 (answered)113 (answered)114 (answered)115 (answered)116 (answered)117 (answered)118 (answered)119 (answered)120 (answered)


Slavery moved from being a sectional issue to a national one

a. when the first territory created from the Louisiana Territory requested statehood

b. when the cotton gin spurred a boom in cotton prices

c. when the slave trade was banned in accordance with the Constitution

d. when the abolitionist movement was founded

e. when newspapers became cheap and easily available



The correct answer is A. Until 1819, slavery was seen as a sectional issue for the South to deal with. Once Missouri requested statehood, the political, economic, and moral divisions between North and South became apparent. Answer C occurred in 1808 with relatively no disturbance to the South or the nation. Answer D is about a decade too soon; Missouri, the first new state made from the Louisiana Territory, requested statehood in 1819. The widespread circulation of newspapers helped publicize the argument over slavery, but did not create it.

Return to Section Report Question: 53 of 120
Jump to question: 1 (answered) 2 (answered) 3 (answered) 4 (answered) 5 (answered) 6 (answered) 7 (answered) 8 (answered) 9 (answered) 10 (answered) 11 (answered) 12 (answered) 13 (answered) 14 (answered) 15 (answered) 16 (answered) 17 (answered) 18 (answered) 19 (answered) 20 (answered) 21 (answered) 22 (answered) 23 (answered) 24 (answered) 25 (answered) 26 (answered) 27 (answered) 28 (answered) 29 (answered) 30 (answered) 31 (answered) 32 (answered) 33 (answered) 34 (answered) 35 (answered) 36 (answered) 37 (answered) 38 (answered) 39 (answered) 40 (answered) 41 (answered) 42 (answered) 43 (answered) 44 (answered) 45 (answered) 46 (answered) 47 (answered) 48 (answered) 49 (answered) 50 (answered) 51 (answered) 52 (answered) 53 (answered) 54 (answered) 55 (answered) 56 (answered) 57 (answered) 58 (answered) 59 (answered) 60 (answered) 61 (answered) 62 (answered) 63 (answered) 64 (answered) 65 (answered) 66 (answered) 67 (answered) 68 (answered) 69 (answered) 70 (answered) 71 (answered) 72 (answered) 73 (answered) 74 (answered) 75 (answered) 76 (answered) 77 (answered) 78 (answered) 79 (answered) 80 (answered) 81 (answered) 82 (answered) 83 (answered) 84 (answered) 85 (answered) 86 (answered) 87 (answered) 88 (answered) 89 (answered) 90 (answered) 91 (answered) 92 (answered) 93 (answered) 94 (answered) 95 (answered) 96 (answered) 97 (answered) 98 (answered) 99 (answered)100 (answered)101 (answered)102 (answered)103 (answered)104 (answered)105 (answered)106 (answered)107 (answered)108 (answered)109 (answered)110 (answered)111 (answered)112 (answered)113 (answered)114 (answered)115 (answered)116 (answered)117 (answered)118 (answered)119 (answered)120 (answered)


The development of the factory system had all of the following effects EXCEPT

a. the relationship among family members remained stable

b. working-class women took in piece work to add to the family income

c. the economic usefulness of the family-based workshop was destroyed

d. woman's role once divorced from earning income became less important in the family

e. many of the necessities women made were now available for purchase



The correct answer is A. The first thing that changed with the introduction of the factory system was the relationship of the father to the other members. Up to this point, work had been done in a family-run workshop in which the father was the artisan and the other members assisted him. When men and sons began to go out to work, this patriarchal influence changed. Answer B is true for a period of time and was also true for lower-middle class because piecework allowed the wife to stay home, which was deemed her proper place.

Return to Section Report Question: 54 of 120
Jump to question: 1 (answered) 2 (answered) 3 (answered) 4 (answered) 5 (answered) 6 (answered) 7 (answered) 8 (answered) 9 (answered) 10 (answered) 11 (answered) 12 (answered) 13 (answered) 14 (answered) 15 (answered) 16 (answered) 17 (answered) 18 (answered) 19 (answered) 20 (answered) 21 (answered) 22 (answered) 23 (answered) 24 (answered) 25 (answered) 26 (answered) 27 (answered) 28 (answered) 29 (answered) 30 (answered) 31 (answered) 32 (answered) 33 (answered) 34 (answered) 35 (answered) 36 (answered) 37 (answered) 38 (answered) 39 (answered) 40 (answered) 41 (answered) 42 (answered) 43 (answered) 44 (answered) 45 (answered) 46 (answered) 47 (answered) 48 (answered) 49 (answered) 50 (answered) 51 (answered) 52 (answered) 53 (answered) 54 (answered) 55 (answered) 56 (answered) 57 (answered) 58 (answered) 59 (answered) 60 (answered) 61 (answered) 62 (answered) 63 (answered) 64 (answered) 65 (answered) 66 (answered) 67 (answered) 68 (answered) 69 (answered) 70 (answered) 71 (answered) 72 (answered) 73 (answered) 74 (answered) 75 (answered) 76 (answered) 77 (answered) 78 (answered) 79 (answered) 80 (answered) 81 (answered) 82 (answered) 83 (answered) 84 (answered) 85 (answered) 86 (answered) 87 (answered) 88 (answered) 89 (answered) 90 (answered) 91 (answered) 92 (answered) 93 (answered) 94 (answered) 95 (answered) 96 (answered) 97 (answered) 98 (answered) 99 (answered)100 (answered)101 (answered)102 (answered)103 (answered)104 (answered)105 (answered)106 (answered)107 (answered)108 (answered)109 (answered)110 (answered)111 (answered)112 (answered)113 (answered)114 (answered)115 (answered)116 (answered)117 (answered)118 (answered)119 (answered)120 (answered)


The United States indirectly helped which European nation when it issued the Monroe Doctrine?

a. Spain

b. Portugal

c. the Netherlands

d. Great Britain

e. France



The correct answer is D. Originally, Great Britain had approached the United States with a proposal to issue a joint declaration of noninterference in the Western Hemisphere. At John Quincy Adams' urging, President James Monroe declined. Adams was Monroe's shrewd secretary of state. Instead he urged Monroe to issue his own policy statement. However, in so doing, the United States also aided Great Britain which looked to the former Spanish colonies as new markets and wanted to ensure that neither Spain nor any other European country tried to gain possession of the new nations. The importance of Portugal, answer B, and the Netherlands, answer C, had declined greatly since the 1500s and the race to gain trade advantages had begun.

Return to Section Report Question: 55 of 120
Jump to question: 1 (answered) 2 (answered) 3 (answered) 4 (answered) 5 (answered) 6 (answered) 7 (answered) 8 (answered) 9 (answered) 10 (answered) 11 (answered) 12 (answered) 13 (answered) 14 (answered) 15 (answered) 16 (answered) 17 (answered) 18 (answered) 19 (answered) 20 (answered) 21 (answered) 22 (answered) 23 (answered) 24 (answered) 25 (answered) 26 (answered) 27 (answered) 28 (answered) 29 (answered) 30 (answered) 31 (answered) 32 (answered) 33 (answered) 34 (answered) 35 (answered) 36 (answered) 37 (answered) 38 (answered) 39 (answered) 40 (answered) 41 (answered) 42 (answered) 43 (answered) 44 (answered) 45 (answered) 46 (answered) 47 (answered) 48 (answered) 49 (answered) 50 (answered) 51 (answered) 52 (answered) 53 (answered) 54 (answered) 55 (answered) 56 (answered) 57 (answered) 58 (answered) 59 (answered) 60 (answered) 61 (answered) 62 (answered) 63 (answered) 64 (answered) 65 (answered) 66 (answered) 67 (answered) 68 (answered) 69 (answered) 70 (answered) 71 (answered) 72 (answered) 73 (answered) 74 (answered) 75 (answered) 76 (answered) 77 (answered) 78 (answered) 79 (answered) 80 (answered) 81 (answered) 82 (answered) 83 (answered) 84 (answered) 85 (answered) 86 (answered) 87 (answered) 88 (answered) 89 (answered) 90 (answered) 91 (answered) 92 (answered) 93 (answered) 94 (answered) 95 (answered) 96 (answered) 97 (answered) 98 (answered) 99 (answered)100 (answered)101 (answered)102 (answered)103 (answered)104 (answered)105 (answered)106 (answered)107 (answered)108 (answered)109 (answered)110 (answered)111 (answered)112 (answered)113 (answered)114 (answered)115 (answered)116 (answered)117 (answered)118 (answered)119 (answered)120 (answered)


Prior to the 1800s, which of the following social classes did not exist?

a. independent farmers and planters

b. planters and industrialists

c. factory workers and industrialists

d. professionals and industrialists

e. slaves and professionals



The correct answer is C. Independent farmers were among the earliest colonists and planters existed by the mid-1660s, so answer A can't be correct. This makes answer B incorrect also. Professionals, answer D, mean doctors, lawyers, and merchants—not shopkeepers—and they, too, were represented early in colonial history. Answer E is incorrect; slavery was enforced by the 1660s. TIP: In two-part questions, remember to go through the answers and read just the first half of each answer, eliminate the wrong answers, and return to the answers where the first part at least is correct. In this case, you'd only have to go back to answer C. This saves you time.

Return to Section Report Question: 56 of 120
Jump to question: 1 (answered) 2 (answered) 3 (answered) 4 (answered) 5 (answered) 6 (answered) 7 (answered) 8 (answered) 9 (answered) 10 (answered) 11 (answered) 12 (answered) 13 (answered) 14 (answered) 15 (answered) 16 (answered) 17 (answered) 18 (answered) 19 (answered) 20 (answered) 21 (answered) 22 (answered) 23 (answered) 24 (answered) 25 (answered) 26 (answered) 27 (answered) 28 (answered) 29 (answered) 30 (answered) 31 (answered) 32 (answered) 33 (answered) 34 (answered) 35 (answered) 36 (answered) 37 (answered) 38 (answered) 39 (answered) 40 (answered) 41 (answered) 42 (answered) 43 (answered) 44 (answered) 45 (answered) 46 (answered) 47 (answered) 48 (answered) 49 (answered) 50 (answered) 51 (answered) 52 (answered) 53 (answered) 54 (answered) 55 (answered) 56 (answered) 57 (answered) 58 (answered) 59 (answered) 60 (answered) 61 (answered) 62 (answered) 63 (answered) 64 (answered) 65 (answered) 66 (answered) 67 (answered) 68 (answered) 69 (answered) 70 (answered) 71 (answered) 72 (answered) 73 (answered) 74 (answered) 75 (answered) 76 (answered) 77 (answered) 78 (answered) 79 (answered) 80 (answered) 81 (answered) 82 (answered) 83 (answered) 84 (answered) 85 (answered) 86 (answered) 87 (answered) 88 (answered) 89 (answered) 90 (answered) 91 (answered) 92 (answered) 93 (answered) 94 (answered) 95 (answered) 96 (answered) 97 (answered) 98 (answered) 99 (answered)100 (answered)101 (answered)102 (answered)103 (answered)104 (answered)105 (answered)106 (answered)107 (answered)108 (answered)109 (answered)110 (answered)111 (answered)112 (answered)113 (answered)114 (answered)115 (answered)116 (answered)117 (answered)118 (answered)119 (answered)120 (answered)


A major victory for early labor unions was winning

a. a ban on the use of immigrants as strikebreakers

b. an eight-hour workday

c. a ban on the use of child labor

d. elimination of the closed shop

e. elimination of imprisonment for debt



The correct answer is E. Imprisonment for debt was a very grave problem for workers. It is estimated that 75,000 people a year were imprisoned for unpaid debts. You would be reasonable to assume that employers would fight hard to keep a strikebreaking tactic like answer A, so that would be easy to eliminate. Answers B and C didn't occur for half a century. Answer D is illogical; a union would want to keep a closed shop. This is one in which all employees must belong to the union.

Return to Section Report Question: 57 of 120
Jump to question: 1 (answered) 2 (answered) 3 (answered) 4 (answered) 5 (answered) 6 (answered) 7 (answered) 8 (answered) 9 (answered) 10 (answered) 11 (answered) 12 (answered) 13 (answered) 14 (answered) 15 (answered) 16 (answered) 17 (answered) 18 (answered) 19 (answered) 20 (answered) 21 (answered) 22 (answered) 23 (answered) 24 (answered) 25 (answered) 26 (answered) 27 (answered) 28 (answered) 29 (answered) 30 (answered) 31 (answered) 32 (answered) 33 (answered) 34 (answered) 35 (answered) 36 (answered) 37 (answered) 38 (answered) 39 (answered) 40 (answered) 41 (answered) 42 (answered) 43 (answered) 44 (answered) 45 (answered) 46 (answered) 47 (answered) 48 (answered) 49 (answered) 50 (answered) 51 (answered) 52 (answered) 53 (answered) 54 (answered) 55 (answered) 56 (answered) 57 (answered) 58 (answered) 59 (answered) 60 (answered) 61 (answered) 62 (answered) 63 (answered) 64 (answered) 65 (answered) 66 (answered) 67 (answered) 68 (answered) 69 (answered) 70 (answered) 71 (answered) 72 (answered) 73 (answered) 74 (answered) 75 (answered) 76 (answered) 77 (answered) 78 (answered) 79 (answered) 80 (answered) 81 (answered) 82 (answered) 83 (answered) 84 (answered) 85 (answered) 86 (answered) 87 (answered) 88 (answered) 89 (answered) 90 (answered) 91 (answered) 92 (answered) 93 (answered) 94 (answered) 95 (answered) 96 (answered) 97 (answered) 98 (answered) 99 (answered)100 (answered)101 (answered)102 (answered)103 (answered)104 (answered)105 (answered)106 (answered)107 (answered)108 (answered)109 (answered)110 (answered)111 (answered)112 (answered)113 (answered)114 (answered)115 (answered)116 (answered)117 (answered)118 (answered)119 (answered)120 (answered)


The election of 1824 was decided in the House of Representatives because



a. Andrew Jackson had a majority of electoral votes but not a majority of popular votes

b. no candidate had a majority of electoral votes

c. John Quincy Adams had a majority of the popular vote, but not a majority of electoral votes

d. Adams had a majority of electoral votes, but not a majority of the popular vote

e. having more than two candidates makes it impossible for anyone to win a majority of popular and electoral votes



The correct answer is B. Who had a majority of the popular votes (it was Jackson) doesn't matter. All that matters in a presidential election is who has the majority of electoral votes. Since neither Adams, Jackson, nor the other two candidates running had won a majority, the election was sent to the House as required by the Twelfth Amendment. Answer E may or may not be true, but is also irrelevant. All that matters is that one candidate has a majority of electoral votes. There have been any number of elections in which more than two parties have had presidential candidates and one candidate has garnered a majority of electoral votes, for example, 1968, 1980, and 1992.

Return to Section Report Question: 58 of 120
Jump to question: 1 (answered) 2 (answered) 3 (answered) 4 (answered) 5 (answered) 6 (answered) 7 (answered) 8 (answered) 9 (answered) 10 (answered) 11 (answered) 12 (answered) 13 (answered) 14 (answered) 15 (answered) 16 (answered) 17 (answered) 18 (answered) 19 (answered) 20 (answered) 21 (answered) 22 (answered) 23 (answered) 24 (answered) 25 (answered) 26 (answered) 27 (answered) 28 (answered) 29 (answered) 30 (answered) 31 (answered) 32 (answered) 33 (answered) 34 (answered) 35 (answered) 36 (answered) 37 (answered) 38 (answered) 39 (answered) 40 (answered) 41 (answered) 42 (answered) 43 (answered) 44 (answered) 45 (answered) 46 (answered) 47 (answered) 48 (answered) 49 (answered) 50 (answered) 51 (answered) 52 (answered) 53 (answered) 54 (answered) 55 (answered) 56 (answered) 57 (answered) 58 (answered) 59 (answered) 60 (answered) 61 (answered) 62 (answered) 63 (answered) 64 (answered) 65 (answered) 66 (answered) 67 (answered) 68 (answered) 69 (answered) 70 (answered) 71 (answered) 72 (answered) 73 (answered) 74 (answered) 75 (answered) 76 (answered) 77 (answered) 78 (answered) 79 (answered) 80 (answered) 81 (answered) 82 (answered) 83 (answered) 84 (answered) 85 (answered) 86 (answered) 87 (answered) 88 (answered) 89 (answered) 90 (answered) 91 (answered) 92 (answered) 93 (answered) 94 (answered) 95 (answered) 96 (answered) 97 (answered) 98 (answered) 99 (answered)100 (answered)101 (answered)102 (answered)103 (answered)104 (answered)105 (answered)106 (answered)107 (answered)108 (answered)109 (answered)110 (answered)111 (answered)112 (answered)113 (answered)114 (answered)115 (answered)116 (answered)117 (answered)118 (answered)119 (answered)120 (answered)


Andrew Jackson called it rotation, whereas his detractors considered it

a. restructuring government

b. the spoil system

c. the Kitchen Cabinet

d. civil service reform

e. democracy in action



The correct answer is B. Or as the saying goes "to the victor, belongs the spoils," that is, the rewards. Rotation was how Jackson described removing incumbent civil servants in the federal government and replacing them with his supporters. According to him, incumbents who were in office a long time tended to place the workings of the bureaucracy above the people they were to serve. You could eliminate some answers if you didn't know the right answer. The question uses the word detractors, so you know you should be looking for a negative answer. Answer A might be negative if the person thought the government was fine the way it was, so hold off deciding on this answer. Answer D and answer E seem like positive things, so eliminate those two answers. Answer C was the name given to Jackson's informal group of advisors, as opposed to the formal Cabinet. That doesn't seem like a likely answer. Even if you have to guess between answers A and B you have a 50-50 chance of being correct.

Return to Section Report Question: 59 of 120
Jump to question: 1 (answered) 2 (answered) 3 (answered) 4 (answered) 5 (answered) 6 (answered) 7 (answered) 8 (answered) 9 (answered) 10 (answered) 11 (answered) 12 (answered) 13 (answered) 14 (answered) 15 (answered) 16 (answered) 17 (answered) 18 (answered) 19 (answered) 20 (answered) 21 (answered) 22 (answered) 23 (answered) 24 (answered) 25 (answered) 26 (answered) 27 (answered) 28 (answered) 29 (answered) 30 (answered) 31 (answered) 32 (answered) 33 (answered) 34 (answered) 35 (answered) 36 (answered) 37 (answered) 38 (answered) 39 (answered) 40 (answered) 41 (answered) 42 (answered) 43 (answered) 44 (answered) 45 (answered) 46 (answered) 47 (answered) 48 (answered) 49 (answered) 50 (answered) 51 (answered) 52 (answered) 53 (answered) 54 (answered) 55 (answered) 56 (answered) 57 (answered) 58 (answered) 59 (answered) 60 (answered) 61 (answered) 62 (answered) 63 (answered) 64 (answered) 65 (answered) 66 (answered) 67 (answered) 68 (answered) 69 (answered) 70 (answered) 71 (answered) 72 (answered) 73 (answered) 74 (answered) 75 (answered) 76 (answered) 77 (answered) 78 (answered) 79 (answered) 80 (answered) 81 (answered) 82 (answered) 83 (answered) 84 (answered) 85 (answered) 86 (answered) 87 (answered) 88 (answered) 89 (answered) 90 (answered) 91 (answered) 92 (answered) 93 (answered) 94 (answered) 95 (answered) 96 (answered) 97 (answered) 98 (answered) 99 (answered)100 (answered)101 (answered)102 (answered)103 (answered)104 (answered)105 (answered)106 (answered)107 (answered)108 (answered)109 (answered)110 (answered)111 (answered)112 (answered)113 (answered)114 (answered)115 (answered)116 (answered)117 (answered)118 (answered)119 (answered)120 (answered)


Because of his Maysville Road veto, Andrew Jackson was expected to support

a. federal money for internal improvements

b. the Kentucky Resolution

c. the Force Bill

d. the tariff of 1828

e. the Ordinance of Nullification



The correct answer is E. The Ordinance of Nullification, answer E, said that any state had the right to declare a federal law unconstitutional and, therefore, didn't have to abide by it. Jackson was a supporter of states' rights to a point, as his veto of the Maysville Road bill had demonstrated, but he was also a strong believer in the Union, which his fellow Southerners didn't understand. Answer A is illogical, since the Maysville Road bill was an appropriation for internal improvements. Answer B refers to an earlier similar attempt at giving states the right to declare federal laws unconstitutional. This earlier document was written by then private citizen Thomas Jefferson. Answer C is illogical; the bill was a threat made to the South. Any secessionist attempt would be put down by force. While Jackson asked for it and supported it, it went against states' rights, the issue in the Maysville Road bill. Answer D is illogical. The tariff prompted the crisis, and Jackson's support of the tariff flew in the face of states' right.

Return to Section Report Question: 60 of 120
Jump to question: 1 (answered) 2 (answered) 3 (answered) 4 (answered) 5 (answered) 6 (answered) 7 (answered) 8 (answered) 9 (answered) 10 (answered) 11 (answered) 12 (answered) 13 (answered) 14 (answered) 15 (answered) 16 (answered) 17 (answered) 18 (answered) 19 (answered) 20 (answered) 21 (answered) 22 (answered) 23 (answered) 24 (answered) 25 (answered) 26 (answered) 27 (answered) 28 (answered) 29 (answered) 30 (answered) 31 (answered) 32 (answered) 33 (answered) 34 (answered) 35 (answered) 36 (answered) 37 (answered) 38 (answered) 39 (answered) 40 (answered) 41 (answered) 42 (answered) 43 (answered) 44 (answered) 45 (answered) 46 (answered) 47 (answered) 48 (answered) 49 (answered) 50 (answered) 51 (answered) 52 (answered) 53 (answered) 54 (answered) 55 (answered) 56 (answered) 57 (answered) 58 (answered) 59 (answered) 60 (answered) 61 (answered) 62 (answered) 63 (answered) 64 (answered) 65 (answered) 66 (answered) 67 (answered) 68 (answered) 69 (answered) 70 (answered) 71 (answered) 72 (answered) 73 (answered) 74 (answered) 75 (answered) 76 (answered) 77 (answered) 78 (answered) 79 (answered) 80 (answered) 81 (answered) 82 (answered) 83 (answered) 84 (answered) 85 (answered) 86 (answered) 87 (answered) 88 (answered) 89 (answered) 90 (answered) 91 (answered) 92 (answered) 93 (answered) 94 (answered) 95 (answered) 96 (answered) 97 (answered) 98 (answered) 99 (answered)100 (answered)101 (answered)102 (answered)103 (answered)104 (answered)105 (answered)106 (answered)107 (answered)108 (answered)109 (answered)110 (answered)111 (answered)112 (answered)113 (answered)114 (answered)115 (answered)116 (answered)117 (answered)118 (answered)119 (answered)120 (answered)


All of the following statements are true about electoral politics in the antebellum period EXCEPT



a. The rise in the number of voters fueled greater interest in politics and the rise of third parties.

b. The party convention replaced the caucus as a way to nominate candidates.

c. What is known as the modern political campaign—slogans, parades, campaign speeches—came into being.

d. Newspapers published by political parties were highly partisan.

e. Suffrage was still limited by property qualifications in western states.



The correct answer is E. The expansion of voting rights to men regardless of whether they owned property was the single greatest factor in expanding the voting base in this period. The credit for answer C belongs to the election of 1840, considered the first modern election.

Return to Section Report Question: 61 of 120
Jump to question: 1 (answered) 2 (answered) 3 (answered) 4 (answered) 5 (answered) 6 (answered) 7 (answered) 8 (answered) 9 (answered) 10 (answered) 11 (answered) 12 (answered) 13 (answered) 14 (answered) 15 (answered) 16 (answered) 17 (answered) 18 (answered) 19 (answered) 20 (answered) 21 (answered) 22 (answered) 23 (answered) 24 (answered) 25 (answered) 26 (answered) 27 (answered) 28 (answered) 29 (answered) 30 (answered) 31 (answered) 32 (answered) 33 (answered) 34 (answered) 35 (answered) 36 (answered) 37 (answered) 38 (answered) 39 (answered) 40 (answered) 41 (answered) 42 (answered) 43 (answered) 44 (answered) 45 (answered) 46 (answered) 47 (answered) 48 (answered) 49 (answered) 50 (answered) 51 (answered) 52 (answered) 53 (answered) 54 (answered) 55 (answered) 56 (answered) 57 (answered) 58 (answered) 59 (answered) 60 (answered) 61 (answered) 62 (answered) 63 (answered) 64 (answered) 65 (answered) 66 (answered) 67 (answered) 68 (answered) 69 (answered) 70 (answered) 71 (answered) 72 (answered) 73 (answered) 74 (answered) 75 (answered) 76 (answered) 77 (answered) 78 (answered) 79 (answered) 80 (answered) 81 (answered) 82 (answered) 83 (answered) 84 (answered) 85 (answered) 86 (answered) 87 (answered) 88 (answered) 89 (answered) 90 (answered) 91 (answered) 92 (answered) 93 (answered) 94 (answered) 95 (answered) 96 (answered) 97 (answered) 98 (answered) 99 (answered)100 (answered)101 (answered)102 (answered)103 (answered)104 (answered)105 (answered)106 (answered)107 (answered)108 (answered)109 (answered)110 (answered)111 (answered)112 (answered)113 (answered)114 (answered)115 (answered)116 (answered)117 (answered)118 (answered)119 (answered)120 (answered)


The Whig Party was established

a. to support the interests of Northern business

b. to oppose the extension of slavery

c. to support the rechartering of the Second Bank

d. to oppose the policies of Andrew Jackson

e. to support territorial expansion



The correct answer is D. Andrew Jackson was given the name "King Andrew" by his opponents. They took the name Whig from the English political party that formed to oppose Charles II in the second half of the 1600s. While to a greater or lesser degree the other answers represent Whig views, the reason the party was established was answer A.

Return to Section Report Question: 62 of 120
Jump to question: 1 (answered) 2 (answered) 3 (answered) 4 (answered) 5 (answered) 6 (answered) 7 (answered) 8 (answered) 9 (answered) 10 (answered) 11 (answered) 12 (answered) 13 (answered) 14 (answered) 15 (answered) 16 (answered) 17 (answered) 18 (answered) 19 (answered) 20 (answered) 21 (answered) 22 (answered) 23 (answered) 24 (answered) 25 (answered) 26 (answered) 27 (answered) 28 (answered) 29 (answered) 30 (answered) 31 (answered) 32 (answered) 33 (answered) 34 (answered) 35 (answered) 36 (answered) 37 (answered) 38 (answered) 39 (answered) 40 (answered) 41 (answered) 42 (answered) 43 (answered) 44 (answered) 45 (answered) 46 (answered) 47 (answered) 48 (answered) 49 (answered) 50 (answered) 51 (answered) 52 (answered) 53 (answered) 54 (answered) 55 (answered) 56 (answered) 57 (answered) 58 (answered) 59 (answered) 60 (answered) 61 (answered) 62 (answered) 63 (answered) 64 (answered) 65 (answered) 66 (answered) 67 (answered) 68 (answered) 69 (answered) 70 (answered) 71 (answered) 72 (answered) 73 (answered) 74 (answered) 75 (answered) 76 (answered) 77 (answered) 78 (answered) 79 (answered) 80 (answered) 81 (answered) 82 (answered) 83 (answered) 84 (answered) 85 (answered) 86 (answered) 87 (answered) 88 (answered) 89 (answered) 90 (answered) 91 (answered) 92 (answered) 93 (answered) 94 (answered) 95 (answered) 96 (answered) 97 (answered) 98 (answered) 99 (answered)100 (answered)101 (answered)102 (answered)103 (answered)104 (answered)105 (answered)106 (answered)107 (answered)108 (answered)109 (answered)110 (answered)111 (answered)112 (answered)113 (answered)114 (answered)115 (answered)116 (answered)117 (answered)118 (answered)119 (answered)120 (answered)


Which of the following was a call to Americans to develop their own culture independent of European thought?

a. Common Sense

b. "The American Scholar"

c. Walden

d. The Dial

e. "Self-Reliance"



The correct answer is B. Ralph Waldo Emerson first delivered this speech at Harvard and then began to tour the country giving it. Answer A is Tom Paine's call to colonists to rebel against Great Britain. Answer C is Henry David Thoreau's account of living in harmony with nature at Walden Pond. Answer D is the magazine that the transcendentalists published; it was edited by Margaret Fuller. Answer E is a good distracter; it's an essay by Emerson.

Return to Section Report Question: 63 of 120
Jump to question: 1 (answered) 2 (answered) 3 (answered) 4 (answered) 5 (answered) 6 (answered) 7 (answered) 8 (answered) 9 (answered) 10 (answered) 11 (answered) 12 (answered) 13 (answered) 14 (answered) 15 (answered) 16 (answered) 17 (answered) 18 (answered) 19 (answered) 20 (answered) 21 (answered) 22 (answered) 23 (answered) 24 (answered) 25 (answered) 26 (answered) 27 (answered) 28 (answered) 29 (answered) 30 (answered) 31 (answered) 32 (answered) 33 (answered) 34 (answered) 35 (answered) 36 (answered) 37 (answered) 38 (answered) 39 (answered) 40 (answered) 41 (answered) 42 (answered) 43 (answered) 44 (answered) 45 (answered) 46 (answered) 47 (answered) 48 (answered) 49 (answered) 50 (answered) 51 (answered) 52 (answered) 53 (answered) 54 (answered) 55 (answered) 56 (answered) 57 (answered) 58 (answered) 59 (answered) 60 (answered) 61 (answered) 62 (answered) 63 (answered) 64 (answered) 65 (answered) 66 (answered) 67 (answered) 68 (answered) 69 (answered) 70 (answered) 71 (answered) 72 (answered) 73 (answered) 74 (answered) 75 (answered) 76 (answered) 77 (answered) 78 (answered) 79 (answered) 80 (answered) 81 (answered) 82 (answered) 83 (answered) 84 (answered) 85 (answered) 86 (answered) 87 (answered) 88 (answered) 89 (answered) 90 (answered) 91 (answered) 92 (answered) 93 (answered) 94 (answered) 95 (answered) 96 (answered) 97 (answered) 98 (answered) 99 (answered)100 (answered)101 (answered)102 (answered)103 (answered)104 (answered)105 (answered)106 (answered)107 (answered)108 (answered)109 (answered)110 (answered)111 (answered)112 (answered)113 (answered)114 (answered)115 (answered)116 (answered)117 (answered)118 (answered)119 (answered)120 (answered)


Ralph Waldo Emerson and Henry David Thoreau were founders of which American literary movement?

a. deism

b. romanticism

c. realism

d. transcendentalism

e. naturalism



The correct answer is D. Answer D was a purely American movement. Answer A was a religious movement of the 1600s and 1700s that believed that God is the source of natural law and people can discern these laws through rational thought. Answers B, C, and E were literary—and artistic—movements that had their roots in Europe.

Return to Section Report Question: 64 of 120
Jump to question: 1 (answered) 2 (answered) 3 (answered) 4 (answered) 5 (answered) 6 (answered) 7 (answered) 8 (answered) 9 (answered) 10 (answered) 11 (answered) 12 (answered) 13 (answered) 14 (answered) 15 (answered) 16 (answered) 17 (answered) 18 (answered) 19 (answered) 20 (answered) 21 (answered) 22 (answered) 23 (answered) 24 (answered) 25 (answered) 26 (answered) 27 (answered) 28 (answered) 29 (answered) 30 (answered) 31 (answered) 32 (answered) 33 (answered) 34 (answered) 35 (answered) 36 (answered) 37 (answered) 38 (answered) 39 (answered) 40 (answered) 41 (answered) 42 (answered) 43 (answered) 44 (answered) 45 (answered) 46 (answered) 47 (answered) 48 (answered) 49 (answered) 50 (answered) 51 (answered) 52 (answered) 53 (answered) 54 (answered) 55 (answered) 56 (answered) 57 (answered) 58 (answered) 59 (answered) 60 (answered) 61 (answered) 62 (answered) 63 (answered) 64 (answered) 65 (answered) 66 (answered) 67 (answered) 68 (answered) 69 (answered) 70 (answered) 71 (answered) 72 (answered) 73 (answered) 74 (answered) 75 (answered) 76 (answered) 77 (answered) 78 (answered) 79 (answered) 80 (answered) 81 (answered) 82 (answered) 83 (answered) 84 (answered) 85 (answered) 86 (answered) 87 (answered) 88 (answered) 89 (answered) 90 (answered) 91 (answered) 92 (answered) 93 (answered) 94 (answered) 95 (answered) 96 (answered) 97 (answered) 98 (answered) 99 (answered)100 (answered)101 (answered)102 (answered)103 (answered)104 (answered)105 (answered)106 (answered)107 (answered)108 (answered)109 (answered)110 (answered)111 (answered)112 (answered)113 (answered)114 (answered)115 (answered)116 (answered)117 (answered)118 (answered)119 (answered)120 (answered)


The major population group of the Spanish borderlands in the early 1800s was

a. Native Americans

b. Mestizos

c. Spaniards

d. Creoles

e. American settlers



The correct answer is A. Answer E is incorrect. Americans weren't invited to settle in Texas until 1821, nor did Americans trade with California ranchos until the 1820s. Only a few American traders settled in California at that time. Answer B defines a person who has part Native American and part Spanish ancestry. Answer D is a person of only Spanish ancestry. Answer C was the smallest percentage of population living in the borderlands.

Return to Section Report Question: 65 of 120
Jump to question: 1 (answered) 2 (answered) 3 (answered) 4 (answered) 5 (answered) 6 (answered) 7 (answered) 8 (answered) 9 (answered) 10 (answered) 11 (answered) 12 (answered) 13 (answered) 14 (answered) 15 (answered) 16 (answered) 17 (answered) 18 (answered) 19 (answered) 20 (answered) 21 (answered) 22 (answered) 23 (answered) 24 (answered) 25 (answered) 26 (answered) 27 (answered) 28 (answered) 29 (answered) 30 (answered) 31 (answered) 32 (answered) 33 (answered) 34 (answered) 35 (answered) 36 (answered) 37 (answered) 38 (answered) 39 (answered) 40 (answered) 41 (answered) 42 (answered) 43 (answered) 44 (answered) 45 (answered) 46 (answered) 47 (answered) 48 (answered) 49 (answered) 50 (answered) 51 (answered) 52 (answered) 53 (answered) 54 (answered) 55 (answered) 56 (answered) 57 (answered) 58 (answered) 59 (answered) 60 (answered) 61 (answered) 62 (answered) 63 (answered) 64 (answered) 65 (answered) 66 (answered) 67 (answered) 68 (answered) 69 (answered) 70 (answered) 71 (answered) 72 (answered) 73 (answered) 74 (answered) 75 (answered) 76 (answered) 77 (answered) 78 (answered) 79 (answered) 80 (answered) 81 (answered) 82 (answered) 83 (answered) 84 (answered) 85 (answered) 86 (answered) 87 (answered) 88 (answered) 89 (answered) 90 (answered) 91 (answered) 92 (answered) 93 (answered) 94 (answered) 95 (answered) 96 (answered) 97 (answered) 98 (answered) 99 (answered)100 (answered)101 (answered)102 (answered)103 (answered)104 (answered)105 (answered)106 (answered)107 (answered)108 (answered)109 (answered)110 (answered)111 (answered)112 (answered)113 (answered)114 (answered)115 (answered)116 (answered)117 (answered)118 (answered)119 (answered)120 (answered)


A major difference between the Democrats and the Whigs was



a. the Democrats support for limited federal government and the Whigs' support of a broader role for the federal government

b. the Democrats' support for the Second Bank and the Whig's opposition to it

c. the Whigs' support for low tariffs and the Democrats' advocacy of a high protective tariff

d. a power base of Southern planters and the wealthy for the Democrats and support among the middle class for the Whigs

e. support among the Democrats for social reform and a more hands-off philosophy by the Whigs



The correct answer is A. Answers B through E are the opposite of what was true. Democratic positions are stated as Whigs' views and vice versa.

Return to Section Report Question: 66 of 120
Jump to question: 1 (answered) 2 (answered) 3 (answered) 4 (answered) 5 (answered) 6 (answered) 7 (answered) 8 (answered) 9 (answered) 10 (answered) 11 (answered) 12 (answered) 13 (answered) 14 (answered) 15 (answered) 16 (answered) 17 (answered) 18 (answered) 19 (answered) 20 (answered) 21 (answered) 22 (answered) 23 (answered) 24 (answered) 25 (answered) 26 (answered) 27 (answered) 28 (answered) 29 (answered) 30 (answered) 31 (answered) 32 (answered) 33 (answered) 34 (answered) 35 (answered) 36 (answered) 37 (answered) 38 (answered) 39 (answered) 40 (answered) 41 (answered) 42 (answered) 43 (answered) 44 (answered) 45 (answered) 46 (answered) 47 (answered) 48 (answered) 49 (answered) 50 (answered) 51 (answered) 52 (answered) 53 (answered) 54 (answered) 55 (answered) 56 (answered) 57 (answered) 58 (answered) 59 (answered) 60 (answered) 61 (answered) 62 (answered) 63 (answered) 64 (answered) 65 (answered) 66 (answered) 67 (answered) 68 (answered) 69 (answered) 70 (answered) 71 (answered) 72 (answered) 73 (answered) 74 (answered) 75 (answered) 76 (answered) 77 (answered) 78 (answered) 79 (answered) 80 (answered) 81 (answered) 82 (answered) 83 (answered) 84 (answered) 85 (answered) 86 (answered) 87 (answered) 88 (answered) 89 (answered) 90 (answered) 91 (answered) 92 (answered) 93 (answered) 94 (answered) 95 (answered) 96 (answered) 97 (answered) 98 (answered) 99 (answered)100 (answered)101 (answered)102 (answered)103 (answered)104 (answered)105 (answered)106 (answered)107 (answered)108 (answered)109 (answered)110 (answered)111 (answered)112 (answered)113 (answered)114 (answered)115 (answered)116 (answered)117 (answered)118 (answered)119 (answered)120 (answered)


The charge that Jackson and his supporters leveled at the Second Bank was that it

a. gave special privileges to the wealthy

b. was irresponsible in lending money to people to buy western land

c. was unable to back its notes with gold and silver

d. supported a system of "pet banks"

e. issued so much many banknotes that inflation resulted



The correct answer is D. Jackson set up the campaign against the Second Bank as the moneyed few against the multitude of ordinary people with himself as the champion of the latter. Answer B was what many state-chartered banks did after Jackson removed federal deposits from the Second Bank and had them deposited in banks chartered by the states. These banks became known as "pet banks," answer D. They had nothing to do with the Second Bank. Answer C is incorrect. The Second Bank was fiscally sound, but it is true of the state-chartered banks. The same reasoning applies to why answer E is incorrect.

Return to Section Report Question: 67 of 120
Jump to question: 1 (answered) 2 (answered) 3 (answered) 4 (answered) 5 (answered) 6 (answered) 7 (answered) 8 (answered) 9 (answered) 10 (answered) 11 (answered) 12 (answered) 13 (answered) 14 (answered) 15 (answered) 16 (answered) 17 (answered) 18 (answered) 19 (answered) 20 (answered) 21 (answered) 22 (answered) 23 (answered) 24 (answered) 25 (answered) 26 (answered) 27 (answered) 28 (answered) 29 (answered) 30 (answered) 31 (answered) 32 (answered) 33 (answered) 34 (answered) 35 (answered) 36 (answered) 37 (answered) 38 (answered) 39 (answered) 40 (answered) 41 (answered) 42 (answered) 43 (answered) 44 (answered) 45 (answered) 46 (answered) 47 (answered) 48 (answered) 49 (answered) 50 (answered) 51 (answered) 52 (answered) 53 (answered) 54 (answered) 55 (answered) 56 (answered) 57 (answered) 58 (answered) 59 (answered) 60 (answered) 61 (answered) 62 (answered) 63 (answered) 64 (answered) 65 (answered) 66 (answered) 67 (answered) 68 (answered) 69 (answered) 70 (answered) 71 (answered) 72 (answered) 73 (answered) 74 (answered) 75 (answered) 76 (answered) 77 (answered) 78 (answered) 79 (answered) 80 (answered) 81 (answered) 82 (answered) 83 (answered) 84 (answered) 85 (answered) 86 (answered) 87 (answered) 88 (answered) 89 (answered) 90 (answered) 91 (answered) 92 (answered) 93 (answered) 94 (answered) 95 (answered) 96 (answered) 97 (answered) 98 (answered) 99 (answered)100 (answered)101 (answered)102 (answered)103 (answered)104 (answered)105 (answered)106 (answered)107 (answered)108 (answered)109 (answered)110 (answered)111 (answered)112 (answered)113 (answered)114 (answered)115 (answered)116 (answered)117 (answered)118 (answered)119 (answered)120 (answered)


In contrast to previous presidents, Andrew Jackson saw his role as president as

a. interpreter of the Constitution

b. defender of the Constitution

c. mediator between state and federal power

d. the direct representative of the people

e. the direct representative of the states



The correct answer is D. This philosophy helps to explain Jackson's emphasis on ordinary people. He considered Congress the direct representatives of the states, but not the people. Answer C doesn't fit, because Jackson used his power and his influence with the electorate to get done what he wanted, hence, the nickname "King Andrew." He may have interpreted and defended the Constitution, but they were part of his role as representative of the people.

Return to Section Report Question: 68 of 120
Jump to question: 1 (answered) 2 (answered) 3 (answered) 4 (answered) 5 (answered) 6 (answered) 7 (answered) 8 (answered) 9 (answered) 10 (answered) 11 (answered) 12 (answered) 13 (answered) 14 (answered) 15 (answered) 16 (answered) 17 (answered) 18 (answered) 19 (answered) 20 (answered) 21 (answered) 22 (answered) 23 (answered) 24 (answered) 25 (answered) 26 (answered) 27 (answered) 28 (answered) 29 (answered) 30 (answered) 31 (answered) 32 (answered) 33 (answered) 34 (answered) 35 (answered) 36 (answered) 37 (answered) 38 (answered) 39 (answered) 40 (answered) 41 (answered) 42 (answered) 43 (answered) 44 (answered) 45 (answered) 46 (answered) 47 (answered) 48 (answered) 49 (answered) 50 (answered) 51 (answered) 52 (answered) 53 (answered) 54 (answered) 55 (answered) 56 (answered) 57 (answered) 58 (answered) 59 (answered) 60 (answered) 61 (answered) 62 (answered) 63 (answered) 64 (answered) 65 (answered) 66 (answered) 67 (answered) 68 (answered) 69 (answered) 70 (answered) 71 (answered) 72 (answered) 73 (answered) 74 (answered) 75 (answered) 76 (answered) 77 (answered) 78 (answered) 79 (answered) 80 (answered) 81 (answered) 82 (answered) 83 (answered) 84 (answered) 85 (answered) 86 (answered) 87 (answered) 88 (answered) 89 (answered) 90 (answered) 91 (answered) 92 (answered) 93 (answered) 94 (answered) 95 (answered) 96 (answered) 97 (answered) 98 (answered) 99 (answered)100 (answered)101 (answered)102 (answered)103 (answered)104 (answered)105 (answered)106 (answered)107 (answered)108 (answered)109 (answered)110 (answered)111 (answered)112 (answered)113 (answered)114 (answered)115 (answered)116 (answered)117 (answered)118 (answered)119 (answered)120 (answered)


"Nor is the Government to be maintained or our Union preserved by invasions of the rights and powers of the several States. In thus attempting to make our General Government strong we make it weak. Its true strength consists in leaving individuals and States as much as possible to themselves . . . ."
This quotation might have been said by anyone of the following EXCEPT



a. Thomas Jefferson

b. James Madison

c. Andrew Jackson

d. Daniel Webster

e. John C. Calhoun



The correct answer is D. This quotation describes the proper role of the federal government in relation to the states and comes down heavily on the side of the states. It would be consistent with the views of any of the men except Daniel Webster who was a Whig and a nationalist. He is famous for his ringing endorsement of federalism, "Liberty and Union, now and forever, one and inseparable!" The quotation is taken from Andrew Jackson's veto of the Second Bank recharter.

Return to Section Report Question: 69 of 120
Jump to question: 1 (answered) 2 (answered) 3 (answered) 4 (answered) 5 (answered) 6 (answered) 7 (answered) 8 (answered) 9 (answered) 10 (answered) 11 (answered) 12 (answered) 13 (answered) 14 (answered) 15 (answered) 16 (answered) 17 (answered) 18 (answered) 19 (answered) 20 (answered) 21 (answered) 22 (answered) 23 (answered) 24 (answered) 25 (answered) 26 (answered) 27 (answered) 28 (answered) 29 (answered) 30 (answered) 31 (answered) 32 (answered) 33 (answered) 34 (answered) 35 (answered) 36 (answered) 37 (answered) 38 (answered) 39 (answered) 40 (answered) 41 (answered) 42 (answered) 43 (answered) 44 (answered) 45 (answered) 46 (answered) 47 (answered) 48 (answered) 49 (answered) 50 (answered) 51 (answered) 52 (answered) 53 (answered) 54 (answered) 55 (answered) 56 (answered) 57 (answered) 58 (answered) 59 (answered) 60 (answered) 61 (answered) 62 (answered) 63 (answered) 64 (answered) 65 (answered) 66 (answered) 67 (answered) 68 (answered) 69 (answered) 70 (answered) 71 (answered) 72 (answered) 73 (answered) 74 (answered) 75 (answered) 76 (answered) 77 (answered) 78 (answered) 79 (answered) 80 (answered) 81 (answered) 82 (answered) 83 (answered) 84 (answered) 85 (answered) 86 (answered) 87 (answered) 88 (answered) 89 (answered) 90 (answered) 91 (answered) 92 (answered) 93 (answered) 94 (answered) 95 (answered) 96 (answered) 97 (answered) 98 (answered) 99 (answered)100 (answered)101 (answered)102 (answered)103 (answered)104 (answered)105 (answered)106 (answered)107 (answered)108 (answered)109 (answered)110 (answered)111 (answered)112 (answered)113 (answered)114 (answered)115 (answered)116 (answered)117 (answered)118 (answered)119 (answered)120 (answered)


Andrew Jackson did which of the following in an effort to halt land speculation and inflation?

a. eliminated subsid
Which of the following dealt a crushing blow to Native American resistance in the Southeast?

a. Battle of Fallen Timbers

b. Battle of Tippecanoe

c. Battle of Horseshoe Bend

d. Sand Creek Massacre

e. Battle of Wounded Knee
The correct answer is C. Led by Andrew Jackson, a coalition of Tennessee militia and Choctaw defeated the Creek in 1814, ending major Native American resistance in the Southeast. Answer A refers to the defeat of a coalition of Native Americans in the Old Northwest in 1794. Answer B is the defeat of a coalition of Native Americans in the Indiana Territory in 1811. Answer D was an attack by Colorado militia led by Colonel John Chivington against a band of 500 Cheyenne who had surrendered. Answer E was the last battle in the Plains wars.
Opposition to the American System came primarily from

a. the Southeast and New England

b. New England and the western states

c. the western states and the Southeast

d. New England and the Mid-Atlantic states

e. the Mid-Atlantic states and the western states
The correct answer is A. Many in the Southeast considered the internal improvements proposal an infringement on states' rights. There was an element of sectional rivalry in the opposition of New Englanders. They didn't want to lose their influence in national politics to the new western states. However, they objected on the practical grounds that they had banks and roads. Answers B and C are half wrong, because the western states is incorrect. Answer D is also half wrong, because Mid-Atlantic states is incorrect. TIP: In two-part answers like this, you can save yourself some time by reading the first part of each answer and eliminating those that are wrong. Then go back and read the second part of just the answers whose first half is correct. You could have eliminated answers C and E this way. Three seconds here, four second there, and you can gain a minute before you know it.
The Tariff of 1816 was passed

a. in reaction to the inability of income taxes to raise enough money to run the government

b. to make up for revenue lost during the War of 1812

c. in reaction to the dumping of cheap British goods into the United States after the War of 1812

d. as an admission of the failure of the U.S. economy to grow

e. as a stopgap measure when the First Bank's charter ran out
The correct answer is C. The Tariff of 1816 is significant because it was the nation's first protective tariff. The modest tariff schedule passed in 1789 was to raise revenue. Answer A is incorrect. The right to tax income didn't become law until the Sixteenth Amendment was passed in 1913. Answer B is just incorrect. Answer D is incorrect; the tariff was meant to protect fledgling industries that sprang up during the war, but the economy was growing. Answer E is incorrect and illogical. The tariff had nothing to do with the banking system.
Which of the following presidents enjoyed a period with little partisan politics?

a. George Washington

b. John Adams

c. Thomas Jefferson

d. James Monroe

e. Andrew Jackson
The correct answer is D. President James Monroe's administration is known as the "Era of Good Feelings" for its political harmony. Partisan politics began in George Washington's first term, answer A, as a result of the differing philosophies of Alexander Hamilton and Thomas Jefferson and continued through until the end of Madison's second term. Answer E, Andrew Jackson, had two highly partisan terms in office.
Slavery moved from being a sectional issue to a national one

a. when the first territory created from the Louisiana Territory requested statehood

b. when the cotton gin spurred a boom in cotton prices

c. when the slave trade was banned in accordance with the Constitution

d. when the abolitionist movement was founded

e. when newspapers became cheap and easily available
The correct answer is A. Until 1819, slavery was seen as a sectional issue for the South to deal with. Once Missouri requested statehood, the political, economic, and moral divisions between North and South became apparent. Answer C occurred in 1808 with relatively no disturbance to the South or the nation. Answer D is about a decade too soon; Missouri, the first new state made from the Louisiana Territory, requested statehood in 1819. The widespread circulation of newspapers helped publicize the argument over slavery, but did not create it.
The development of the factory system had all of the following effects EXCEPT

a. the relationship among family members remained stable

b. working-class women took in piece work to add to the family income

c. the economic usefulness of the family-based workshop was destroyed

d. woman's role once divorced from earning income became less important in the family

e. many of the necessities women made were now available for purchase
The correct answer is A. The first thing that changed with the introduction of the factory system was the relationship of the father to the other members. Up to this point, work had been done in a family-run workshop in which the father was the artisan and the other members assisted him. When men and sons began to go out to work, this patriarchal influence changed. Answer B is true for a period of time and was also true for lower-middle class because piecework allowed the wife to stay home, which was deemed her proper place.
The United States indirectly helped which European nation when it issued the Monroe Doctrine?

a. Spain

b. Portugal

c. the Netherlands

d. Great Britain

e. France
The correct answer is D. Originally, Great Britain had approached the United States with a proposal to issue a joint declaration of noninterference in the Western Hemisphere. At John Quincy Adams' urging, President James Monroe declined. Adams was Monroe's shrewd secretary of state. Instead he urged Monroe to issue his own policy statement. However, in so doing, the United States also aided Great Britain which looked to the former Spanish colonies as new markets and wanted to ensure that neither Spain nor any other European country tried to gain possession of the new nations. The importance of Portugal, answer B, and the Netherlands, answer C, had declined greatly since the 1500s and the race to gain trade advantages had begun.
Prior to the 1800s, which of the following social classes did not exist?

a. independent farmers and planters

b. planters and industrialists

c. factory workers and industrialists

d. professionals and industrialists

e. slaves and professionals
The correct answer is C. Independent farmers were among the earliest colonists and planters existed by the mid-1660s, so answer A can't be correct. This makes answer B incorrect also. Professionals, answer D, mean doctors, lawyers, and merchants—not shopkeepers—and they, too, were represented early in colonial history. Answer E is incorrect; slavery was enforced by the 1660s. TIP: In two-part questions, remember to go through the answers and read just the first half of each answer, eliminate the wrong answers, and return to the answers where the first part at least is correct. In this case, you'd only have to go back to answer C. This saves you time.
A major victory for early labor unions was winning

a. a ban on the use of immigrants as strikebreakers

b. an eight-hour workday

c. a ban on the use of child labor

d. elimination of the closed shop

e. elimination of imprisonment for debt
The correct answer is E. Imprisonment for debt was a very grave problem for workers. It is estimated that 75,000 people a year were imprisoned for unpaid debts. You would be reasonable to assume that employers would fight hard to keep a strikebreaking tactic like answer A, so that would be easy to eliminate. Answers B and C didn't occur for half a century. Answer D is illogical; a union would want to keep a closed shop. This is one in which all employees must belong to the union.
The election of 1824 was decided in the House of Representatives because



a. Andrew Jackson had a majority of electoral votes but not a majority of popular votes

b. no candidate had a majority of electoral votes

c. John Quincy Adams had a majority of the popular vote, but not a majority of electoral votes

d. Adams had a majority of electoral votes, but not a majority of the popular vote

e. having more than two candidates makes it impossible for anyone to win a majority of popular and electoral votes
The correct answer is B. Who had a majority of the popular votes (it was Jackson) doesn't matter. All that matters in a presidential election is who has the majority of electoral votes. Since neither Adams, Jackson, nor the other two candidates running had won a majority, the election was sent to the House as required by the Twelfth Amendment. Answer E may or may not be true, but is also irrelevant. All that matters is that one candidate has a majority of electoral votes. There have been any number of elections in which more than two parties have had presidential candidates and one candidate has garnered a majority of electoral votes, for example, 1968, 1980, and 1992.
Andrew Jackson called it rotation, whereas his detractors considered it

a. restructuring government

b. the spoil system

c. the Kitchen Cabinet

d. civil service reform

e. democracy in action
The correct answer is B. Or as the saying goes "to the victor, belongs the spoils," that is, the rewards. Rotation was how Jackson described removing incumbent civil servants in the federal government and replacing them with his supporters. According to him, incumbents who were in office a long time tended to place the workings of the bureaucracy above the people they were to serve. You could eliminate some answers if you didn't know the right answer. The question uses the word detractors, so you know you should be looking for a negative answer. Answer A might be negative if the person thought the government was fine the way it was, so hold off deciding on this answer. Answer D and answer E seem like positive things, so eliminate those two answers. Answer C was the name given to Jackson's informal group of advisors, as opposed to the formal Cabinet. That doesn't seem like a likely answer. Even if you have to guess between answers A and B you have a 50-50 chance of being correct.
Because of his Maysville Road veto, Andrew Jackson was expected to support

a. federal money for internal improvements

b. the Kentucky Resolution

c. the Force Bill

d. the tariff of 1828

e. the Ordinance of Nullification
The correct answer is E. The Ordinance of Nullification, answer E, said that any state had the right to declare a federal law unconstitutional and, therefore, didn't have to abide by it. Jackson was a supporter of states' rights to a point, as his veto of the Maysville Road bill had demonstrated, but he was also a strong believer in the Union, which his fellow Southerners didn't understand. Answer A is illogical, since the Maysville Road bill was an appropriation for internal improvements. Answer B refers to an earlier similar attempt at giving states the right to declare federal laws unconstitutional. This earlier document was written by then private citizen Thomas Jefferson. Answer C is illogical; the bill was a threat made to the South. Any secessionist attempt would be put down by force. While Jackson asked for it and supported it, it went against states' rights, the issue in the Maysville Road bill. Answer D is illogical. The tariff prompted the crisis, and Jackson's support of the tariff flew in the face of states' right.
All of the following statements are true about electoral politics in the antebellum period EXCEPT



a. The rise in the number of voters fueled greater interest in politics and the rise of third parties.

b. The party convention replaced the caucus as a way to nominate candidates.

c. What is known as the modern political campaign—slogans, parades, campaign speeches—came into being.

d. Newspapers published by political parties were highly partisan.

e. Suffrage was still limited by property qualifications in western states.
The correct answer is E. The expansion of voting rights to men regardless of whether they owned property was the single greatest factor in expanding the voting base in this period. The credit for answer C belongs to the election of 1840, considered the first modern election.
The Whig Party was established

a. to support the interests of Northern business

b. to oppose the extension of slavery

c. to support the rechartering of the Second Bank

d. to oppose the policies of Andrew Jackson

e. to support territorial expansion
The correct answer is D. Andrew Jackson was given the name "King Andrew" by his opponents. They took the name Whig from the English political party that formed to oppose Charles II in the second half of the 1600s. While to a greater or lesser degree the other answers represent Whig views, the reason the party was established was answer A.
Which of the following was a call to Americans to develop their own culture independent of European thought?

a. Common Sense

b. "The American Scholar"

c. Walden

d. The Dial

e. "Self-Reliance"
The correct answer is B. Ralph Waldo Emerson first delivered this speech at Harvard and then began to tour the country giving it. Answer A is Tom Paine's call to colonists to rebel against Great Britain. Answer C is Henry David Thoreau's account of living in harmony with nature at Walden Pond. Answer D is the magazine that the transcendentalists published; it was edited by Margaret Fuller. Answer E is a good distracter; it's an essay by Emerson.
Ralph Waldo Emerson and Henry David Thoreau were founders of which American literary movement?

a. deism

b. romanticism

c. realism

d. transcendentalism

e. naturalism
The correct answer is D. Answer D was a purely American movement. Answer A was a religious movement of the 1600s and 1700s that believed that God is the source of natural law and people can discern these laws through rational thought. Answers B, C, and E were literary—and artistic—movements that had their roots in Europe.
The major population group of the Spanish borderlands in the early 1800s was

a. Native Americans

b. Mestizos

c. Spaniards

d. Creoles

e. American settlers
The correct answer is A. Answer E is incorrect. Americans weren't invited to settle in Texas until 1821, nor did Americans trade with California ranchos until the 1820s. Only a few American traders settled in California at that time. Answer B defines a person who has part Native American and part Spanish ancestry. Answer D is a person of only Spanish ancestry. Answer C was the smallest percentage of population living in the borderlands.
A major difference between the Democrats and the Whigs was



a. the Democrats support for limited federal government and the Whigs' support of a broader role for the federal government

b. the Democrats' support for the Second Bank and the Whig's opposition to it

c. the Whigs' support for low tariffs and the Democrats' advocacy of a high protective tariff

d. a power base of Southern planters and the wealthy for the Democrats and support among the middle class for the Whigs

e. support among the Democrats for social reform and a more hands-off philosophy by the Whigs
The correct answer is A. Answers B through E are the opposite of what was true. Democratic positions are stated as Whigs' views and vice versa.
The charge that Jackson and his supporters leveled at the Second Bank was that it

a. gave special privileges to the wealthy

b. was irresponsible in lending money to people to buy western land

c. was unable to back its notes with gold and silver

d. supported a system of "pet banks"

e. issued so much many banknotes that inflation resulted
The correct answer is D. Jackson set up the campaign against the Second Bank as the moneyed few against the multitude of ordinary people with himself as the champion of the latter. Answer B was what many state-chartered banks did after Jackson removed federal deposits from the Second Bank and had them deposited in banks chartered by the states. These banks became known as "pet banks," answer D. They had nothing to do with the Second Bank. Answer C is incorrect. The Second Bank was fiscally sound, but it is true of the state-chartered banks. The same reasoning applies to why answer E is incorrect.
In contrast to previous presidents, Andrew Jackson saw his role as president as

a. interpreter of the Constitution

b. defender of the Constitution

c. mediator between state and federal power

d. the direct representative of the people

e. the direct representative of the states
The correct answer is D. This philosophy helps to explain Jackson's emphasis on ordinary people. He considered Congress the direct representatives of the states, but not the people. Answer C doesn't fit, because Jackson used his power and his influence with the electorate to get done what he wanted, hence, the nickname "King Andrew." He may have interpreted and defended the Constitution, but they were part of his role as representative of the people.
Nor is the Government to be maintained or our Union preserved by invasions of the rights and powers of the several States. In thus attempting to make our General Government strong we make it weak. Its true strength consists in leaving individuals and States as much as possible to themselves . . . ."
This quotation might have been said by anyone of the following EXCEPT



a. Thomas Jefferson

b. James Madison

c. Andrew Jackson

d. Daniel Webster

e. John C. Calhoun
The correct answer is D. This quotation describes the proper role of the federal government in relation to the states and comes down heavily on the side of the states. It would be consistent with the views of any of the men except Daniel Webster who was a Whig and a nationalist. He is famous for his ringing endorsement of federalism, "Liberty and Union, now and forever, one and inseparable!" The quotation is taken from Andrew Jackson's veto of the Second Bank recharter.
Andrew Jackson did which of the following in an effort to halt land speculation and inflation?

a. eliminated subsidies to western railroads

b. destroyed the Second Bank

c. issued the Specie Circular

d. established the independent treasury system

e. urged Congress to lower the protective tariff
The correct answer is C. The Specie Circular required that public lands be paid for in gold or silver, not bank notes. This actually worsened the situation and the nation went into a depression, the Panic of 1837. If you didn't know, answer A wouldn't be a good guess, because railroads were just beginning to be built. Ending subsidies to them wouldn't have had much effect. Answer B was one of the factors leading to the land speculation in the first place. Answer D was about the one major accomplishment of Jackson's successor, Martin Van Buren. Answer E wouldn't affect land speculation.
Which of the following courses would most likely have been added to a college's curricula in the 1830's?

a. Latin

b. French

c. English literature

d. engineering

e. biology
The correct answer is D. With the growing interest in industry, expansion, and economic growth, colleges were adding engineering and mechanical courses, especially in the West and South.
The development of the short story as an American literary form was the work of

a. Nathaniel Hawthorne

b. Mark Twain

c. James Fenimore Cooper

d. Bret Harte

e. Henry David Thoreau
The correct answer is A. Both answer B, Mark Twain, and answer D, Bret Harte, are too late in the century to have developed this form. Answer C, James Fenimore Cooper, developed the American novel through his use of American themes in the novel form. Answer E wrote nonfiction works including essays.
If a person was a member of the Hudson River School, he or she was

a. a novelist

b. a painter

c. a poet

d. an architect

e. a journalist
The correct answer is B. The Hudson River School was the first American artistic movement. Its painters such as Thomas Cole and Asher B Durand painted the scenery of the Hudson River Valley.
In the first half of the 1800s, which of the following was a major theme in the work of American artists?

a. everyday life

b. Revolutionary War heroes

c. statesmen

d. the wilderness and the West

e. family portraits
The correct answer is D. While it is not accurate to say that none of the other themes were important or were painted, much of the nation was fascinated by the settling of the frontier. Artists both reflected this fascination and fed it with their paintings.
The major reason that Mexico invited Americans to settle in Texas was to

a. to raise revenue by taxing the new settlers

b. to gain converts for the Roman Catholic Church

c. to build a buffer between Mexicans and Native American in the borderlands

d. to expand the population through immigration

e. to take some of the cotton export business from the U.S. Lower South
The correct answer is C. Historians estimate that in the 1820s about half the population of the Mexican borderlands were Native Americans. Few on the Texas plains were part of the Mexican mission system. Most still roamed free and the Comanches and Kiowas especially made the occasional raid on Mexican settlements. Answers A, B, D, and E are true but were incidental to the main reason. TIP: Be sure to find the key words and phrases in the question.
Which of the following statements is not true about the transportation revolution?

a. A network of roads, canals, and railroads strengthened ties between the Northeast, Midwest, and South.

b. The costs of transporting goods decreased.

c. An increase in markets for farm products resulted in the cultivation of more acreage.

d. Railroad construction became an important new industry.

e. People, goods, and information traveled more quickly and easily between and within regions.
The correct answer is A. The South lagged behind the other two regions in construction of transportation infrastructure. This is one reason why the South lacked effective transportation routes to move troops and supplies during the Civil War. Answer C led to the territorial expansion of the nation.
All of the following contributed to the economic growth of the United States between 1820 and 1860 EXCEPT

a. the increase in population

b. industrialization

c. government loans and favorable legislation

d. the availability of investment capital

e. the importing of cheap raw materials from overseas
The correct answer is E. The United States imported little in the way of raw materials because of its own abundance of natural resources, including fertile farmland. Answer A resulted partly from natural increase and partly from the large influx of immigrants beginning around mid-century.
Which of the following was established as a trade route for merchants?

a. Oregon Trail

b. Santa Fe Trail

c. Mormon Trail

d. California Trail

e. Western Trail
The correct answer is B. Answer B, the Santa Fe Trail, was established in 1821 and was used until 1880 when the Santa Fe Railroad replaced it. Traders sold manufactured goods in exchange for furs and gold. Answers A, Oregon Trail; answer B, Mormon Trail; and answer D, California Trail, all took settlers overland to the West and Far West. Answer E, Western Trail, was a cattle trail from Texas to Nebraska.
All of the following ideas about the Second Great Awakening are true EXCEPT

a. More women than men participated.

b. Ministers in the period were men.

c. Camp meetings were a popular venue for religious services.

d. The renewed interest in religion spurred the growth of social reform movements.

e. In terms of women, this was a movement of upper- and middle-class women.
The correct answer is E. The Second Great Awakening was a movement that attracted upper- and middle-class women. Lower-class women were too busy trying to contribute to the financial well-being of their families as well as take care of them. Answer A is true, but note that the men who were attracted to religious revival meetings were often young single workingmen who found an outlet for their time and energies.
Which of the following worked to improve education for the deaf?

a. Dorothea Dix

b. Horace Mann

c. Samuel Gridley Howe

d. Thomas Gallaudet

e. Mary Lyons
The correct answer is D. Thomas Gallaudet opened the first school for the deal. Answer A, Dorothea Dix, worked to establish asylums and treatment for those with mental illness. Answer B, Horace Mann, was the first secretary of education in Massachusetts and established universal public education. Answer C, Samuel Gridley Howe, established the first school for the blind. Answer E, Mary Lyon, founded Mount Holyoke, one of the earliest colleges for women.
"Our proclamations to the other states of the Mexican Confederation, asking them to support us in our struggle for the restoration of our former rights, and for the protection of the Constitution of 1824, have, as you all know, been without results. . . . No other help remains for us now than our strength and the consciousness that we have seized our arms for a just cause."
This quotation was probably written by



a. Sam Houston

b. Moses Austin

c. Stephen Austin

d. General Antonio López de Santa Anna

e. Miguel Hidalgo
The correct answer is A. This is from a speech to Houston's soldiers in January 1836, about six weeks before the siege of the Alamo. Answer B is illogical, since Moses Austin never lived to move to Texas. Answer C is illogical, since he is not mentioned as a major actor in the fight for Texas independence. Answer D commanded the Mexican Army against the Americans and the Tejanos, their Mexican allies. Answer E is commemorated as the father of the Mexican independence from Spain.
The middle class supported universal public education because

a. it would take immigrant children off the streets

b. it provided an honorable occupation for their daughters until they married

c. education would provide upward mobility for poor children

d. schools would instill civic responsibility and patriotism

e. schools would turn out highly motivated workers
The correct answer is D. Answer A is incorrect, because immigrant children weren't typically on the streets. They were working. Answer B is a true statement, but it is not the reason, nor is answer D, which is why workers supported universal public education. Answer E is not the correct answer and not true. Business owners wanted punctual, hard-working, and docile workers, their reason for supporting public education.
Of all of the following, which contributed the most to the success of the temperance movement?

a. state prohibition laws

b. local option laws

c. changes in social and cultural behaviors connected with alcohol consumption

d. churches' role in the fight against alcohol consumption

e. enlistment of men in the fight against alcohol consumption
The correct answer is C. Answers A, B, D, and E were all means to the end—changing human behavior. Prohibition laws were enacted mostly in New England.
Which of the following had the unintended effect of silencing the abolition movement in the South?

a. publication of The Liberator

b. Nat Turner's rebellion

c. Stono Uprising

d. publication of North Star

e. Denmark Vesey conspiracy
The correct answer is B. Nat Turner led his aborted rebellion the same year that the William Lloyd Garrison began to publish The Liberator. The coincidence convinced Southerners that abolitionists were fomenting insurrection. Up until this time, Southern abolitionists felt free to speak their minds, but once the revolt occurred with the accompanying harsh and somewhat hysterical reaction to it in the South, open organizing and protests were effectively silenced except for the brave few. Answer C occurred in South Carolina in 1739. Answer D was Frederick Douglass' abolitionist newspaper. Answer E was stopped before the rebellion began in Charleston, South Carolina in 1822.
The gag rule was invoked by Congress in the 1830s and 1840s

a. to keep Congressional debate on slavery bills secret

b. to prevent discussion of antislavery petitions

c. to prevent members of Congress from reading newspaper articles into the official record

d. to prevent members of Congress from discussing proceedings outside Congress

e. to limit debate on slavery bills
The correct answer is B. Abolitionists were flooding Congress with antislavery petitions and Democrats passed a resolution, or gag rule, against their admission into the record or their discussion on the floor of either house. John Quincy Adams, the former president now representing Massachusetts in the House, worked tirelessly to have the rule revoked, which was accomplished in 1844.
The views of which of the following divided the abolition movement?

a. Frederick Douglass

b. Lucretia Mott

c. David Walker

d. William Lloyd Garrison

e. Anthony Burns
The correct answer is D. William Lloyd Garrison was an intemperate man who would rather see the Union split than life in a nation that condoned slavery. Answer A, Frederick Douglass, a former slave, preferred to work within the system. Answer B, Lucretia Mott, was also a women's rights' activist. Answer C, David Walker, published Walker's Appeal, a booklet in which he called on African American slaves to rebel against their masters. Answer E, Anthony Burns, was an escaped slave who was caught and returned to his owner as a result of the Fugitive Slave Law. His case created national interest and resulted in several Northern states passing personal liberty laws to flaunt the Fugitive Slave Law.
"Women may, if they exert their talents and the opportunities nature has furnished, obtain an influence in society . . . . They may enjoy the luxuries of wealth, without enduring the labors to acquire it; and the honors of office, without feeling its cares; and the glory of victory, without suffering the dangers of battle."
This sentiments of this quotation found an audience among which of the following groups?



a. American women in general

b. the Southern plantation mistress

c. wives of politicians

d. working women

e. the middle-class wife and mother
The correct answer is E. This is an example of the kind of advice given to middle-class women in women's magazines and manuals. The quotation is from a piece written in 1830 by Sara Josepha Hale, the editor of Ladies Magazine.
All of the following were characteristics of American romanticism in art and literature EXCEPT

a. an interest in the natural world

b. a focus on the individual

c. folklore and folk culture

d. religious subjects

e. feelings as paramount over rational thought
The correct answer is D. Romantics were interested in the supernatural and the mystical but not as represented in traditional religion. Answer B, a focus on the individual, both reflected society's attitude and encouraged its belief in the rugged individual.
The success of lyceums in the antebellum United States was because of

a. a lack of public high schools

b. a lack of public libraries

c. the same impulse for improvement that prompted the reform movements

d. the desire of women isolated in their homes for adult company

e. a national feeling of cultural inferiority
The correct answer is C. The lyceum movement was the first adult education movement in the nation. While some lyceum groups supported the establishment of schools and public libraries, their main aim was self-education. Although answer D may have spurred some women to attend, that was not the reason for the movement's success.
Few scientific advances were made in the United States in the first part of the 1800s because

a. Americans tended to be tinkerers rather than scientists

b. people were too busy trying to survive

c. Americans were interested in practical, mechanical applications rather than pure science

d. science was left to the federal government to subsidize and there was little interest

e. Americans were cut off from scientific advances in Europe
The correct answer is C. Alexis de Tocqueville noted that Americans tended to be practical and to distrust the abstract. Answer A echoes the observation that Americans had a "mechanical genius." Answer D is incorrect, because the government subsidized some scientific expeditions including Lewis and Clark to collect plant, animal, and geological samples. Answer E is incorrect. Americans was no more cut off from European scientists than Benjamin Franklin had been in the previous century.
The resolution of the Oregon boundary dispute adversely affected which of the following?

a. immigrants

b. Mexico

c. Texas

d. Native Americans

e. Democrats
The correct answer is B. With the United States no longer facing the possibility of a conflict with Great Britain over Oregon, President Polk was able to throw the full force of the American military against Mexico. Answers A and D are illogical. Immigrants had been welcome in Oregon before the agreement, so there is no reason to think that would change. There was little reason to think policies toward the Native Americans would change either; they were ill-treated before the agreement and ill-treated afterward. Answer E is incorrect; the Democrats benefited from being the party in power while the United States was gaining 50 percent more territory.
The natural outcome of the abolitionists who believed in political activism was

a. a letter-writing campaign against the Kansas-Nebraska Act

b. a boycott of Southern cotton

c. support for Stephen Douglas

d. formation of the Liberty Party

e. espousal of women's rights
The correct answer is D. The founding of the Liberty Party was a direct result of the split among abolitionists. In 1840 those who believed in working through the political process fielded a presidential candidate who received less than 1 percent of the vote. However, a few districts in the North sent Liberty Party members to the House of Representatives. Answer E can't be correct, because one of the causes that split the abolition movement was the proper role of women.
If the expansion of slavery was the central political issue of presidential elections in the 1850s, the central issue of elections in the 1840s was

a. the economy

b. territorial expansion

c. social reform

d. military strength of the nation

e. immigration
The correct answer is B. The annexation of Texas, the dispute over the border with Canada, and the possibility of adding California to the Union swirled through the campaigns in the 1840s, answer B. The Whigs and the Democrats tried to ignore the issue of slavery in their campaigns, although it was always there in the question of whether a new state would be slave or free.
At mid-century, nativists' reaction to immigrants was based on the former's view of immigrants as

a. a source of cheap labor

b. a threat to American values and traditions

c. the cause of potential overpopulation

d. a drain on public welfare

e. a source of union membership
The correct answer is B. Answer A is true in that competition for work was a motivating factor for the nativists, but their major stated concern, at least, was that immigrants would undermine the American way of life. Answer C was not an issue in a nation that had so much unexplored and unsettled land. Answer D is illogical, because there was no public welfare. Since unions were not very powerful at the time and closed to immigrants, answer E is illogical.
Which of the following was championed by the Free Soil Party?

a. the abolition of slavery

b. maintaining the free state and slave state balance

c. encouraging the migration of free African Americans to western lands

d. banning slavery in all new territories

e. allowing new states to vote on whether they wanted to be a free or a slave state
The correct answer is D. The Free Soil Party's slogan was "Free Soil, Free Speech, Free Labor, Free Men." Those free men didn't include African Americans. Free-Soilers wanted to see slavery banned from new territories, because they believed that the slaveholding South was depressing the wages of free white men. (They didn't seem to realize that women worked.)
Which of the following religions that gained influence in the United Sates was based on communal living and the authority of male leaders?

a. Methodism

b. Presbyterianism

c. Society of Friends

d. Mormonism

e. Calvinism
The correct answer is D. Only Mormonism was based on communal living. Answer A, Methodism, was founded in the 1700s in England and spread to the colonies in the First Great Awakening. Answer B, Presbyterianism, was established during the Reformation as was answer E, Calvinism, named after John Calvin. Answer C, Society of Friends, is the formal name for the group that has come to be known as Quakers. They originated in England in the 1600s.
All of the following dealt with the question of extending slavery to new territories EXCEPT

a. Compromise of 1850

b. Tallmadge Amendment

c. Wilmot Proviso

d. Missouri Compromise

e. "Fifty-Four Forty or Fight"
The correct answer is E. This was the slogan and the boundary line aggressive expansionists wanted for the United States–Canadian border. They settled for a boundary line at the forty-ninth parallel. Answer A preserved the fragile balance between free and slave states by admitting California as a free state to offset the admission of Texas as a slave state. Among other things, the package of bills banned the slave trade in the District of Columbia and enacted a Fugitive Slave Act. Answer B proposed by James Tallmadge of New York in 1819 would have gradually abolished slavery in Missouri and banned it in all states made from the Louisiana Territory; it was defeated. Answer C was proposed by David Wilmot of Pennsylvania in 1846 and again in 1847 to forbid the extension of slavery into any state created from territory purchased from Mexico. Answer D, also known as the Compromise of 1820, set the slave–free state boundary at 36º30'N. With the exception of Missouri, any state created from the Louisiana Territory north of this line would be free and any state below it would be slave.
The Fort Laramie Treaty of 1851 was significant for all of the following reasons EXCEPT

a. the federal government acknowledged the Plains Native Americans' loss of buffalo and grasslands

b. the federal government agreed to pay $50,000 a year in trade goods as compensation

c. the federal government adopted an official policy of separating Native Americans and European Americans on the Plains

d. the Plains Native Americans who signed the treaty agreed to give up hunting for farming

e. Plains Native Americans signatories agreed to live within clearly defined boundaries
The correct answer is D. There were two treaties signed at Fort Laramie. If you missed the date in this question stem and thought it asked about the 1868 treaty, you'd wonder about the test writer. Turning Native Americans into farmers didn't become government policy until the Dawes Act of 1887. (The 1868 Treaty of Fort Laramie ended the Second Sioux War and recognized the Sioux' rights to western South Dakota.) TIP: Be sure to read questions carefully.
Which of the following motivated women to seek greater rights for themselves?

a. subservient role male abolitionists expected women to take in the movement

b. grant of the right to vote to freed male African Americans

c. jailing of Susan B. Anthony for attempting to vote in an election

d. the promises of the Constitution

e. the freedom of economic choice that industrialization offered
The correct answer is A. The proper place for respectable women in the abolition movement was behind the scenes—or the curtain in the case of Elizabeth Cady Stanton and Lucretia Mott at the World Anti-Slavery Convention in 1840. After the Civil War, women who had put a hold on their fight for suffrage in an effort to end slavery were told to work for the vote for freed African American men. Answer C actually happened and she was convicted, but did not serve her sentence. Answers D and E are vague; beware of vague answers
All of the following were true about free African Americans in the antebellum South EXCEPT

a. they were discriminated against in housing

b. skilled workers were unable to find work

c. they could not testify against a white person in court

d. they could neither vote nor hold public office

e. they could not move from one slave state into another
The correct answer is B. Because of the inability of the South to attract white labor, skilled African Americans such as carpenters and blacksmiths were able to find work in Southern cities. There were more skilled free African American workers in the South than in the North. Some were former slaves who had been trained by their owners and then bought their freedom by saving money from jobs they did on the side for other whites.
By the mid-nineteenth century, most immigrants to the United States were

a. English

b. Irish

c. Scot Irish

d. German

e. Mexican
The correct answer is B. The largest group was the Irish, but the Germans, answer D, were the second largest group in this period and overall the largest group to immigrate between 1820 and 2000. Answer C, Scot Irish, would have been correct if the question had asked about immigrants in the 1700s. Answer E, Mexican, is incorrect for this period, but the largest group to immigrate between 1980 and 2000.
Which of the following is NOT an accurate statement about the South's economy between 1840 and 1860?

a. The number of families owning slaves increased.

b. Industrial development stagnated because capital was tied up in land and labor.

c. The Border States saw an increase in free labor and a decrease in slave labor.

d. Slave labor was becoming concentrated in the Lower South.

e. Economic opportunities for non-slave holding whites were limited in the agrarian South.
The correct answer is A. The number of families owning slaves dropped from 31 percent to 25 percent during this period. While Southerners defended slavery with great emotion, the economic development of the South was slipping further behind the rest of the nation.
The cult of domesticity refers to

a. an association for household help

b. a type of architecture

c. a characteristic of middle-class religion in the antebellum period

d. the proper role of women

e. style of household decoration
The correct answer is D. As work moved from the artisan workshop to the factory and office, the role of men and women changed. Men went out to work to support the family and women stayed home to take care of the family and home, and thus was born the cult of domesticity, which elevated women and their roles. Women were to be the guardians of the morals of society and the inculcators of these virtues into their children, while at the same time attempting to tame the immoral passions of their husbands—or so the manuals of housework, sentimental novels, and moral tracts proclaimed.
The largest number of whites in the antebellum South were

a. plantation owners

b. middle-class urban professionals

c. small farmers who owned slaves

d. non-slave owning farmers

e. laborers
The correct answer is D. The largest group of Southern whites owned no slaves, which is what made the South's defense of slavery seem so illogical. About 75 percent of white families had no slaves. Common sense eliminates answer A, because in any area at any time, the wealthy are only a small group. Answer B, lawyers, doctors, and merchants, would also be a small group compared to the general population. Answer E wouldn't be a large group, because labor was done by slaves or white farmers, whether they owned their land or leased it as tenant farmers.
Which of the following was NOT an argument used by Southerners to justify slavery?

a. Slavery was beneficent, because it provided for the lifelong care of slaves.

b. Slaves preferred slavery, because they could not take care of themselves.

c. According to the Bible, slavery was ordained by God.

d. By taking care of their slaves, slave owners were practicing Christian charity and perfecting their own virtue.

e. Slave labor freed whites from toil and menial work.
The correct answer is D. While some slave owners probably deluded themselves into thinking this to be true, it was not one of the arguments Southerners used to justify the existence of slavery. Answer C is, however, an actual argument; it is called the Biblical defense and is premised on the idea that God established a patriarchy and slaves fit into as part of an extended family.
The most controversial aspect of the Declaration of Sentiments was

a. that it demanded the right to vote for women

b. that it was drawn up by women themselves

c. that it was modeled on the Declaration of Independence

d. that it used ideas from the Enlightenment as applied to women

e. that it demanded property rights for women
The correct answer is A. There were 12 resolutions in the document and 11 passed with little or no debate, but the final one demanding the right to vote caused heated dissent. It finally squeaked through—and this was in a convention of women called to demand women's rights. Answer E is incorrect, because slowly since the founding of the new nation various states had been loosening restrictions on women's property rights.
Most immigrants at mid-century settled in which of the following regions of the country?

a. Far West

b. Midwest

c. Upper South

d. Lower South

e. New England and Mid-Atlantic
The correct answer is E. About 80 percent of all immigrant to the United States in the middle of the 1800s settled in New England and the Mid-Atlantic states. You could think through and eliminate at least some of the answers. Answer A would cost a lot of money for immigrants who had already spent a lot of money to get to the United States, so eliminate it as impractical. Answers C and D wouldn't attract many immigrants who had little money, because there was little work to be had in these areas where the agricultural economies were based on slave labor.
All of the following were push factors that motivated immigrants to leave their home countries EXCEPT

a. political instability

b. religious persecution

c. famine

d. financial backing from joint-stock companies

e. inability to make a living in agriculture
The correct answer is D. Wrong century. Joint-stock companies operated in the 1600s to back colonization ventures. Answer A relates to Germans and the French. Answer B relates to Jews and Catholics. Answer C refers mainly to the Irish. Answer E was a reason for Italian immigration in the latter half of the 1800s.
A major stumbling block that early unions faced was

a. the reluctance of women to join unions

b. the fact that unions were organized by craft

c. the lack of public sympathy

d. court rulings that held strikes were criminal conspiracies

e. their own fear of African Americans and immigrants as competition
The correct answer is D. In Commonwealth v. Hunt, the Massachusetts Supreme Court ruled that unions were not necessarily criminal conspiracies. Prior rulings that they were in violation of the law had held unions back from striking for their demands. Answer A is incorrect; women weren't allowed to join unions. Answer E may be objectively true, but was not a consideration of the early unions.
Cultural tastes in the antebellum United States were set by and large by

a. the Southern planter elite

b. New York high society

c. the Brahmins of Boston

d. the Northern middle class

e. Midwesterners
The correct answer is D. Answers A, B, and C influenced the middle class to aspire to culture and refinement, but mass tastes were set by the middle class. Answer C is the term given to old-line social elite of Boston.
The Republican Party drew members from all of the following groups EXCEPT

a. northern antislavery Whigs

b. Free-Soilers

c. immigrants

d. Whig industrialists and merchants

e. Know-Nothings
The correct answer is C. While Republicans might have wished to stop the spread of slavery, they were as a group anti-immigrant and anti-Catholic. Know-Nothings felt at home in this party.
Which of the following Supreme Court decisions established the principle that slaves were chattel?

a. Plessy v. Ferguson

b. Scott v. Sandford

c. Gibbons v. Ogden

d. McCulloch v. Maryland

e. Gideon v. Wainwright
The correct answer is B. Answer B is also known as the Dred Scott decision. Answer B established the principle of "separate but equal" facilities for African Americans, which was overturned in 1954 in Brown v. Board of Education. Answer C was an early interstate commerce case that established the superiority of federal authority over states' rights under the Constitution. Answer D refers to an attempt by a state to tax a federal activity. The decision enlarged the powers of Congress under implied powers. Answer E established the rights of all prisoners to be represented by an attorney.
"Can the people of a Territory in any lawful way, against the wishes of any citizen of the United States, exclude slavery from their limits prior to the formation of a State constitution? I answer emphatically . . . . that in my opinion the people of a Territory can, by lawful means, exclude slavery from their limits prior to the formation of a State constitution."
The above quotation was probably spoken by



a. Abraham Lincoln

b. Stephen A Douglas

c. John C Calhoun

d. Frederick Douglass

e. John Brown
The correct answer is B. The quotation is from what is known as the Freeport Doctrine after the town where Douglas and Abraham Lincoln were debating in the 1858 Senate campaign in Illinois. Douglas's opinion was in contrast to the decision in Sandford v. Scott in which the Supreme Court appeared to rule that a territory could not ban slavery. Douglas' doctrine hurt him badly with fellow Democrats in the South and cost him the 1860 presidential election. Answer A baited Douglas into giving this answer. Answer C is illogical; Calhoun was a diehard supporter of slavery
Which of the following was a problem that the Confederacy had, but the Union did not?

a. lack of a well-trained officer corps

b. inflation

c. states who put their interests above the national interest

d. fighting an offensive war rather than a defensive war

e. lack of support on the home front
The correct answer is C. The problem with trying to join 11 states that believe in states' right is getting them to cooperate. States refused to pay taxes, raise units for the army, and contribute supplies to the national effort. Answer A was a problem for the Union, not the Confederacy. The best officers like Lee served on the side of the Confederacy. Answer B was a problem for both sides. Answer D is incorrect; with the one exception of the drive into Pennsylvania in 1863, the South fought a defensive war. Answer E is not a true statement
Which of the following is NOT a true statement about British attitudes toward the Confederacy?



a. Great Britain took neither side, declaring itself neutral

b. Great Britain badly needed Southern cotton for its mills or it would face an economic depression.

c. Members from different segments of British society condemned slavery in the South.

d. Among some British aristocrats there were feelings of sympathy for Southern planters.

e. Some British politicians calculated that Great Britain would face less competition on the world stage if the United States was divided into two small nations.
The correct answer is B. While some mill owners were concerned about losing their source of cotton, answer B, in reality Great Britain had other sources, such as Egypt and India. Note the difference between this statement and answers C, D, and E. The others have qualifiers such as "members from different," "among some," and "some British." TIP: Beware of answers that use absolutes like never and always, and in this case, positive and negative adjectives or adverbs.
Lincoln's purpose in issuing the Emancipation Proclamation was

a. to free slaves in Southern states immediately

b. to free slaves in border states

c. to pressure the Confederacy into surrendering before January 1, 1863.

d. to encourage slaves to escape to the Union with the promise of freedom

e. to free slaves in Southern states and those parts of states under Union control
The correct answer is C. The Emancipation Proclamation actually freed no one. The Union had no power to free slaves in states under Confederate control, so answer A is illogical. Even when it went into effect, the Proclamation had no force. Answer B is illogical, because Lincoln had held off in issuing such a document for fear of driving the border states into the Confederacy.
The Wade-Davis Bill included all of the following provisions EXCEPT

a. a majority of white males had to take an oath of allegiance to the United States

b. African Americans had to be given the vote

c. new state constitutions had to abolish slavery

d. approval of new state constitutions rested with Congress

e. delegates to state constitutional conventions had to swear they had never aided the Confederacy in any way
The correct answer is B. There was no thought at this point to give freed African Americans the right to vote. Answer A was a divergence from Lincoln's plan; he had required that only 10 percent take the oath. Answer E was an effort by Republicans to see that the conventions were heavily pro-Union.
Andrew Johnson vetoed the Civil Rights Act of 1866 because he said it

a. was unnecessary because of the Thirteenth Amendment

b. usurped the power of the president

c. unfairly excluded Native Americans

d. violated states' rights and was, therefore, unconstitutional

e. used military courts in time of peace
The correct answer is D. Others in Congress came to share Johnson's view and drafted and passed on to the states the Fourteenth Amendment, which granted citizenship to African Americans. Answer A abolished slavery, whereas the Civil Rights Act dealt with citizenship. Answer C is a true statement about the Fourteenth Amendment, but would not have been of much concern in 1866. Answer E is untrue about the Act.
One of the leading Congressional advocates of Radical Reconstruction was

a. Blanche K. Bruce

b. David Wilmot

c. Daniel Webster

d. Charles Sumner

e. Edward M. Stanton
The correct answer is D. Charles Sumner, answer D, along with his Senate colleague Thaddeus Stevens were the leading Radical Republicans. Sumner proposed the idea of giving "40 acres and a mule" to all freed African American slaves. Both he and Stevens were instrumental in the impeachment of Andrew Johnson. Answer A, Blanche K. Bruce, was the first African American elected to the Senate. He represented Mississippi, but was not elected until 1874. By that time, Radical Reconstruction had run its course. Answer B, David Wilmot, was the author of the Wilmot Proviso in 1846 and 1847. Answer C, Daniel Webster, had been active in politics at the time of the Compromise of 1820, so it would be logical to consider he was dead by 1866. Answer E, Edward M. Stanton, was Johnson's secretary of war. Johnson's dismissal of him in violation of the Tenure of Office Act was the direct cause of the impeachment action against Johnson. Stanton was an ally of the Radical Republicans.
Businessmen were natural supporters of the Republican Party in the years after the Civil War, because the Party supported all of the following EXCEPT

a. low taxes

b. subsidies for western railroads

c. protective tariff

d. restrictions on immigration

e. sound national banking system
The correct answer is D. While some Americans didn't like the waves of immigrants that were coming to the United States in the mid- and latter 1800s, no attempt was made to stop them, because their labor was needed to expand the economy. If you weren't sure of the answer, you could go down the answers and determine that answers A, B, C, and E were all good for business and, therefore, wrong answers. This is a reverse answer question; you want the thing that doesn't fit.
Removal of the last federal troops from the South was the condition for

a. the South's ratification of the Fifteenth Amendment

b. repeal of the black codes

c. Democratic support for Rutherford B. Hayes as the winner in the disputed election of 1876

d. Congressional release of subsidies for Southern railroads

e. Southern enforcement of laws against the Ku Klux Klan
The correct answer is C. This was the Compromise of 1877. Democrats also got a Southern Cabinet member. Answer A had been ratified in 1870. Answer B and E never happened. Answer D is incorrect; Southern states funded their own railroads through bonds and subsidies.